Haryana Board Class 7 Maths Solutions For Chapter 13 Visualising Solid Shapes

Haryana Board Class 7 Maths Solutions For Chapter 13 Visualising Solid Shapes

Key Concepts

  • Introduction:
    In our day-to-day life, we see several objects around us which have different shapes. One thing common about most of these items is that they all have some length, breadth, and height or depth.
    That is they all occupy space and have three dimensions. Hence they are called three-dimensional shapes.
  • Two-dimensional figures:
    The figures drawn on a paper which have only length and breadth are called two-dimensional figures.
  • Faces, Edges, and Vertices:
    The comers of a solid shape are called its vertices. The line segment of its skeleton are its edges. Its flat surfaces are its faces. The 8 corners of the cube are its vertices.
    The 12 line segments that form the skeleton of the cube are its edges.
    The 6 flat square surfaces that are the skin of the cube are its faces.Faces, Edges and Vertices
  • 2 – dimensional figures

 

2 dimentional figure

  • 3- dimensional figures –

3- dimensional figures
3- dimensional figures 1

3- dimensional figures 2

The sum of number of faces and number of vertices - number of edges

HBSE Class 7 Visualising Solid Shapes Solutions Ex 13.1

Hint: The sum of number of faces and number of vertices – number of edges = 2

i.e, F+V- E = 2

Net: A net is a sort of skeleton outline of a solid that can be folded to make it

  • The same solid can have several types of nets.
  • Solid shapes can be drawn on a flat surface like paper realistically.
  • We call this “2-D representation of a 3-D solid”
  • Drawing solids on a flat surface:
    • Visual illusion: Our drawing surface is a paper, which is flat. When we draw a. solid shape, the images are somewhat distorted, to make them appear three-dimensional. It is a visual illusion.
  • There are two ways of drawing solids on a flat surface. They are
    • oblique sketches,
    • Isometric sketches.
  • Oblique sketches:
    Oblique sketches
    Here is a picture of a cube. It gives a clear idea of how the cube looks like. When seen from the front, We do not see certain faces. In the drawn picture, the lengths are not equal, as they should be in a cube. Such a sketch of a solid is called an “oblique sketch’
  • In an oblique sketch, it is clear that.
    1. The sizes of the front faces and its opposite faces are same.
    2. The edges which are all equal in a cube, appear so in the sketch, though the actual measures of edges are not taken so.
  • Isometric sketches:
    Isometric dot sheet is such a sheet which divides the paper into small equilateral triangles made up of dots or lines. To draw sketches, in which measurements also agree with those ofthe solid, we can use isometric dot sheets
  • Isometric means equal measurements.
  • In an isometric sketch, The vertical lines denotes height of the solid and the horizontal lines generally drawn at 30° to the baseline to denote length and width.
  • A shadowplay:
    1. Another way is by observing a 2-D shadow of a 3-D shape.
    2. A third way is to look at the shape from different angles. The ‘front’ view, the side view’ and the ‘top view’ can provide a lot of information about the shape observed.

Haryana Board Class 7 Maths Visualising Solid Shapes solutions

Solutions To Try These

Match the shape with the name:

Match the shape with the name

Solution. (i)- b (11)- d (iii)- a (iv) – c (v) – f (vi) – e

Solutions To Try These

Match the 2-dimensional figures with the names:

Match the 2 dimensional figures with the names

Solution:

(1) – b (2) – a (3) – e (4)- c (5)- d

Solutions For Pratice

Complete the following table:

Complete the following table

Solution:

 

Complete the following table Solution

Solutions To Try These

Here youfind four nets. There are two correct nets among them to make a tetrahedron. See if you can work out which nets will make a tetrahedron.

Solution:

See if you can work out which nets will make a tetrahedron

(1) and (2) are the correct nets for making a tetrahedron

Haryana Board Class 7 Maths Solutions For Chapter 13 Exercise 13.1

1. Identify make cubes the nets(cut out which copies can of be the used nets to a and try it):

Identify the nets which can be used to make qubesIdentify the nets which can be used to make qubes 1

Solution: Nets (2), (3),(4) and (5) can be used to make cubes.

2) Dice are cubes with dots on each face. Opposite faces of a die always have a total of neven dots on them.

Here are two nets to make dice (cubes). the numbers inserted in each square indicate the number of dots in that box.

the numbers inserted in each square indicate the number of dots in that box

Insert suitable numbers in the blanks, remembering that the number on the opposite faces should total to 7.

Solution:

the numbers inserted in each square indicate the number of dots in that box 1

HBSE 7th Class Visualising Solid Shapes Real-Life Applications

3. Can this be a net for a die?

Explain your answer.

Can this be a net for a Die

Solution:

No, we cannot use the given net as a die Because one pair at opposite face will have 1 and 4 on them whose total is not equal to 7 and another pair of opposite faces will have 3 and 6 on them whose total is also not equal to 7.

HBSE Class 7 Visualising Solid Shapes Chapter 13 Definitions Faces Edges Vertices

4. Here is an incomplete net to making a cube. Complete it in at least two different ways. Remember that a cube has six faces. How many are there in the net here? (Give two separate diagrams. If you like, you may use a squared sheet for easy manipulation )

How many are there in the net here

Solution:

To complete the net for making a cube in at latest two different ways Such that a cube has six faces as follows:

How many are there in the net here box

It is clear that there are three faces in the net.

How to draw 3D shapes on paper Class 7 HBSE

5. Match the nets with appropriate Solids

Match the netS with appropriate Solids

Solution: a -(2); b -(3) ;c – (4); d)- (1)

Haryana Board Class 7 Maths Solutions For Chapter 13 Exercise-13.2

1. Use isometric dot paper and make an isometric sketch for each one of the given shapes:

(1)

Use isometric dot paper and make an isometric sketch for each one of the given shapes 1

Top Front Side Views of Solids Class 7 Haryana Board

(2)

Use isometric dot paper and make an isometric sketch for each one of the given shapes 2

(3)

Use an isometric dot paper and make an isometric sketch for each one of the given shapes

(4)

Use isometric dot paper and make an isometric sketch for each one of the given shapes 4

Solution:

(1)

Use isometric dot paper and make an isometric sketch for each one of the given shapes Solution 1

(2)

Use isometric dot paper and make an isometric sketch for each one of the given shapes Solution 2

(3)

Use isometric dot paper and make an isometric sketch for each one of the given shapes Solution 2

Faces, edges, and vertices of 3D shapes Class 7 HBSE

(4)

Use isometric dot paper and make an isometric sketch for each one of the given shapes Solution 4

2. The dimensions of a cuboid are 5 cm, 3cm and 2 cm. Draw three different isometric sketches of this cuboid.

Solution:

(1)

Isometric sketches

(2)

isometric sketches of this cuboid

(3)

Isometric sketches for the figure given

The above are the three different isometric sketches of cuboid of dimensions 5 cm, 3 cm and 2 cm. By changing length, breadth andheight we get the above sketches.

Drawing Oblique Sketches Class 7 HBSE Solutions

3. Three cubes each with 2 cm edge are placed side by side to form a cuboid. Sketch an oblique or isometric sketch of this cuboid.

Solution: Three cubes each with 2 cm edge are placed side by side to form a cuboid then

length = 2 + 2 + 2 = 6cm

Breadth = 2 cm

Height = 2 cm

Oblique sketch of the cuboid:

Oblique sketch of the cuboid

Isometric sketch of the cuboid :

Isometric sketch of the cuboid

4. Make an oblique sketch for each one of the given isometric shapes.

Make an oblique sketch for each one of the given isometric shapes

Solution: Oblique sketch:

Oblique sketch

5. Give (1) an oblique sketch and (2) an isometric sketch for each of the following:

1) A cuboid of dimensions 5 cm, 3 cm and 2 cm. (Is your sketch unique?)

2) A cube with an edge 4 cm long.

Solution:

(1) Oblique sketches:

1)

Oblique sketches 1-1

2)

Oblique sketches 1-2

(2) Isometric sketches:

1)

Isometric sketches

2)

Isometric sketches 1

Nets of solid shapes examples Class 7 HBSE

Solutions To Try These

Try to guess the number of cubes in the. following arrangements:

Try to guess the number of cubes in the following arrangements

Solution:

The number of cubes in the arrangements are :

(1) 24 cubes (2) 8 ctibes (3) 9 cubes

Solutions To Try These

1. Two dice are placed side by side as shown: Can you say what the total would be on the face opposite to

1) 5 + 6

2) 4 + 3

(Remember that in a die sum of the numbers on opposite faces is 7).

Solution:

Numbers on the opposite face

1) 5 + 6 is 2 + 1

2) 4 + 3 is 3 + 4

Numbers on the opposite face in a dice

2. Three cubes each with 2 cm edge are $ § § placed side by side to form a cuboid. Try to make an oblique sketch and say what could be its length, breadth and height.

Solution:

Length = 2 cm + 2 cm + 2 cm = 6 cm

Breadth = 2 cm

Height =2 cm

Haryana Board Class 7 Maths Solutions For Chapter 13 Exercise-13.3

1. What cross-sections do you get when you give a
(1) vertical cut (2) horizontal cut to the following solids?

1) A brick; 2) A round apple 3) A die 4) A circular pipe 5) An ice cream cone

Solution:

What crosss - sections do you get when you give a following actions for the image

Haryana Board Class 7 Maths Solutions For Chapter 13 Exercise-13.4

1. A bulb is kept binning just above the following solids. Name the shape of the shadows obtainedin each case. Attempt to give a rough sketch of the shadow. (You may try to experiment first and then answer these questions)

Attempt to give a rough sketch of the shadow

(1) A ball (2) A cylindrical pipe (3) A book

Solution:

1)

A ball

2)

A cylindrical pipe

3)

Hexagon

2. Here are shadows of some 3-D objects, when seen under the lamp of an overhead projector. Identify the solid(s) that match each shadow. (There may be multiple answers for these !)

The top view of the cylinder is

Identify the solid(s) that match each shadow.

3. Examine if the following are true statements:

(1) The cube can cast a shadow in the shape of a rectangle.
(2) The cube can cast a shadow in the shape of a hexagon

Solution: (1) True (2) False

Solutions To Try These

1. For each solid, the three views (1), (2), (3) are given. Identify for each solid the corresponding top, front, and side views.

For each solid, the three views

Solution:

1) 1 -> Front 2 -> Side 3 -> Top

2) 1 -> Top 2 -> Side 3 -> Front

3) 1 -> Side 2 -> Front3 -> Top

4) 1 -> Side 2 -> Top 3 -> Front

HBSE Class 7 Maths Chapter 13 Guide Visualising Solid Shapes

2. Draw a view of each solid as seen from the direction indicated by the arrow.

Draw a view of each solid as seen from the direction indicated by the arrow

Solution:

View of solid as seen from the direction indicated by the arrow:

View of solid as seen from the direction indicated by the arrow

Haryana Board Class 7 Maths Solutions For Chapter 13 Very Short Answer Questions!

1. Give examples of ‘plane figures’

Solution:

The circle, the square, the rectangle, the quadrilateral, and the triangle.

2. Give examples of ‘solid shapes’.

Solution:

The cube, the cuboid, the sphere, the cylinder, the cone, and the pyramid.

3. How many types of sketches of a solids are possible? What are they?

Solution:

There are-two types of sketches of a solid that are possible. They are (1) Oblique sketches (2) Isometric sketches.

Key Questions in Visualising Solid Shapes for Class 7 HBSE

4. What shape is (1) A brick (2) A road roller (3) A sweet laddu

Solution:

(1) A brick- Cuboid
(2) A road roller- Cylinder
(3) A sweet laddu – Sphere

Different views of solid shapes Class 7 Haryana Board

5. Give two examples of the following

(1) Cone (2) Cylinder (3) Cuboid

Solution:

(1) Cone- Ice cream cone; birthday cap
(2) Cylinder – Pillar, road roller
(3) Cuboid- Book, matchbox

6. Write the difference between an oblique sketch and an isometric sketch.

Solution:

An oblique sketch:

It does not have proportional lengths. Still it conveys all important aspects of the appearance of the solid.

An isometric sketch:

It is drawn on an isometric dot paper. In anisometric sketch of the solid the measurements are of exact size.

Haryana Board Class 7 Maths Solutions For Chapter 13 Short Answer Questions

7. Write names of at least 2 objects from day-to-day life, which arc in the shape of the basic 3D shapes given below:

Write names of at least 2 objects from day-to-day life, which arc in the shape of the basic 3D shapes given below

Haryana Board Class 7 Maths Solutions For Chapter 13 Long Answer Questions

8. Given below are the pictures of some objects. Categorise and write their names according to the shape and fill the table with name of it.

Given below are the pictures of some objects

Solution:

Given below are the pictures of some objects. Categorise and write their names according to the shape and fill the table with name of it.

9. Use an isometric dot paper and make an isometric sketch for each one of the given shapes.

1)

Use an isometric dot paper and make an isometric sketch for each one of the given shapes

2)

Use an isometric dot paper and make an isometric sketch for each one of the given shapes 2

3) Use an isometric dot paper and make an isometric sketch for each one of the given shapes 3

Solution:

1) Isometric sketch for the given shape

Isometric sketch for the given shape

2) Isometric sketches for the figure given

Isometric sketches for the figure given

3) Isometric sketches for the figure given

Isometric sketch for the given shape 2

Important Concepts Visualising Solid Shapes Class 7 HBSE

10. The dimensions of a cuboid are 5 cm, 3 cm and 2 cm. Draw two different isometric sketches of this cuboid.

Solution: The two different isometric sketches of the cuboid whose dimensions are 5 cm, 3 cm and 2 cm. are
1)

The two different isometric sketches of the cuboid whose dimensions are 5 cm, 3 cm and 2 cm are 1

2)

The two different isometric sketches of the cuboid whose dimensions are 5 cm, 3 cm and 2 cm are 2

11. Three cubes each with 2 cm edge are placed side by side to form a cuboid. Draw an oblique or isometric sketch of this cuboid.

Solution:

Three cubes each with 2 cm edge are placed side by side to form a cuboid;

1) The oblique sketch of thus formed cuboid

The oblique sketch of thus formed cuboid

2) Isometric sketch of cuboid.

Isometric sketch of cuboid

Haryana Board Class 7 Maths Solutions For Chapter 13 Multiple Choice Answer Questions

1. Match the following.

Match the following

  1. 1- a,2 – d,3 – e, 4-b
  2. 1 – b,2- c,3 – d,4- e
  3. 1 – a,2- c,3 – d,4 -b
  4. 1 – c,2- d,3 – a,4-b

Answer: 1

Practice Problems Visualising Solid Shapes Class 7 Haryana Board with Nets of Solids

2. What is the number on the face opposite to 4 on a die?

  1. 5
  2. 1
  3. 3
  4. 2

Answer: 3

3. What is the horizontal cross-section of cone?

  1. Circle
  2. Right triangle
  3. Rectangle
  4. Square

Answer: 1

4. What is the vertical cross-section of cone?

  1. Circle
  2. Right triangle
  3. Rectangle
  4. Square

Answer: 2

5. When we rotate a right triangle we get a

  1. Cube
  2. Cuboid
  3. Cone
  4. Cylinder

Answer: 3

6. The solid with one curved surface and one flat surface is……..

  1. Cuboid
  2. Cylinder
  3. Cone
  4. Sphere

Answer: 3

7. When we cut a brick horizontally then the shape of the cutting is……..

  1. Square
  2. Rectangle
  3. Triangle
  4. Circle

Answer: 2

You can draw sketches in which measurements also agree with those of a given solid. To do this we need anisometric sheet.

Read the above para and answer the following (8-9).

8. What is the difference between oblique sketch and isometric sketch?

  1. Shape
  2. Faces
  3. Measurements
  4. None

Answer: 3

9. Ravi wants to draw 6 cm, 3 cm, and 2 cm cuboid exactly with these measurements. Which method is suitable?

  1. Oblique sketch
  2. Isometric sketch
  3. Kitchen play
  4. Shadow play

Answer: 2

10. Nani cuts the carrot as shown in figure. What is the shape of cross-section?

Nani cuts the carrot as shown in figure

  1. What is the shape of cross section 1
  2. What is the shape of cross section 2
  3. What is the shape of cross section 3 What is the shape of cross section 4

Answer: 1

11. Pyramid is a ……dimensional object.

  1. 1
  2. 2
  3. 3
  4. Infinite

Answer: 3

12. A point has…….. dimensions.

  1. 1
  2. 2
  3. 3
  4. ‘0’ (zero)

Answer: 4

13. A two-dimensional figure in the following is

  1. Ball
  2. Square
  3. Cylinder
  4. Matchbox

Answer: 2

14. A three-dimensional figure in the following is

  1. Ball
  2. Square
  3. Rectangle
  4. Triangle

Answer: 1

15. Can this be a net for a cube?

Can this be a net for a cube

  1. Yes
  2. No
  3. Sometimes
  4. None

Answer: 2

16. Which of the following is not the net of a cube?

Can this be a net for a cube 1

Answer: A

17. Identify the correct statement

  1. A cube has 8 vertices
  2. A cuboid has 10 faces
  3. A cone has 2 vertices
  4. A cylinder has vertex

Answer: 1

18. One face of a dice has 3 dots then its opposite face contains……..number of dots.

  1. 3
  2. 4
  3. 5
  4. 1

Answer: 2

19. Which solid has four triangular faces and one square face?

Which solid has four triangular faces and one square face

Answer: C

20. is the net diagrani of the following is the net diagram of the following

  1. cylinder
  2. triangular pyramid
  3. cube
  4. square pyramid

Answer: 2

21. The top view of the cylinder is…………..

The top view of the cylinder is

Answer: A

22. Rishi made a cube of side 3 cm with a wire. What is the length of the wire? 

Rishi made a cube of side 3 cm with a wire

  1. 32 cm
  2. 42 cm
  3. 48 cm
  4. 36 cm

Answer: 4

23. The following is the shadow of which 3D object?

The following is the shadow of which 3D object

  1. sphere
  2. cylinder
  3. cuboid
  4. cone

Answer: 3

24. The diagram represents the front view of…………..

The diagram represents the front view of

  1. die
  2. book
  3. ball
  4. pyramid

Answer: 4

25. The solid which has square shaped from front, top view and side view is

  1. cuboid
  2. cylinder
  3. cube
  4. cone

Answer: 3

26. The following has only one flat surface

The following has only one flat surface

Answer: A

27. What is the shape Of resulting figure of the combination of two cubes?

  1. rectangle
  2. cuboid
  3. cylinder
  4. cone

Answer: 2

28. The number of vertices of the figure

The number of vertices of the figure

  1. 4
  2. 5
  3. 6
  4. 7

Answer: 2

29. The net of cylinder is …………

The net of cylinder is

Answer: B

30. How many cubes are there in the figure?

How many cubes are there in the figure

  1. 6
  2. 8
  3. 10
  4. 12

Answer: 2

31. What are the measurements of resulting figure  What are the measurements of resulting figure?

  1. l = 6 cm, b = 3 cm, h = 3 cm
  2. l = 3 cm, b = 3 cm, h = 6 cm
  3. l = 3 cm, b = 6 cm, h = 3 cm
  4. l = 6 cm, b = 6 cm, h = 6 cm

Answer: 1

32. Relavent 3D shape of the net

Relavent 3D shape of the net

Relavent 3D shape of the net

 

Answer: A

Haryana Board Class 7 Maths Solutions For Chapter 13 Fill in the blanks:

33. Plane figures are of……………

Answer: two dimension

34. ……….shapes are of three dimensions.

Answer: Solid

35. The corners of a solid shape are called……..

Answer: vertices

36. The line segments that forms the skeleton of the cube are its……..

Answer: edges

37. The flat surfaces that are the skin of the cube are its………

Answer: faces

38. A …….is a skeleton outline of a solid that can be folded to make it.

Answer: net

39…………is a very useful skill.

Answer: Visualising solid shapes

40. A cylinder has …….faces.

Answer: 2

41. The vertical cut of a circular pipe is a ………..

Answer: circle

42. Ball and laddu are examples for a ……….shape.

Answer: sphere

Shape No. of faces or edges

43. Match the following:

Shape                                                                             No Of Faces Or Edges

1. Cube                                                                              (     )  A) 4

2. Triangular pyramid                                                      (     )  B) 12

3. Triangular prism in the shape of a kaleidoscope      (     )  C) 6

4. Number of edges in a cuboid                                     (     )  D) 8

5. Number of verticesin a cuboid                                   (     )  E) 5

Answer:

1. C 2. A 3. E 4. B 5. D

Haryana Board Class 7 Maths Solutions For Chapter 10 Algebraic Expressions

Haryana Board Class 7 Maths Solutions For Chapter 10 Algebraic Expressions

Key Concepts

  1. Introduction:
    Expressions are a central concept in algebra. x + 3, y- 5, 4x + 5,10y- 5 are some simple algebraic expressions.
  2. Variable:
    A variable can take various values. Its value is not fixed. We use the letters x, y, l, m ………..etc. to denote variables.
  3.  Constant:
    A constant has a fixed value.
    Examples: 4, 100, -17 etc.
  4. Algebraic Expression:
    A number of combination of numbers using the signs of fundamental operations is called an expression.
    We combine variables and constants to make algebraic expressions. For this we use the operations of addition, subtraction, multiplication, and division.
    Examples: 4x + 5, 10y – 20.
    Look at how the following expressions are obtained:

    1. x²,
    2. 2y²,
    3. 3x²- 5,
    4. xy,
    5. 4xy+7
      1. The expression x2 is obtained by multiplying the variable x by itself.
        x × x = x². It is commonly read as squared
        x × x × x=x³. It is read as ‘x’ cubed.
        x, x², x³……..are all algebraic expressions obtainedfrom x.
      2. The expression 2y² is obtained from y = 2y²= 2 × y × y. Here by multiplying y with y, we obtain y² and then we multiply y² by the constant 2.
      3. In 3x² – 5, we first obtain x² and multiply it by 3 to get 3x². From 3x², we subtract 5 to finally arrive at 3x²- 5.
      4. In xy, we multiply the vanable x with another variable y. Thus, x xy = xy.
      5. In,4xy + 7, we first obtain xy, and multiply it by 4 to get 4xy one add 7 to 4xy to get the expression.
  5. Terms of an expression:
    The expression is separated by ‘+’ or ‘ – ‘ into several parts each part along with its sign is known as the term of the expression.
    Example: 8x²- 6xy. The terms in this expression are 8x² and- 6xy.

Solutions To Try These

Describe how the following expressions are obtained.

7xy + 5,x²y, 4X²- 5x 

Solutions:

7xy + 5: We multiply the variable x with another variable y to obtain xy and then multiply by the constant 7 to get 7xy. Adding 5 to 7xy we obtain 7xy + 5.

x²y: Multiply the variable x with itself to obtain x² and then multiply with y to get x²y.

4x²- 5x: Multiply the variable x with itself to obtain x² and. then multiply with 4 to get 4x².

Multiply the variable x with a constant 5 to get 5x. Then subtract 5x from 4x² to get 4x²-5x

Solutions To Try These

1. What are the terms in the following expressions? Show how the terms are formed. Draw a tree diagram for each expression :

  1. 8y + 3x²,
  2. 7mn- 4,
  3. 2x²y.

1) 8y + 3x²

Solution: Terms: 8y, 3x²

The term 8y is formed by multiplying the variable y by 8.

The term 3x² is formed by multiplying 3, x, and x.

Tree diagram

Expression: 8y+3x²

Tree diagram

2) 7mn- 4

Solutions: Terms: 7mn,- 4

The term 7mn is formed by multiplying 7, m, and n. The term – 4 is a constant.

Tree diagram

Expression: 7mn- 4

7mn- 4

HBSE Class 7 Algebraic Expressions Solutions Ex 10.1 Solved

Solution: Terms: 2x²y

The term 2x²y is formed by multiplying

2, x, x and y

Tree diagram

Expression: 2x²y

Tree diagram 2x²y

Haryana Board Class 7 Maths Algebraic Expressions solutions

2. Write three expressions each having 4 terms.

Solution:

(1) 4x²- 3xy + 4x + 13

(2) 3x²- 5y²-+ 7 xy + 8

(3) 5x³- 5x²- 5x- 5

Solutions To Try These

Identify the coefficients of the terms of the following expressions :

(1) 4x-3y

Solutions: 4 is the coefficient of x; – 3 is the coefficient of. y.

2) a + b + 5

Solution: The coefficient of a and b is 1.

3) 2y + 5

Solution: The coefficient of y is 2.

4) 2xy

Solution:

The coefficient of xy is 2.

The coefficient of x is 2y.

The coefficient of y is 2x.

Solutions To Try These

Group the like terms together from the following: 12x, 12, -25x, -25, -25y, 1, x, 12y, y

Solution:

Like terms are 12x, -25x, x

-25y, 12y,y

12, -25, 1

Solutions To Try These

Classify the following expressions as a monomial, a binomial or a trinomial: a, a + b, ab + a + b, ab + a +b- 5, xy, xy + 5, 5x²- x + 2, 4pq – 3q + 5p, 7, 4m- 7n + 10, 4mn + 7.

Solution:

Monomials: a, xy, 7

Binomials: a + b, xy + 5, 4mn + 7

Trinomials : ab + a + b, 5x²- x + 2,

4m- 7n + 10, 4pq- 3p + 5p

Polynomial: ab+a+b-5

Haryana Board Class 7 Maths Solutions For Chapter 10 Exercise-10.1 :

1. Get the algebraic expression in the following cases using variables, constants, and arithmetic operations.

  1. Subtraction of z from y. → y- z
  2. One-half of the sum of numbers, x and \( y \rightarrow \frac{1}{2}(x+y) \)
  3. The number z multiplied by itself. → z²
  4. One-fourth of the product of numbers p and q.→\( \frac{1}{4} \mathrm{pq}\)
  5. Numbers x and y both squared and added. x²+, y²
  6. Number 5 added to three times the product of numbers m and n. → 3mn + 5
  7. Product of numbers y and z subtracted from 10. → 10- yz
  8. Sum of numbers a and b subtracted from their product→ ab-(a + b)

2. (1) Identify the terms and their factors in the following expressions. Show the terms and factors by tree diagrams.

1) x- 3

Solution:

Expression: x-3

Expression x-3

2) 1+x + x²

Solution: Expression: 1 + x + x²

Expression 1+x + x²

3) y – y³

Solution:

Expression: y – y³

y - y³

HBSE Class 7 Maths Chapter 10 Get Algebraic Expressions

3) 2x²y

4) 5xy² + 7x²y

Solution:

Expression: 5xy²+ 7x²y

5xy²+ 7x²y

5) -ab + 2b²-3a²

Solution:

Expression: -ab + 2b²-3a²

-ab + 2b²-3a²

(2) Identify terms and factors in the expressions given below:

1) – 4x + 5

Solution:

Terms: – 4x, 5

Factors : – 4, x; 5

2) – 4x + 5y

Solution:

Terms: – 4 x; 5y

Factors: – 4, x; 5, y

3) 5y + 3y²

Solution:

Terms: 5y, 3y²

Factors: 5, y; 3, y, y

4) xy +2x²y²

Solution:

Terms: xy; 2x²y²

Factors: x, y; 2, x, x, y, y

5) pq + q

Solution:

Terms: pq, q

Factors : p, q; q

6) 1.2ab – 2.4b + 3.6a

Solution:

Terms: 1.2ab; -2.4b; 3.6a

Factors: 1.2, a, b; -2.4, b; 3.6, a

7) \( \frac{3}{4} x+\frac{1}{4} \)

Solution:

Terms: \( \frac{3}{4} x ; \frac{1}{4} \)

Factors: \( \frac{3}{4}, x ; \frac{1}{4} \)

8) 0.1p² + 0.2q²

Solution:

Terms: 0.1p², 0.2q²

Factors: 0.1, p, p; 0.2, q, q

3. Identify the numerical coefficients of terms (other than constants) in the following expressions:

1) 5 – 3t²

Solution:

Term which Is not constant is -3t²

Numerical coefficient is -3

2) 1 + t + t²+ t³

Solution:

Terms which are not constant are t, t²,t³

Numerical coefficients: 1, 1,1

3) x + 2xy + 3y

Solution:

Terms which are not constant are x, 2xy,3y

Numerical coefficients: 1, 2, 3

HBSE 7th Class Algebraic Expressions Word Problems Solutions

4)100m + 1000n

Solution:

Terms which are not constant are 100m 1000n.

Numerical coefficients: 100, 1000

5) – p²q² + 7pq

Solution:

Terms which are not constant are -p²q²;7pq

Numerical coefficients: -1, 7

6) 1.2a + 0.8b

Solution:

Terms which are not constant are 1.2a; 0.8b

Numerical coefficients: 1.2, 0.8

7) 3.14r²

Solution:

Term: 3.14r²

Numerical coefficient is 3.14

How to simplify algebraic expressions Class 7 HBSE

8) 2(l+ b)

Solution:

2(l + b) =2l+ 2b

Terms: 2l; 2b

Numerical coefficients: 2, 2

9) 0.1y + 0.01y²

Solution:

Terms: 0.1y + 0.01y²

Numerical coefficients: 0.1; 0.01

4. a) Identify terms which contain x and give the coefficient of x.

1) y³x + y 

Solution:

Term which contain x is y²x

Coefficient of x is y²

2) 13y² – 8yx

Solution:

Term which contain x is -8 yx

Coefficient of x is- 8y

3) x+y + 2

Solution:

Term which contain x is x

Coefficient ‘of x is 1

4) 5 + z + zx

Solution:

Term which contain x is zx

Coefficient of x is z

5) 1 + x + xy

Solution:

Term which contain x are x, xy

Coefficients of x are 1, y

6) 12xy² + 25

Term which contain x is 12xy²

Coefficient of x is 12y².

7) 7x + xy²

Solution:

Terms which contain x are 7x and xy²

Coefficients of x are 7 and y²

2) Identify terms which contain y² and give the coefficient of y².

1) 8-xy²

Solution:

Term which contain y² is – xy²

Coefficient of y² is – x.

2) 5y² + 7x

Solution:

Term which contain y² is 5y²

Coefficient of y² is 5

3) 2x²y – 15xy² + 7y²

Solution:

Terms which contains y² is -15xy²; 7y²

Coefficients of y² are -15x; 7

5. Classify into monomials, binomials and trinomials.

  1. 4y-7z
  2.  y²
  3. x + y- xy
  4. 100
  5. ab – a – b
  6. 5 – 3t
  7. 4p²q-4pq²
  8. 7mn
  9. z²- 3z + 8
  10.  a² + b²
  11. z² + z
  12. 1 + x + x²

Solution:

Monomials: (2) y², (4)100, (8) 7 mn

Binomials: (1) 4y- 7z,(6) 5 -3t, (7) 4p²q -4pq², (10) a² + b², (11) z² + z

Trinomials : (3) x + y- xy, (5) ab- a-b, (9) z²- 3z + 8, (12)1+ x + x²

6. State whether a given pair of terms is of like or unlike terms.

1) 1,100

Solution: 1, 100 are like terms.

2) \( -7 x, \frac{5}{2} x \)

Solution: \( -7 x, \frac{5}{2} x \) are like terms.

3) -29x, -29y

Solution: -29x, -29y are unlike terms.

4) 14xy, 42yx

Solution: 14xy, 42yx are like terms.

5) 4m²p,4mp²

Solution: 4m²p, 4mp² are unlike terms.

7) 12xz, 12x²z²

Solution: 12xz, 12x²z² are unlike terms.

7. Identify like terms in the following:

1) -xy², – 4yx², 8x², 2xy², 7y, -11x², -100x,-11yx, 20x²y,- 6x², y, 2xy, 3x

Solution:

Like terms are:-

xy², 2xy²;- 4yx², 20x²y;- 8x², -6x², – 11x²; 7y, y; -100x, 3x; -11yx, 2xy

2)10pq, 7p, 8q, -p²q², -7qp, – 100q, -23,12q²p², -5p², 41, 2405p, 78qp,13p²q, qp², 701p²

Solution:

Like terms are :

10pq, -7qp, 78qp; 7p, 2405p; 8q, -100q; -p²q²,12qy;- 23, 41; -5p², 701p²; 13p²q, qp²

Haryana Board Class 7 Maths Solutions For Chapter 10 Exercise-10.2

1. If m = 2, find the value of:

1) m-2

Solution: m-2

Putting m = 2 in m- 2

we get m-2=2-2=0

2) 3m -5

Solution: Putting m = 2 in 3m- 5

we get 3m-5 = 3 x 2-5

=6-5=1

3) 9 -5m

Solution: 9 -5m

Putting m = 2 in 9- 5m

we get 9- 5m = 9- (5 x 2) ,

= 9- 10 = -1

4) 3m² -2m -7

Solution:

Putting m = 2 in 3m²- 2m- 7 we get

3m²- 2m- 7 = 3(2)² -2(2) -7

=3 x 4-2 x 2-7

=12-4-7

=12-11 =1

5) \( \frac{5 m}{2}-4 \)

Solution:

Putting m = 2 in \( \frac{5 m}{2}-4 \)

we get

\( \frac{5 m}{2}-4=\frac{(5 \times 2)}{2}-4 \)

=5-4=1

2. If p- – 2 find the value of:

(1) 4p + 7

Solution:

Putting p = -2 in 4p + 7 we get

4p + 7 = 4(-2) + 7

=-8+7

=-1

2) – 3p² + 4p + 7 

Solution:

Putting p =- 2 in -3p² + 4p + 7 we get

– 3p² + 4p + 7 = – 3(-2)²+ 4 (-2) + 7

= (-3 x 4) + (-4 x 2) + 7

=-12 – 8 + 7

= -20 + 7 = -13

Simplifying Algebraic Expressions Class 7 HBSE Solved Examples

3) -2p³ – 3p² + 4p + 7

Solution:

Putting p = -2 in -2p³- 3p² + 4p + 7 we get

get

-2p³- 3p² + 4p + 7 =

-2 (-2)³- 3 (-2)² + 4 (-2) + 7

=-2 x (-8) -3 x 4-4 x 2 + 7

=16-12-8 + 7

= 23 – 20 = 3

Practice Problems Algebraic Expressions Class 7 Haryana Board with Substitution

3. Find the value of the following expressions, when x = -1:

1) 2x-7

Solution:

Putting x = -1 in 2x- 7 we get

2x- 7 = 2(-1) -7

= -2-7

= -9

2) -x + 2

Solution:

Putting x = -1 in -x + 2 we get

= x + 2 = – (-1) + 2

=1 + 2

= 3

3) x² + 2x +1

Solution:

Putting x = -1 in x² + 2x +1 we get

x²+ 2x +1 = (-1)² + 2 (-1) +1

=1-2+1 =2-2=0

4) 2x²- x- 2

Solution:

Putting x = -1 in 2x²- x- 2 we get

2x²- x- 2 = 2(-1)²- (-1)- 2

=2+1-2 =3-2=1

4. If a = 2 b = -2, find the values of:

1) a² + b²

Solution:

Putting a = 2;b = -2 in a² + b² we get a² + b² = (2)² + (-2)²

-4+4=8

2) a² + ab + b²

Solution: Putting a = 2; b = -2in a² + ab + b² we get

a²+ ab + b²

= (2)² + 2(-2) + (-2)²

=4-4+4

=8-4=4

3) a²- b²

Solution:

Putting a = 2; b = -2 in a²- b² we get

a²-b²=(2)²-(-2)²

=4-4=0

5. When a = 0, b = -1, find the value of the given expressions:

1) 2a + 2b

Solution:

Putting a =0; b = -1 in 2a + 2b we get

2a + 2b = 2(0) +2(-1)

= 0- 2 = -2

Addition and subtraction of algebraic expressions Class 7

2) 2a² + b²+1

Solution:

Putting a = 0; b= -1 in 2a²+ b² +1 we get

2a² + b² +1 = 2(0)² + (-1)² +1

=0+1 +1 =2

3) 2a²b + 2ab² + ab

Solution:

Putting a = 0;b = -1 in 2a²b + 2ab² + ab we get

2a²b + 2ab² + ab =

2(0)² (-1) + 2(0) (-1)² +0(-1)

0+0+0=0

4) a² + ab + 2

Solution:

Putting a = 0;b =-1 in a² + ab+ 2 we get

a² + ab + 2 = (0)² + 0 (-1) + 2

=0+0+2=2

6. Simplify the expressions and find the value if x is equal to 2

1) x + 7 + 4 (x- 5)

Solution:

x + 7 + 4 (x- 5)

= x + 7 + 4x-20

= x + 4x + 7- 20

= 5 x- 13

Putting x = 2 in 5x- 13 we get

5x- 13 = 5(2) -13

=10-13

=- 3

2) 3(x + 2) + 5x- 7

Solution:

3(x + 2) + 5x- 7

= 3x + 6 + 5x- 7

= 3x + 5x + 6 – 7

=8x-1

Putting x = 2 in 8x-1 we get

8x-1 = 8(2) -1

=16-1=15

3) 6x + 5 (x- 2)

Solution:

6x + 5(x- 2)

= 6x + 5x- 10

=11x -10

Putting x = 2 in 11x- 10 we get

11x- 10 = 11(2) – 10

= 22-10 =12

Important Concepts Algebraic Expressions Class 7 HBSE NCERT Based

4) 4(2x- 1) + 3x +11

Solution: 4(2x- 1) + 3x + 11

= 8x- 4 + 3x +11

= 8x + 3x +11 – 4

=11x + 7

Putting x = 2inllx + 7 we get

11x + 7 = 11(2) + 7= 22 + 7= 29

7. Simplify these expressions and find their values if x = 3, a = -1, b = -2.

1) 3x- 5 – x + 9 ‘

Solution: 3x-5-x + 9

=3x-x-5 + 9 = 2x + 4

Putting x = 3 in 2x + 4 we get

2x + 4 = 2(3) + 4

= 6 + 4 = 10

2) 2- 8x + 4x + 4

Solution: 2-8x + 4x + 4

= -8x + 4x + 4 + 2

= -4x + 6

Putting x = 3 in- 4x + 6 we get

-4x + 6 = -4 (3) + 6

= -12 + 6 = -6

3) 3a + 5 – 8a +1

Solution: 3a + 5- 8a +1

= 3a- 8a + 5 +1

= -5a + 6

Putting a = -1 in -5a + 6 we get

-5a + 6- -5 (-1) + 6

= 5 + 6 =11

4) 10- 3b – 4 – 5b

Solution: 10-3b-4-5b

= -3b- 5b + 10- 4

= -8b + 6

Putting b = -2 in – 8b + 6 we get

– 8b + 6 = -8 (-2) + 6

= 16 + 6 = 22

5) 2a -2b -4 -5 + a

Solution:

2a-2b-4-5 + a

= 2a + a -2b- 4- 5

= 3a- 2b- 9

Putting a = -1; b = -2 in 3a – 2b- 9 we get

3a -2b- 9 = (3 (-1) -2 (-2) -9)

= -3 + 4- 9 = – 12 + 4

= -8

8. (1) If z = 10 find the value of z³ – 3(z- 10)

Solution:

z³ – 3 (z- 10)

Putting z = 10 in z³- 3 (z- 10) we get

z³- 3(z- 10) = (10)3- 3(10- 10)

= 1000- 3 x 0

=1000- 0 = 1000

Substituting Values in Algebraic Expressions Class 7 Haryana Board Solutions Ex 10.3

2) If p = -10 find the value of p²-2p -100

Solution:

p²- 2p -100

Putting p = -10 in p²- 2p -100 we get

p²- 2p -100 = (-10)² – 2(-10) – 100

= 100 + 20-100

=120-100 = 20

9. What should be the value of ‘a’ if the value of 2x² + x – a equals to 5, when x = 0 ?

Solution:

2x²+ x-a = 5

Putting x = 0 in 2 x²+ x- a = 5 we get ,

2(0)² + 0-a = 5

-a = 5 1

a =- 5

10. Simplify the expression and find its value when a = 5 and b =- 3.

2(a² + ab) + 3 – ab

Solution:

2(a² + ab) + 3- ab = 2a²+ 2ab + 3- ab

= 2a² + 2ab- ab + 3

= 2a² + ab + 3

Putting a = 5 and b = -3 in 2a²+ ab + 3

2a² + ab + 3 = 2(5)² + (5) (-3) + 3

= 2×25-15 + 3

= 50-15 + 3

= 53-15 = 38

Haryana Board Class 7 Maths Solutions For Chapter 10 Very Short Answer Questions

1. Define

  1. Monomial
  2. Binomial
  3. Trinomial

Solution:

  1. An expression with only one term is called a monomial.
  2. An expression which contain two unlike terms is called a binomial.
  3. An expression which contain three terms is called a trinomial.

2. Define polynomial.

Solution: In general an expression with one or more terms is called a polynomial.

3. Give examples to each

  1. Monomial
  2. Binomial
  3. Trinomial.

Solution:

  1. Monomial: – 3x,- 5m
  2. Binomial : -2x + y,z-3
  3. Trinomial: -a+b+3,x+y+z

4. Find the value of the expression a³- b³ for a = 3;b = 2

Solution:

Substituting a =3;b = 2 in a³- b³ we get

(3)³- (2)³ = 3 x 3 x 3 – 2 x 2 x 2

= 27-8 =19

5. Simplify the expression 4 (2x-1)+3x +11 

Solution:

4 (2x-1)+ 3x + 11 =- 8

= 4 x 2x- 4 x1 + 3x + 11

= 8x-4 + 3x +11

= 8x + 3x + 11 – 4

=11x + 7

Important formulas for algebraic expressions Class 7

6. Write 3 algebraic expressions with 3 terms each.

Solution:

2x² + 3x + 5

px² + q x + r

ax² + bx + c

7. Find the value of the expression – 9x if x = -3.

Solution:

Given x = -3

=  -9x

= -9(-3)

= 27

8. Write the expression whose value is equal to -9 when x = -3.

Solution: -9

= -3×3

= (-3)3

= (x)3 [∵Given x = -3]

= 3x

The required expression is 3x.

Haryana Board Class 7 Maths Solutions For Chapter 10 Short Answer Questions

9. Identify the expressions given below as monomial, binomial, trinomial, and polynomial.

1) 5x² + y + 6

Solution:

In this expression three unlike terms.

So, it is Trinomial.

2) 3xy

Solution: In this expression, only one term

So, it is Monomial.

3) 5x²y + 6x 

Solution: In this expression, two unlike terms

So, it is Binomial.

4) a +4x-xy + xyz

Solution:

In this expression, more than one, unlike terms. So, it is polynomial.

10. Identify and write the like terms in each of following groups.

1) a², b², – 2a², c², 4a

Solution:

a², b², – 2a², c2, 4a are unlike terms

Here, a²,- 2a², are like terms

2) 3a, 4xy, – yz, 2zy •

Solution:

3a, 4xy, – yz, 2zy are unlike terms.

Here, – yz, 2zy are like terms.

3) -2xy², x²y, 5y²x, x²z

Solution:

2xy², and 5y²x, are like terms.

4) 7p, 8pq, -5pq, -2p, 3p

Solution:

7p, 8pq, -5pq, -2p, 3p are unlike terms.

8pq, -5pq, are like terms

7p, -2p, 3p are like terms.

11. Find the value of the following monomials, if x =1.

Given x = 1.

1) -x 

Solution: Consider -x

= -(1)

= -1

2) 4x.

Solution: Consider 4x

= 4(1)

= 4

3) -2X²

Solution: Consider

-2x²

– -2(1)²

= -2(1)

= -2

12. Simplify and find the value of 4x + x- 2x² + x-1 when x = -1.

Solution:

Consider

4x + x – 2x² + x-1

=-2x²+ (4 +1 + 1)x-1

= -2x² + 6x- 1

But given x = -1

= -2(-1)² + 6(-1) -1

= -2(1) -6-1

= -9

13. Write the expression

5x² – 4 – 3x² + 6x + 8 + 5x – 13 in its simplified form. Find its value when x = -2.

Solution:

5x²- 4- 3x² + 6x + 8 + 5x- 13

= (5x²- 3x²) + (6x + 5x) + (8- 4- 13)

= (5 – 3)x² + (6 + 5)x + (8 – 17)

= 2x² +11x – 9

But given x = -2

= 2(-2)² + 11(-2) – 9

= 2(4) – 22- 9

= 8-22-9

= 8-31

= -23

Key Questions in Algebraic Expressions Ex 10.3 for Class 7 HBSE

14. If x = 1, y = 2 find the values of the following expressions.

Given x =1,y = 2

1) 4x-3y + 5

Solution:

4x- 3y + 5

= 4(1) -3(2) +5

= 4-6+5

=’ 9-6 = 3

2) x² + y²

Solution:

Consider

x² + y²

= (1)² +(2)²

= 1+ 4

= 5

3) xy + 3y-9

Solution: Consider

xy + 3y- 9

= (1) (2) +3(2) -9

= 2+6-9

= 8-9 = -1

15. Group the like terms together 12x, 12, 25x, – 25, 25y, 1, x, 12y, y, 25xy, 5x²y, 7xy²,2xy, 3xy², 4x²y.

Solution:

Group A                 12x, 25x, x

Group B                  25y,12y,y

Group C                  25,xy, 2xy

Group D                 5x²y, 4x²y

Group E                 7xy², 3xy²

Group F                 12,1,-25

Haryana Board Class 7 Maths Solutions For Chapter 10 Long Answer Questions

16. State true or false and give reasons for your answer.

1) 7x² and 2x are unlike terms.

Solution: It is True.

Both terms contain the same variable x.

However, their exponents are not same.

In the first term, the exponent of x is

2 and in the second term it is 1.

2) pq² and -4pq² are like terms

Solution: It is True.

Both terms contain the same variables p and q. However, the exponent of p is 1, and exponent of q is 2.

Multiplication of polynomials Class 7 Haryana Board

3) xy, -12x²y and 5xy² are like terms.

Solution: It is false.

The above terms contain the same variables x and y. However, their exponents are not the same.

In the first term, the exponent of x is 1, and second it is 1.

In the second term, the exponent of x is 2, and second it is 1.

In the third term, the exponent of x is ,1 and second it is 2.

17. State whether the algebraic expression given below is monomial, binomial, trinomial or polynomial.

1) y²

Solution: Monomial.

2) 4y-7z

Solution: Binomial

3) 1+x + x²

Solution: Trinomial

4) 7mn

Solution: Monomial

5) a² + b²

Solution: Binomial

6) 100 xyz

Solution: Monomial,

7) ax + 9

Solution: Binomial

8) p²- 3pq +r

Solution: Trinomial

9) 3y² – x²y² + 4x

Solution: Trinomial

10) 7x²-2xy + 9y²-11

Solution: Polynomial

Haryana Board Class 7 Maths Solutions For Chapter 10 Multiple Choice Answer Questions

Choose the correct answers:

1. In the expression 4x + 5 variable is

  1. 4
  2. 5
  3. x
  4. x+5

Answer: 3

2. The coefficient of y in 2y + 5 is

  1. 2
  2. 2y
  3. 5
  4. y + 5

Answer: 1

3. The nth term of the number pattern 11, 21, 31, 41. is

  1. 10n
  2. n + 10
  3. 10 (n + 1)
  4. 10n +1

Answer: 4

4. What is the coefficient of xin 6xy² + 7y is

  1. 6
  2. y2
  3. 6y²
  4. 7

Answer: 3

5. The value of 4z +1 for z = 2 is

  1. 5
  2. 1
  3. 8
  4. 9

Answer: 4

6. Which type of expression is 2x² + 3x +1 ?

  1. monomial
  2. binomial
  3. trinomial
  4. multinomial

Answer: 3

7. How many terms are there in this expression 4x² y4 z?

  1. 1
  2. 2
  3. 3
  4. 4

Answer: 1

8. What is the value of 3P² -5P + 6 at P =1?

  1. 6
  2. 4
  3. 5
  4. 7

Answer: 2

9. Find the value of 6b – 3a for a = 2, b = 1.

  1. 3
  2. 4
  3. 0
  4. 7

Answer: 3

10. Choose the correct matching.

1) \(\frac{2 m}{5} \) at m = 5                  (  ) a) 20

2) x² + 8x at x = 2                                                     (  ) b) 6

3) a² + b at a = 0, b =1                                             (  )(c) 1

4) pq at p = 2, q = 3                                                  (  ) d) 2

  1. i – d,ii – a,iii – c, iv – b
  2. i -b,ii – c,iii – a, iv – d
  3. i – b,ii – a,iii – c, iv – d
  4. i – a,ii – b,iii – d, iv-c

Answer: 1

11. What is the value of the expression 2x²y + xy² + xy at x = ( -1) and y = 2?

  1. -4
  2. -2
  3. -6
  4. -8

Answer: 2

12. Value of x²-y + 2 at x = 0, y = -1 is

  1. -3
  2. 2
  3. 3
  4. 1

Answer: 3

13. What is the coefficient of ‘P²’in 4P²y- 5P

  1. 4
  2. 4y
  3. A or B
  4. None

Answer: 2

14. What do we call the terms with same algebraic factors?

  1. unlike terms
  2. like terms
  3. constants
  4. variables

Answer: 2

15. a², b², c² are called

  1. Like terms
  2. Unlike terms
  3. Numerical terms
  4. None, of these

Answer: 2

16. Which of the following is a trinomial?

  1. 2x
  2. 2a-3b + c
  3. 2x + y

Answer: 3

17. If A = 2x- 4 then- 3A =

  1. -6x +12
  2. -4 + 2x
  3. 8x – 12
  4. 0

Answer: 1

18. If ‘n’ denotes the natural number, then formula for even number is

  1. n +1
  2. 2n +1
  3. 2n
  4. n-1

Answer: 3

19. “The cost of 5 pencils and 7 pens is 50” Expression algebraic form.

  1. 5x + 7y = 90
  2. 5x + 7y = 50
  3. 5y + 7x = 50
  4. 5y + 7x = 90

Answer: 2

20. Value of x²- 5y + 2 at x = 0, y = 3 is

  1. -3
  2.  2
  3. -13
  4. 1

Answer: 3

Haryana Board Class 7 Maths Solutions For Chapter 10 Fill in the blanks:

21…….. are formed from variables and constants

Answer: Algebraic expressions

22. Expressions are made up of……….

Answer: terms

23. A term is a ………..

Answer: product of factors

24. Terms which have the same algebraic factors are called………

Answer: like terms

25. The…..is the numerical or alphabetical factor of the term.

Answer: coefficient

26. Factors containing variables are said to be……….

Answer: algebraic factors

27. Tire value of 7x – 3 at x = 5 is…….

Answer: 32

28. The nth term of tire number pattern 6, 11,16, 21…..is………

Answer: 5n +1

29. Match the following:

1. The perimeter of the equilateral triangle whose side l is                                 (  ) A) 4l

2. The perimeter of a square whose side l is                                                         (  ) B) 2n 

3. The perimeter of a regular pentagon whose sidel is                                        (  ) 3) 3l

4. If a natural number is denoted by n, general form of a even number is        (  ) D) 2n+1

5. If a natural number is denoted by n, the general form of an odd number is (  ) E) 5l

Answer:

1. C 2. A 3. E 4. B 5. D

Haryana Board Class 9 Maths Solutions For Chapter 4 Linear Equations In Two Variables

Haryana Board Class 9 Maths Solutions For Chapter 4 Linear Equations In Two Variables

  • Equation: An equation is a statement in which one expression equals to another expression.
    Eg: 2x + 3 = 7y
    5m – 9 = 0
    3 + 6 = 9
    3\(x^2\)-5x+6=0
  • Linear equation in one variable: An equation with only one variable of degree one is called linear equation in one variable. (or) An equation of the form ax + b = 0, where a, and b are real numbers such that a ≠ 0, is called a linear equation in one variable.
    Eg: 5x + 6 = 7; 3p = -7
  • Standard form: ax + b = 0, where a and b ∈ R and a ≠ 0.
  • Solution: A linear equation in one variable has a number that can satisfy the equation. This numbers are called the solution of the linear equation in one ” variable.
    • Linear equation has unique solution.

Class 9 Maths Chapter 4 Linear Equations In Two Variables Linear Equation

  • Linear equation in two variables
    • An equation with two variables both of degree one is called linear equation in two variables. (or) An equation of the form ax + by + c = 0, where a, b and c are real numbers such that a ≠ 0 and b ≠ 0, is called a linear equation in two variables.
      Eg: 7a + 3b = 12
      2x = 3y – 5
    • Standard form: ax + by + c = 0, where a, b, c ∈ R and a, b ≠ 0.
  • Solution of linear equation in two variables:
    A linear equation in two variables has a pair of numbers that can satisfy the equations. This pair of numbers is called the solution of the linear equation in two variables.

    • There are infinitely many solutions for a single linear equation in two variables.
    • The process of finding solution(s) is called solving an equation.
    • The solution of a linear equation is not affected when
      • the same number is added to (subtracted from) both sides of the equation.
      • both sides of the equation are mutiplied or divided by the same non-zero number.
  • Graphical representation of linear equations:
    • Any linear equation in the standard form ax + by + c = 0 has a pair of solutions (x,y), that can be represented in the coordinate plane.
    • The graph of every linear equation in two variables (ax + by + c = 0) is a straight line.
    • Every point on the graph of a linear equation in two variables is a solution of the linear equation.
    • Every solution of the linear equation is a point on the graph of the linear equation.
    • The linear equation with constant value zero (in ax + by + c = 0, c = 0) passes through origin.
    • An equation of the type y = mx represents a straight line passing through the origin. Certain linear equations exist such that their solution is (0,0).
  • Steps to draw the graph of linear equations in two variables:
    • Step 1: Let the given equation be ax + by + c = 0.
    • Step 2: Make the y as subject. i.e., y = \(-\left(\frac{a x+c}{b}\right)\)
    • Step 3: Take any 2 values (Most probably integrals) to x and calculate the values of y to obtain solutions (ordered pairs).
    • Step 4: Plot the ordered pairs on the graph paper on a suitable
    • Step 5: Draw the line passing through plotted points.
      Now the obtained line represents the equation: ax + by + c = 0.
      Note: We can take more than 2 values to x to get more solutions to check the correctness of the graph.
  • Equations of the lines parallel to the coordinate axes:
    • The equation of the X-axis: y = 0.
    • The equation of the Y-axis: x = 0.
    • The equation of the straight line parallel to X-axis: y = k. It lies at k units from X-axis and passes through (0,k).

Class 9 Maths Chapter 4 Linear Equations In Two Variables Lines Parallel to the coordinate axes

    • The equation of straight line parallel to Y-axis: x = k. It lies at k units from Y-axis and passes through (k, 0).

Class 9 Maths Chapter 4 Linear Equations In Two Variables Straight line parallel

Haryana Board Class 9 Maths Chapter 4 Solutions

Haryana Board Class 9 Maths Solutions For Chapter 4 Linear Equations In Two Variables Exercise – 4.1

Question 1. The cost of a notebook is twice the cost of a pen. Write a linear equation in two variables to represent this statement.

Solution. Let the cost of a notebook = ₹ x

Let the cost of a pen = ₹ y

The cost of a notebook is twice the cost of a pen.

Cost of a notebook = 2 × cost of a pen

⇒ x = 2xy

⇒ x = 2y

⇒ x – 2y = 0

∴ Required equation

Question 2. Express the following linear equations in the form ax + by + c = 0 and indicate the values of a, b and c in each case:

Solution. (1) 2x + 3y = 9.35

Given equation: 2x + 3y = 9.35

⇒ 2x + 3y – 9.35 = 0

⇒ 2x + 3y + (-9.35) = 0

On comparing with ax + by + c = 0,

We get, a = 2,b = 3,c = -9.35

(2) Given equation: x – \(\frac{y}{5}\) – 10 = 0

⇒ \(x+\left(-\frac{1}{5}\right) y+(-10)=0\)

On comparing with ax + by + c = 0,

We get, a = 1, b = [latex-\frac{1}{5}[/latex], c = -10

(3) Given equation: -2x + 3y = 6

⇒ -2x + 3y – 6 = 0

On comparing with ax + by + c = 0,

We get, a = -2, b = 3, c = -6

(4) Given equation: x = 3y

⇒ x – 3y = 0

⇒ x + (-3)y + 0 = 0

On Comparing with ax + by + c = 0,

We get, a = 1, b = -3, c = 0

(5) Given equation: 2x = -5y

⇒ 2x + 5y = 0

⇒ 2x + 5y + 0 = 0

On comparing with ax + by + c = 0,

We get, a = 2, b = 5, c = 2.

(6) Given equation: x = 3y

⇒ x – 3y = 0

⇒ x + (-3)y + 0 = 0

On comparing with ax + by + c = 0,

We get, a = 1, b = -3, c = 0.

(5) Given equation : 2x = -5y

⇒ 2x + 5y = 0

⇒ 2x + 5y + 0 = 0

On comparing with ax + by + c = 0,

We get, a = 2, b = 5, c = 0.

(6) Given equation: 3x + 2 = 0

⇒ 3x + (0)y + 2 = 0

On comparing with ax + by + c = 0,

We get, a = 3, b = 0, c = 2.

(7) Given equation: y – 2 = 0

⇒ 0(x) – 1(y) – 2 = 0

On comparing with ax + by + c = 0,

We get, a = 0,b = 1, c = -2.

(8) Given equation: 5 = 2x

⇒ 2x – 5 = 0

⇒ 2x + (0)y + (-5) = 0

On comparing with ax + by + c = 0,

We get, a = 2, b = 0, c = -5.

Class 9 Maths Chapter 4 Important Questions Haryana Board

Haryana Board Class 9 Maths Solutions For Chapter 4 Linear Equations In Two Variables Exercise – 4.2

Question 1. Which one of the following options is true, and why?

(1) A unique solution

(2) Only two solutions

(3) Infinitely many solutions

Solution. A linear equation in two variables has infinitely many solutions.

So, y = 3x + 5 has infinitely many solutions. So, option (3) is correct.

Question 2. Write four solutions for each of the following equations:

(1) 2x + y = 7

Solution. Given equation: 2x + y = 7

⇒ y = 7 – 2x

(a) Let x = 0 ⇒ y = 7 – 2(0) = 7 – 0 = 7.

Here solution = (0,7).

(b) Let x = 1 ⇒ y = 7 – 2(1) = 7 – 2 = 5.

Here solution = (1,5)

(c) Let x = -1 ⇒ y = 7 – 2(-1) = 7 + 2 = 9.

Here solution = (-1,9).

(d) Let x = 2 ⇒ y = 7 – 2(2) = 7 – 4 = 3.

Here solution = (2, 3).

∴ The solutions are (0,7), (1, 5), (-1, 9), (2, 3).

(2) πx + y = 9

Solution. Given equation: πx + y = 9

⇒ y = 9 – πx

(a) Let x = 0 ⇒ y = 9 – π(0) = 9 – 0 = 9.

Here solution = (0,9)

(b) Let x = 1 ⇒ y = 9 – π(1) = 9 – π

Here solution = (1, 9 – π)

(c) Let x = -1 ⇒ y = 9 – π(-1) = 9 + π

Here solution = (-1, 9 + π)

(d) Let x = 2 ⇒ y = 9 – π(2) = 9 – 2π

Here solution = (2, 9 – 2π)

∴ The solutions are (0,9), (1, 9 – π), (-1, 9 + π), (2, 9 – 2π)

(3) x = 4y

Solution. Given equation: x = 4y

⇒ y = \(\frac{x}{4}\)

(a) Let x = 0 ⇒ y = \(\frac{0}{4}\) = 0

Here solution = (0,0).

(b) Let x = 1 ⇒ y = \(\frac{1}{4}\)

Here solution = (1,7).

(c) Let x = 4 ⇒ y = \(\frac{4}{4}\) = 1

Here solution (4, 1).

(d) Let x = 2 ⇒ y = \(\frac{2}{4}\) = \(\frac{1}{2}\)

Here solution = (2, \(\frac{1}{2}\))

∴ The solutions are (0,0), (1, \(\frac{1}{4}\)), (4,1), (2,\(\frac{1}{2}\)).

Step-by-step Solutions for Class 9 Maths Chapter 4 Haryana Board

Question 3. Check which of the following are solutions of the equation x – 2y = 4 and which are not:

(1)(0, 2)

(2) (2,0)

(3) (4,0)

(4) (√2,4√2)

(5) (1,1)

Given equation: x – 2y = 4

Solution. (1) (0,2)

Substitute x = 0 and y = 2

LHS = x – 2y = 0 – 2(2) = 0 – 4 = -4 ≠ RHS

∴ (0, 2) is not a solution of the given equation.

(2) (2, 0)

Substitute x = 2 and y = 0

LHS = x – 2y = 2 – 2(0) = 2 – 0 = 2 ≠ RHS

∴ (2, 0) is not a solution of the given equation.

(3) (4,0)

Substitute x = 4 and y = 0

LHS = x – 2y = 4 – 2(0) = 4 – 0 = 4 = RHS

∴ (4,0) is a solution of the given equation.

(4) (√2,4√2)

Substitute x = √2 and y = 4√2

LHS = x – 2y = √2 – 2(4√2)

= √2 – 8√2 = -7√2 ≠ RHS

∴ (√2, 4√2) is not a solution of the given equation.

(5) (1,1)

Substitute x = 1 and y = 1

LHS = x – 2y = 1 – 2(1) = 1 – 2 = -1 ≠ RHS

∴ (1, 1) is not a solution of the given equation.

Question 4. Find the value of K, if x = 2, y = 1 is a solution of the equation 2x + 3y = K.

Solution. Given equation 2x + 3y = K.

x = 2, y = 1 is a solution.

on substituting x = 2 and y = 1.

⇒ 2(2) + 3(1) = K

4 + 3 = K

7 = K

∴ K = 7

Haryana Board Class 9 Maths Solutions For Chapter 4 Linear Equations In Two Variables Very Short Answer Type Questions

Question 1. Define linear equation in two variables.

Solution. An equation of the form ax + by + c = 0, where a, b and c are real numbers such that a ≠ 0 and b ≠ 0, is called a linear equation in two variables.

Question 2. Panth and Pandya scored 125 runs together. Express the information in the form of an equation.

Solution. Let the runs scored by Panth = x

Let the runs scored by Pandya = y

Total runs = 125

⇒ x + y = 125

Graphical Method of Solving Linear Equations Class 9 Haryana Board

Question 3. Express each of the equation y = 3 in the form of ax + by + c = 0 and write the values of a, b and c.

Solution. Given equation : y = 3

⇒ 0.x + y + (-3) = 0

On comparing with ax + by + c = 0,

We get, a = 0, b = 1, c = -3.

Question 4. Express each of the equation x – 5 = √3y in the form of ax + by + c = 0 and write the values of a, b and c.

Solution. Given equation: x – 5 = √3y

⇒ x – √3y – 5 = 0

⇒ x +(-√3)y + (-5) = 0

On comparing with ax + by + c = 0,

We get, a = 1, b = -√3,c = -5.

Question 5. How many solutions does a linear equation in two variables have?

Solution. A linear equation in two variables has infinitely many solutions.

Question 6. How many linear equations in two variables exist for which (2, 4) is a solution?

Solution. There are infinitely many linear equations in two variables exist for which (2, 4) is a solution.

Example: x + y = 6; x – y = -2; y = 2x.

Question 7. Write any two linear equations in two variables whose solution is (6,2).

Solution. Given solution: (6,2)

Sum of x-coordinate and y coordinate = 8 ⇒ x + y = 8

Difference of x-coordinate and y coordinate ⇒ 4x – y = 4

Question 8. Check whether (2, -5) is a solution of equation 2x + 5y = 2 or not.

Solution. Given equation : 2x + 5y = 2

Substitute x = 2 and y = -5

LHS = 2x + 5y = 2(2) + 5(-5)

= 4 – 25 = -21 ≠ RHS

∴ (2,-5) is not a solution of the given equation.

Question 9. Check whether (5, 0) is a solution of equation x + 3y = 5 or not.

Solution. Given equation: x + 3y = 5

Substitute x = 5 and y = 0

LHS = x + 3y = 5 + 3(0)

= 5 + 0 = 5 = RHS

∴ (5, 0) is a solution of the given

Question 10. Find 2 different solutions of x + y = 9

Solution. Given equation: x + y = 9

⇒ y = 9 – x

(a) Let x = 0 ⇒ y = 9 – 0 = 9.

Here solution = (0,9).

(b) Let x = 2 ⇒ y = 9 – 2 = 7.

Here solution = (2,7).

Question 11. Find two different solutions of 4x + y = 3.

Solution. Given equation: 4x + y = 3

⇒ y = 3 – 4x

(a) Let x = 0 ⇒ y = 3 – 4(0) = 3-0 = 3.

Here solution = (0,3).

(b) Let x = 2 ⇒ y = 3 – 4(2) = 3 – 8 = -5

Here solution = (2,-5).

Question 12. Which type of graph of a linear equation ax + by + c = 0 (Here a, b and c ∈ R & a ≠ 0, b ≠ 0) represents?

Solution. The graph of every linear equation in two variables (ax + by + c = 0) is a straight line.

Question 13. If (2, 0) is a solution of the linear equation 5x – 4y = k, then find the value of k.

Solution. Given equation: 5x – 4y = k

(2, 0) is a solution.

On substituting, x = 2 and y = 0.

⇒ 5(2) – 4(0) = k

⇒ 10 – 0 = k

⇒ 10 = k

∴ k = 10

Question 14. Write the equations of coordinate axes x and y.

Solution. The equation of the X-axis: x = 0

The equation of the Y-axis: y = 0

Question 15. Linear equation x – 2 = 0 is parallel to which axis?

Solution. Given equation: x – 2 = 0 ⇒ x = 2.

It is in the form of x = k.

∴ It is parallel to Y-axis.

Question 16. Write the equation of the line parallel to y-axis and passing through the point (-7,3).

Solution. The equation of the straight line parallel to Y-axis: x = k

Required equation: x = -7

⇒ x + 7 = 0

Question 17. Write the equation of the line parallel to X-axis and passing through the point (-2,-4)

Solution. The equation of the straight line parallel to X-axis: y = k

∴ Required equation: y = -4

⇒ y + 4 = 0

Question 18. Write the equation of three lines that are parallel to X-axis

Solution. The equation of three lines that are parallel to X-axis:

(1) y = 2

(2) y + 9 = 0

(3) 2y = 3

Question 19. Write the equation of three lines that are parallel to Y-axis

Solution. The equation of three lines that are parallel to X-axis:

(1) x = -2

(2)x – 9 = 0

(3) 5x = -3

Question 20. Find the distance between the graph of x – 5 = 0 and the Y-axis.

Solution. Given equation: x – 5 = 0 ⇒ x = 5

The graph of x = k is a straight line parallel to Y-axis. It lies at k units from Y-axis.

∴ The distance between the graph of x – 5 = 0 and the Y-axis is 5 units.

Question 21. Find the distance between the graph of 2y – 5 = 0 and the X-axis.

Solution. Given equation: 2y – 5 = 0 ⇒ x = 2.5

The graph of y k is a straight line parallel to X-axis. It lies at k units from X-axis.

∴ The distance between the graph of 2y – 5 = 0 and the Y-axis is 2.5 units.

Haryana Board Class 9 Maths Solutions For Chapter 4 Linear Equations In Two Variables Short Answer Type Questions

Question 22. Bhargavi got 10 more marks than double of the marks of Sindhu. Express the information in the form of an equation.

Solution. Let marks got by Bhargavi = x

Let marks got by Sindhu = y

Given that Bhargavi got 10 more marks than double of the marks Sindhu

∴ x = 10 + 2y

⇒ x – 2y – 10 = 0

Question 23. A number is 27 more than the number obtained by reversing its digits. If its units and tens digits are x and y respectively, write the linear equation representing the above statement.

Solution. Let the digit in units place = x

Let the digit in tens place = y

The number = 10y + x

If we reverse the digits, then the new number = 10x + y

From problem,

(Two-digit number) – (number formed by reversing the digits) = 27.

i.e., 10y + x – (10x + y) = 27

⇒ 10y + x – 10x – y – 27 = 0

⇒ 9y – 9x – 27 = 0

⇒ y – x – 3 = 0

⇒ x – y + 3 = 0

Haryana Board 9th Class Maths Chapter 4 Exercise Solutions

Question 24. The cost of a ball pen is 5 less than half the cost of a fountain pen. Write a linear equation in two variables to rep- resent this statement.

Solution. Let the cost of fountain pen = ₹ x

Let the cost of ball pen = ₹ y

Given that the cost of a ball pen is 5 less than half the cost of a fountain pen.

⇒ y = \(\frac{x}{2}\) – 5

⇒ 2y = x – 10

⇒ x – 2y = 10

Question 25. If x = 2k + 1 and y = k is a solution of the equation 5x + 3y – 7 = 0, find the value of k.

Solution. Given equation: 5x + 3y – 7 = 0

x = 2k + 1 and y = k is a solution of the equation.

On substituting, x = 2k + 1 and y = k.

⇒ 5(2k+1) – 3(k) – 7 = 0

⇒ 10k + 5 – 3k – 7 = 0

⇒ 7k – 2 = 0

⇒ 7k = 2

⇒ k = \(\frac{2}{7}\)

Question 26. Find 4 different solutions of 5x + y = 3

Solution. Given equation: 5x + y = 3

⇒ y = 3 – 5x

(a) Let x = 0 ⇒ y = 3 – 5(0) = 3 – 0 = 3.

Here solution = (0,3).

(b) Let x = 1 ⇒ y = 3 – 5(1) = 3 – 5 = -2.

Here solution = (1,-2).

(c) Let x = -1 ⇒ y = 3 – 5(-1) = 3 + 5 = 8.

Here solution = (-1, 8).

(d) Let x = 2 ⇒ y = 3 – 5(2) = 3 – 10 = -7

Here solution (2, -7).

(e) ∴ The solutions are

(0,3), (1, -2), (-1, 8), (2, -7).

Question 27. At which point the graph of the linear equation 2x – 3y = 6 cuts the Y-axis.

Solution. Given equation: 2x – 3y = 6

The x-coordinate of any point on the y-axis is zero.

Let the point of the line cuts the y-axis is (0, a).

On substituting, x = 0 and y = a.

⇒ 2(0) – 3(a) = 6

⇒ 0 – 3a = 6

⇒ -3a = 6

⇒ a = \(-\frac{6}{3}\)

⇒ a = -2

∴ The required point = (0, -2).

Haryana Board Class 9 Maths Solutions For Chapter 4 Linear Equations In Two Variables Long Answer Type Questions

Question 28. If (0, a) and (b, 0) are the solutions of the following linear equation 2x – 3y = 6. Find ‘a’ and ‘b’.

Solution. Given equation: 2x – 3y = 6.

(0, a) is one of the solutions of equation.

On substituting, x=0 and y = a.

⇒ 2(0) – 3(a) = 6

⇒ 0 – 3n = 6

⇒ -3a = 6

⇒ a = \(-\frac{6}{3}\)

⇒ a = -2

(b, 0) is another solution of equation.

On substituting, x = b and y = 0.

⇒ 2(b) – 3(0) = 6

⇒ 2b – 0 = 6

⇒ 2b = 6

⇒ b = \(\frac{6}{2}\)

⇒ b = 3

∴ a = -2 and b = 3

Haryana Board Class 9 Maths Solutions For Chapter 4 Linear Equations In Two Variables Objective Type Questions

Multiple Choice Questions:

Question 1. Which of the following is an equation?

  1. 2x + 3 ≠ 4
  2. 3x + y – 3
  3. 7x – 7
  4. 3m2+ m = 1

Answer. 4. 3m2 + m = 1

Question 2. General form of linear equation in two variables is where a, b, c ∈ R and a and b ≠ 0.

  1. ax + b = c
  2. ax + by = cz
  3. ax + by + c = 0
  4. ay + bx = c

Answer. 3. ax + by + c = 0

Question 3. Sankar and Sanvi collected 100 together. Suitable equation for this information is

  1. x + y + 100 = 0
  2. x + y = 100
  3. x – y = 100
  4. x – y = -100

Answer. 2. x + y = 100

MCQ Questions on Linear Equations in Two Variables Class 9 Haryana Board

Question 4. On comparing 4x – y = 0 with ax + by + c = 0, we get c =

  1. 4
  2. -1
  3. 0
  4. -4

Answer. 3. 0

Question 5. On comparing -y = 0 with ax + by + c = 0, we get a + b + c =.

  1. -1
  2. 0
  3. -2
  4. 3

Answer. 1. -1

Question 6. The linear equation 2x+5 has _______ solution(s).

  1. Unique
  2. two
  3. No
  4. Infinitely many

Answer. 1. Unique

Question 7. The linear equation 2x + y = 5 has _______ solution(s).

  1. Unique
  2. two
  3. No
  4. Infinitely many

Answer. 4. Infinitely many

Question 8. The linear equation x + y = 5 has _______ natural solution(s).

  1. Unique
  2. two
  3. No
  4. Infinitely many

Answer. 1. Unique

Question 9. The solution of equation x – 2y = 4 are

(1)(0-2)

(2) (8.0)

(3)(6, 1)

(4) (-2,3)

  1. (1) only
  2. (1) & (3) only
  3. (2) & (4) only
  4. (4) only

Answer. 2. (1) & (3) only

Question 10. Number of linear equations in x and y can be satisfied by x=1 and y = 2 is

  1. one
  2. two
  3. zero
  4. Infinitely many

Answer. 4. Infinitely many

Question 11. If (2, 0) is a solution of the linear equation 2x + 3y = k, then the value of k is:

  1. 4
  2. -4
  3. 3
  4. 5

Answer. 1. 4

Question 12. x = 9, y = 4 is a solution of the linear equation

  1. x + y + 13 = 0
  2. x – y = 5
  3. x – y + 5 = 0
  4. x + y = -5

Answer. 2. x – y = 5

Question 13. (3,1) is the solution of the equation

  1. 2x – y = 7
  2. x – 2y = -7
  3. 2x + y = 7
  4. x – 2y = 7

Answer. 3. 2x + y = 7

Question 14. (2,-1) is the solution of the equation

(1) 3x + y = 5

(2) 2x – y = 5

(3) x + y + 1 = 0

(4) x + 2y = 1

  1. (1) & (2) only
  2. (1) & (3) only
  3. (3) & (4) only
  4. (4) only

Answer. 1. (1) & (2) only

Question 15. If x = -2 and y = 3 is the solution of equation x + 2y = k, then k=

  1. 4
  2. -4
  3. 3
  4. 5

Answer. 1. 4

Question 16. The geometrical representation of linear equation in two variables is

  1. Straight line
  2. Circle
  3. Parabola
  4. Triangle

Answer. 1. Straight line

Class 9 Maths Chapter 4 Theorems and Formulas Haryana Board

Question 17. For the equation 5x – 7y = 35, if y = 5, then the value of ‘x’ is

  1. -14
  2. 0
  3. 14
  4. 10

Answer. 3. 14

Question 18. The straight line passing through the points (0,0), (-1, 1) and (1, -1) has the equation

  1. x – y = 0
  2. x + y = 0
  3. x + y = -1
  4. x – y = 1

Answer. 2. x + y = 0

Question 19. Any point of the form (a, a) always lies on the graph of the equation

  1. x – y = 0
  2. x + y = 0
  3. x + y = -1
  4. x – y = 1

Answer. 1. x – y = 0

Question 20. Which of the following is not a solution of 2x – y + 3 = 0?

  1. (3,9)
  2. (0,3)
  3. (-1,1)
  4. (-1,-2)

Answer. 4. (-1,-2)

Question 21. The graph of the equation 2x + 3y = 6 cuts the X-axis at the point

  1. (3,0)
  2. (0,-3)
  3. (-2,0)
  4. (0,2)

Answer. 1. (3,0)

Question 22. The graph of linear equation x + 2y = 2, cuts the Y-axis at the point

  1. (1,0)
  2. (0,-3)
  3. (-2,0)
  4. (0,1)

Answer. 4. (0,1)

Question 23. Which of the following is true?

  1. The line y = 2 parallel to Y-axis.
  2. The line y – 3 = 0 parallel to X-axis.
  3. The line y = 2 passes through (2,0)
  4. The line y – 3 = 0 passes through (-3,0)

Answer. 2. The line y – 3 = 0 parallel to X-axis.

Question 24. Which of the following is not a solution of 3x – y = 6?

  1. (0, -6)
  2. (2, 0)
  3. (-1,9)
  4. (1,-3)

Answer. 3. (-1,9)

Question 25. The value of k if x = 2, y = 1 is a solution of equation 2x – k = -3y is

  1. 7
  2. 6
  3. -6
  4. -7

Answer. 1. 7

Question 26. Which of the following lines pass through origin?

(1) √3x + 3y = 0

(2) 4y = 3

(3) 5x = 2

(4) 4y = 3y

  1. (1) & (2) only
  2. (2) & (3) only
  3. (3) & (4) only
  4. (1) & (4) only

Answer. 4. (1) & (4) only

Question 27. The equation 2x+5y= 7 has a unique solution, if x, y are _______ numbers.

  1. natural
  2. integers
  3. rational
  4. real

Answer. 1. natural

Linear Equations in Two Variables Word Problems Class 9 Haryana Board

Question 28. A linear equation in two variables is of the form ax + by + c = 0 where

(1) a ≠ 0

(2) b ≠ 0

(3) c ≠ 0

(4) c = 0

  1. (1) & (2) only
  2. (2) & (3) only
  3. (3) & (4) only
  4. (4) only

Answer. 1. (1) & (2) only

Question 29. A linear equation in two variables is of the form ax + by + c = 0 passes through origin if

(1) a = 0

(2) b ≠ 0

(3) c ≠ 0

(4) c = 0

  1. (1) & (2) only
  2. (2) & (3) only
  3. (3) & (4) only
  4. (4) only

Answer. 4. (4) only

Question 30. The number of linear equations in 2 variables passes through 2 distinct points is

  1. one
  2. two
  3. zero
  4. Infinitely many

Answer. 1. one

Question 31. The positive solutions of the equation ax + by + c = 0 always lie in

  1. 1st Quadrant
  2. 2nd Quadrant
  3. 3rd Quadrant
  4. 4th Quadrant

Answer. 1. 1st Quadrant

Question 32. If we multiply or divide both sides of a linear equation with a non-zero number, then the solution of the linear equation

  1. Changes
  2. Remains the same
  3. Changes in case of multiplication only
  4. Changes in case of division only

Answer. 2. Remains the same

Question 33. Which of the following equation has graph parallel to Y-axis?

  1. x = 3
  2. y = 5
  3. 2x + 3y = 0
  4. 2x = 3y

Answer. 1. x = 3

Question 34. Which of the following equation has graph parallel to X-axis ?

  1. 4x = 3
  2. y + 6 = 0
  3. 2x – y = 0
  4. 2x + 4y = 8

Answer. 2. y + 6 = 0

Question 35. The equation of the line parallel to X-axis and passing through the point

  1. y – 3 = 0
  2. y + 3 = 0
  3. y + 2 = 0
  4. y – 2 = 0

Answer. 2. y + 3 = 0

36-40: Direction: There are two statements are given in each question. Select the options as the following.

1) Both statements are true

2) Statement A is true, statement B is false.

3) Both statements are false

4) Statement A is false, statement B is true.

Question 36. Statement A: The linear equation 2x – 5y = 7 has infinitely many solutions.

Statement B: Only one linear equation in x and y can be satisfied by x = 1 and y = 2.

Answer. 2. Statement A is true, statement B is false.

Question 37. Statement A: (1, -4) is one of the solution of equation x – 2y = 9.

Statement B: The equation x + y = 5 has only one pair of natural number solutions.

Answer. 1. Both statements are true

Question 38. Statement A: Equation of Y-axis is : y = 0.

Statement B: y = 2 line parallel to y axis.

Answer. 3. Both statements are false

Question 39. Statement A: The line 3x + 5y = 0 passes through origin.

Statement B: The y = x passes through (4,4).

Answer. 1. Both statements are true

Question 40. Statement A: The line parallel to the Y-axis at a distance 4 units to the left of Y-axis is x = -4.

Statement B: The equation of X-axis is of the form y = 0.

Answer. 4. Statement A is false, statement B is true.

41-50: Assertion and Reasoning questions

Direction: In each of the following questions, a statement of Assertion is given followed by a corresponding statement of Reason just below it. Of the statements, mark the correct answer as

1) Both assertion and reason are true and reason is the correct explanation of assertion.

2) Both assertion and reason are true but reason is not the correct explanation of assertion.

3) Assertion is true but reason is false.

4) Assertion is false but reason is true.

Question 41. Assertion: There are infinite number of lines which passes through (-3,5)

Reason: A linear equation in two variables has infinitely many solutions.

Answer. 1. Both assertion and reason are true and reason is the correct explanation of assertion.

Question 42. Assertion: The graph of the equation 3x + y = 0 is a line passing through the origin.

Reason: An equation of the form ax + by + c = 0, where a, b, c R and a0, b0 is a linear equation in x and y.

Answer. 1. Both assertion and reason are ture and reason is the correct explanation of assertion.

Question 43. Assertion: The point (0, 3) lies on the graph of the linear equation 3x + 4y = 12.

Reason: (0, 3) satisfies the equation 3x + 4y = 12.

Answer. 2. Both assertion and reason are true but reason is not the correct explanation of assertion.

Question 44. Assertion The graph of the linear equation x – 2y = 1 passes through the point (-1, -1)

Reason: The linear equation x – 2y = 1 has unique solution.

Answer. 3. Assertion is true but reason is false.

Question 45. Assertion: The point (2, 2) lies on the line y = x

Reason: Any point on the line y = x is of the form (a, a)

Answer. 1. Both assertion and reason are true and reason is the correct explanation of assertion.

Question 46. Assertion: A linear equation 2x + 3y = 5 has a unique solution.

Reason: There are infinitely many solutions for a linear equation in two variables.

Answer. 4. Assertion is false but reason is true.

Question 47. Assertion: The graph of a line 3x – 4y + 12 = 0 intersects Y-axis at (0,3).

Reason: The line ax + by + c = 0 intesects X-axis at (\(-\frac{c}{a}\),0).

Answer. 2. Both assertion and reason are true but reason is not the correct explanation of assertion.

Question 48. Assertion: All the points (0, 0), (0,5), (0,3) and (0, 6) lie on the Y-axis.

Reason: Equation of the Y-axis is x = 0.

Answer. 1. Both assertion and reason are true and reason is the correct explanation of assertion.

Question 49. Assertion: The line y = 5x passes through origin..

Reason: The linear equation y = mx + c (c+0) passes through origin.

Answer. 3. Assertion is true but reason is false.

Question 50. Assertion: The geometric representation of x = -2 meets the X-axis at (0, -2).

Reason: The line y = k is parallel to X-axis and passes through the point (0, k).

Answer. 4. Assertion is false but reason is true.

Haryana Board Class 9 Maths Solutions For Chapter 4 Linear Equations In Two Variables Match the following:

Question 51. Match the following linear equations and points on those lines.

Class 9 Maths Chapter 4 Linear Equations In Two Variables Match the following Question 51

Answer. 1 – C, 2 – E, 3 – B, 4 – A, 5 – G.

Question 52. Match the following with suitable equations

Class 9 Maths Chapter 4 Linear Equations In Two Variables Match the following Question 52

Answer. 1 – G, 2 – C,3 – B, 4 – D, 5 – A.

Question 53. Match the following general form of points with their equations. (Here a ≠ 0)

Class 9 Maths Chapter 4 Linear Equations In Two Variables Match the following Question 53

Answer. 1 – C, 2 – D, 3 – A, 4 – E, 5 – F.

Haryana Board Class 9 Maths Solutions For Chapter 4 Linear Equations In Two VariablesTrue or False Questions

Question 54. Any equation which can be put in the form ax + by + c = 0, where a, b and c are real numbers, and a and b are not both zero is called a linear equation in two variables

Answer. True

Question 55. 5x – 3 = 2x is a linear equation in one variable

Answer. True.

Question 56. 3x – y2 = 5 is a linear equation in two variables

Answer. False

Question 57. ax2 + by + c = 0 where a, b and c are real numbers, is a linear equation in two variables.

Answer. False

Question 58. There are infinitely many solutions for a given linear equation in two variables

Answer. True

Question 59. 3x + 5 = 0 has a unique solution

Answer. True

Question 60. The linear equation 2x + 5y = 7 has unique natural solutions.

Answer. True

Question 61. The linear equation 2x – 3y = 4 has unique solution.

Answer. False

Question 62. The point of the form (a,a) always lies on X-axis

Answer. False

Question 63. The graph of every linear equation in two variables need not be a line

Answer. False

Question 64. (0,2) is a solution for x – 2y = 4

Answer. False

Question 65. The point (0,-3) lies on the graph of the linear equation 3x + 4y = 12

Answer. True

Question 66. The graph of the linear equation x + 2y = 7 passes through the point (0,7)

Answer. False

Question 67. The graph of linear equation 2x – 3y = 0 is a line parallel to X-axis

Answer. False

Question 68. The line x – y = 4 intersects X-axis at (4,0)

Answer. True

Question 69. The line ax + by + c = 0 intersects Y-axis at (0, \(-\frac{c}{a}\))

Answer. False

Question 70. The line ax + by + c = 0 passes through origin if c = 0.

Answer. True

Question 71. (3, -3) lies on x + y = 3.

Answer. False

Question 72. The line passes through (4,7) and parallel to X-axis is y = 7.

Answer. True

Question 73. The line passes through (-3,7) and parallel to Y-axis is x – 3 = 0

Answer. False

Question 74. The line y = -3 intersects Y-axis at (0,-3)

Answer. True

Question 75. The line x + 2 = 0 intersects Y-axis at (0,-2)

Answer. False

Question 76. The line y = mx passes through origin.

Answer. True

Question 77. x = 0 is the equation of the X-axis.

Answer. False

Question 78. The linear equation 3x + 2 = 0 represents a line parallel to Y-axis.

Answer. True

Question 79. The graph of y = 6 is a line parallel to X-axis at a distance 6 units from the X-axis

Answer. True

Haryana Board Class 9 Maths Solutions For Chapter 4 Linear Equations In Two Variables Fill in the Blanks:

Question 80. A statement in which one expression equals to another expression is _______

Answer. equation

Question 81. An equation with only one variable of degree one is called as _______ equation in one variable.

Answer. linear

Question 82. General form of linear equation in one variable is _______

Answer. ax + b = 0

Question 83. Solution of linear equation ax + b = 0 is _______

Answer. –\(\frac{b}{a}\)

Question 84. An equation with two variables both of degree one is called as _______ equation in _______ variables.

Answer. linear, 2

Question 85. General form of linear equation in two variables is _______

Answer. ax + by + c = 0

Question 86. The number of solutions of linear equations in two variables is _______

Answer. infinite

Question 87. The graph of every linear equation in two variables (ax + bt + c = 0) is a _______

Answer. line

Question 88. If x = 1 and y = 1 is one of the solutions of x – y = k then k = _______

Answer. 0

Question 89. All the points (2,0), (-3,0), and (5,0) lie on the _______ axis.

Answer. x

Question 90. All the points (0,3), (0,0), (0,-4) and (0,7) lie on the _______ axis.

Answer. y

Question 91. Abscissa of all points on the Y-axis is _______

Answer. 0

Question 92. The negative solutions of the equation ax + by + c = 0 always lie in the _______ quadrant.

Answer. 3rd

Question 93. The positive solutions of the equation ax + by + c = 0 always lie in the _______ quadrant.

Answer. 1st

Question 94. If the point (3,4) lies on the graph of 3y = ax + 7, then the value of a = _______

Answer. \(\frac{5}{3}\)

Question 95. The line ax + by + c = 0 intersects X-axis at _______

Answer. (-\(\frac{c}{a}\), 0)

Question 96. The graph of the linear equation 2x + 3y = 6 is a line which meets the X-axis at _______

Answer. (3,0)

Question 97. The graph of the linear equation 3x – 4y – 12 = 0 is a line which meets the Y-axis at _______

Answer. (0,-3)

Question 98. The value of y if x = 2 in the linear equation 3x – 4y – 12 = 0 is _______

Answer. 3

Question 99. The value of x for which y = -4 is a solution of the linear equation 5x – 8y = 40 is _______

Answer. \(\frac{8}{5}\)

Question 100. An ordered pair that satisfy an equation in two variables is called its _______

Answer. solution

Question 101. If x = 1 and y = 0 is the solution of equation 2x + y = 3a, then the value of a _______

Answer. \(\frac{2}{3}\)

Question 102. If (3,-2) is a solution of the equation 3x – py – 7 = 0, then the value of p is _______

Answer. -1.

Question 103. If 7x – 3y = k passes through origin, then k = _______

Answer. 0

Question 104. The point of the form (a,a) always lies on _______

Answer. x = y

Question 105. The equation x = 7, in two variables, can be written as _______

Answer. 1.x + 0.y – 7 = 0

Question 106. If (a,1) lies on the graph of 3x – 2y + 4 = 0, then a = _______

Answer. –\(\frac{2}{3}\)

Question 107. The area of a triangle formed by coordinate axes and line x + y = 4 is _______

Answer. 8 sq. units

Question 108. The area of a rectangle formed by coordinate axes, line x = 2 and y = 6 is _______

Answer. 12 sq. units

Question 109. The line passes through (0,p) and parallel to X-axis is _______

Answer. y = p

Question 110. The linear equation such that each point on its graph has an ordinate 3 times its abscissa _______

Answer. y = 3x

Question 111. The line y = mx passes through _______

Answer. origin

Question 112. The equation of the X-axis is _______

Answer. y = 0

Question 113. The line parallel to the Y-axis at a distance 4 units to the left of Y-axis is given by the equation _______

Answer. x + 4 = 0

Question 114. The graph of y = 6 is a line parallel to _______ axis.

Answer. x

Question 115. The line x + 3 = 0 passes through _______ and _______ quadrants.

Answer. 2nd, 3rd

Question 116. The line passes through (-6,-5) and parallel to Y-axis is _______

Answer. x + 6 = 0

Question 117. The line passes through (-2,-3) and parallel to X-axis is _______

Answer. y + 3 = 0

Question 118. The line x + 2 = 0 intersects X-axis at _______

Answer. (-2,0)

Question 119. The line y = 2 intersects Y-axis at _______

Answer. (0,2)

Haryana Board Class 7 Maths Solutions For Chapter 9 Perimeter And Area

Haryana Board Class 7 Maths Solutions For Chapter 9 Perimeter And Area

Key Concepts

  1. Perimeter: Perimeter is the distance around a closed figure.
  2. Area: Area is the region occupied by a closed figure.
  3. Remember:
    1. Perimeter of a regular polygon = number of sides x length of one side
    2. Perimeter of square = 4 x side
    3. Perimeter of a rectangle = 2x(l+b)
    4. Area of a rectangle = l x b
    5. Area of a square = side x side
    6. Increase of perimeter does not necessarily imply that area also increases.
  • A quadrilateral is a closed figure with four sides, four angles and four vertices.
  • Quadrilateral ABCD is said to be a convex quadrilateral if all line segments joining points in the interior of the quadrilateral also tie in interior of the quadrilateral.
  • Quadrilateral PQRS is said to be a concave quadrilateral if all line segments joining points in the interior.of the quadrilateral do not
    necessarily lie in the interior of the quadrilateral.
  • Trapezium is a quadrilateral with one pair of parallel sides.
  • The diagonals ofa parallelogram bisect each other.
  • The diagonals of a rhombus are perpendicular bisectors of one another.
  • Area of parallelogram = base x height
    Area of rectangle = length x breadth (length = base; breadth = height)
  • If all the sides of a parallelogram are equal,it is called a Rhombus’.
  • The area of a rhombus is equal to half of the product of its diagonals i.e., A = \( \frac{1}{2} d1:d2 \)
  • The approximate value of the ratio of the circumference to the diameter of a circle is \( \frac{22}{7} \) or 3.14. It is a constant and is denoted by π (Pi).
  • \( \frac{c}{d} \) = where ‘c’ is the circumference of the circle and ‘d’ is its diameter

Since, \( \frac{c}{d} \) =
= n, where ‘c’ is the circumference of the circle and W is its diameter.
= n, c= nd

Since, diameter of a circle is twice the radius i.e. d = 2r, c=π x 2r or c=2πr.

The area of a triangle is equal to half of the product of its base (b) and height (h) i.e. A =\( \frac{1}{2} \) bh

Solutions To Try These

Find the area of the following parallelograms:

1)

Find the area of the following parallelograms 1

Solution:

Base = 8 cm

Height = 3.5 cm

Area of the parallelogram= base x height = 8 X 3.5 = 28 cm²

2)

Find the area of the following parallelograms 2

Solution:

Base = 8 cm

Height = 2.5 cm

Area of the parallelogram=base X height = 8 X 2.5 = 20 cm²

Haryana Board Class 7 Maths Perimeter and Area Solutions

3) In a parallelogram ABCD, AB = 7.2 cm, and the perpendicular from C on AB is 4.5 cm.

Solution:

Base = 7.2 cm

Height = 4.5 cm

Area of the parallelogram = base X height = 7,2 X 4.5 = 32. 40 cm²

Solutions To Try These

1. Try the above activity with different types of triangles.

Solution:

Try yourself

2. Take different parallelograms. Divide each of the parallelograms into two triangles by cutting along any of its diagonals. Are the triangles congruent?

Solution:

Try yourself.

Hint: All the congruent triangles are equal in area but the triangles equal in area need not be congruent.

Haryana Board Class 7 Maths Solutions For Chapter 9Exercise-9.1 :

1. Find the area of each of the following parallelogram:

1)

Find the area of each of the following parallelograms

Solution:

Base = 7 cm 7cm

Height = 4 cm

Area of parallelogram= base X height

= 7 x 4 = 28 cm²

2)

Find the area of each of the following parallelograms 2

Solution:

Base = 5 cm

Height = 3 cm

Area of parallelogram = b x h

= 5×3 = 15 cm²

3)

Find the area of each of the following parallelograms 3

Solution:

Base = 2.5 cm

Height = 3.5 cm

Area of parallelogram =b x h

= 2.5 x 3.5 = 8.75 cm²

Area of parallelogram =b x h

= 2.5 x 3.5 = 8.75 cm²

Class 7 Maths Chapter 9 Perimeter and Area Haryana Board

4)

Find the area of each of the following parallelograms 4

Solution:

Base = 5 cm

Height = 4.8 cm

Area of parallelogram = b x h

= 5 X 4.8 = 24 cm²

HBSE Class 7 Rational Numbers Solutions Ex 9.1

5)

Find the area of each of the following parallelograms 5

Solution:

Base = 2 cm

Height = 4.4 cm

Area of parallelogram = base x height

= 2×4.4 = 8.8 cm²

2. Find the area of each of the following triangles:

Solution:

1)

Find the area of each of the following triangles

Base = 4 cm

Height = 3 cm

Area of triangle = \( \frac{1}{2} b x h \)x b x h

\( =\frac{1}{2} \times 4 \times 3=\frac{12}{2}=6 \mathrm{~cm}^2 \)

2)

Find the area of each of the following triangles 2

Base = 5 cm

Height= 3.2 cm

Area of triangle = \( \frac{1}{2} \times b \times h \)

\( =\frac{1}{2} \times 5 \times 3.2=\frac{16}{2}=8 \mathrm{~cm}^2 \)

3)

Find the area of each of the following triangles 3

Solution:

Base = 3 cm

Height =4 cm

Area of triangle =\( \frac{1}{2} \times b \times h \)

\( =\frac{1}{2} \times 3 \times 4=\frac{12}{2}=6 \mathrm{~cm}^2 \)

HBSE Class 7 Rational Numbers Solutions Ex 9.2

4)

Find the area of each of the following triangles 4

Solution:

Base = 3 cm

Height = 2 cm

Area of triangle = \( \frac{1}{2} \times b \times h \)

\( =\frac{1}{2} \times 3 \times 2=\frac{6}{2}=3 \mathrm{~cm}^2 \)

Haryana Board 7th Class Maths Perimeter and Area Questions and Answers

3. Find the missing values:

Find the missing values

Solution:

Area of parallelogram = bh

Find the missing values solutions

4. Find the missing values:

 

Find the missing values q4

Solution:

Find the missing values Solutions q4

Area of triangle = \( \frac{1}{2} \mathrm{bh} \)

5. PQRS is a parallelogram. QM is the height from Q to SR and QN is the height from Q to PS.If SR = 12 cm and QM = 7.6 cm.

PQRS is a parallelogram

Find:

1) The area of the parallelogram PQRS

2) QN,if PS = 8 cm

Solution:

1)

PQRS is a parallelogram.

Its base SR = 12 cm

Height QM = 7.6 cm

Area of the parallelogram PQRS

= base x height

=12×7.6 = 91.2 cm2

2) Base PS = 8 cm

Corresponding height QN =?

Area of parallelogram PQRS

= 91.2 cm²

= 8 X QN = 91.2

QN = \( \frac{91.2}{8}=11.4 \mathrm{~cm} \)

Key Questions in Rational Numbers Ex 9.1 for Class 7 HBSE

6. DL and BM are the heights on sides AB and AD respectively of parallelogram ABCD. If the area of the parallelogram is 1470cm², AB = 35cm, and AD = 49 cm, find the length of BM and DL.

DL and BM are the heights on sides AB and AD respectively of parallelogram ABCD

Solution:

ABCD is a parallelogram;

AD = 49 cm

Base AB = 35 cm

DL and BM are the heights.

Area of parallelogram ABCD =1470 cm²

AD X BM = 1470

49 X BM = 1470

\( \mathrm{BM}=\frac{1470}{49}=30 \mathrm{~cm} \)

The length of BM = 30 cm

Area of parallelogram ABCD = 1470 cm²

AB x DL = 1470

35 X DL = 1470

\( \mathrm{DL}=\frac{1470}{35}=42 \mathrm{~cm} \)

The length of DL 42 cm

Chapter 9 Perimeter and Area Class 7 Solutions in Hindi Haryana Board

7. AABC is right-angled at A. AD is perpendicular to BC. If AB = 5 cm, BC = 13 cm, and AC = 12 cm. Find the Area of ΔABC. Also, find the length of AD.

ABC is right angled at A. AD is perpendicular to BC

Solution:

AABC is right-angled at A.

When we take base AC = 12 cm and height AB = 5 cm

Area of the triangle ABC

\( \begin{aligned}
& =\frac{1}{2} \times \text { base } \times \text { height } \\
& =\frac{1}{2} \times 12 \times 5=30 \mathrm{~cm}^2
\end{aligned} \)

AD is perpendicular to BC.

When we take base BC = 13 cm

Area of the triangle ABC = 30 cm2

Its height AD =?

\( \begin{aligned}
& \frac{1}{2} \times \text { base } \times \text { height }=30 \\
& \frac{1}{2} \times 13 \times \mathrm{AD}=30
\end{aligned} \) \( \mathrm{AD}=\frac{30 \times 2}{13}=\frac{60}{13} \mathrm{~cm}=4 \frac{8}{13} \mathrm{~cm}\)

8. ΔABCis isosceles with AB=AC=7.5 cm and BC = 9 cm. The height AD from A to BC, is 6 cm. Find the area of AABC. What will be height from C to AB i.e CE?

Solution:

AABC is an isosceles triangle.

AB = AC = 7.5.cm

Base BC = 9 cm

Height AD = 6 cm

Area of triangle ABC

\( \begin{aligned}
& =\frac{1}{2} \times \text { base } \times \text { height } \\
& =\frac{1}{2} \times 9 \times 6=\frac{54}{2}=27 \mathrm{~cm}^2
\end{aligned} \)

Area of the AABC = 27 cm²

ABC is isosceles with AB=AC=7.5 cm and BC = 9 cm

When we take base AB = 7.5 cm

Its corresponding height CE =?

Area of triangle ΔABC

\( \begin{aligned}
& =\frac{1}{2} \times \text { base } \times \text { height } \\
& =\frac{1}{2} \times 7.5 \times \mathrm{CE}=27 \\
& C E=\frac{27 \times 2}{7.5}=\frac{54}{7.5}=7.2 \mathrm{~cm}
\end{aligned} \)

The height from C to AB is 7.2 cm

Practice Problems Rational Numbers HBSE Class 7

Solutions To Try These

From the figure,

From the figure

Which square has the larger perimeter?

Solution: The outer square has the larger perimeter.

Which is larger, perimeter of smaller square or the circumference of the circle?

Solution:

The circumference of the circleis larger.

Solutions To Try These

Draw circles of different radii on a graph paper. Find the area by counting the number of squares. Also find the area by using formula. Compare the two answers.

Solution:

Try yourself with the help your teacher.

Haryana Board Class 7 Maths Solutions For Chapter 9 Exercise-9.2

1. Find the circumference of the circles with the following radius:

1) 14 cm

Solution:

Radius of the circle (r) = 14 cm

Circumference of the circle = 2Πr

\( =2 \times \frac{22}{7} \times 14 \)

= 88 cm

2) 28 mm

Solution:

Radius of the circle (r) = 28 mm

Circumference of the circle = 2Πr

\( =2 \times \frac{22}{7} \times 28 \)

Haryana Board Class 7 Maths Exercise 9.1 Solutions

3) 21 cm

Solution:

Radius of the circle (r) = 21 cm

Circumference of the circle = 2Πr

=176 mm

\( =2 \times \frac{22}{7} \times 21 \)

= 132 cm

2. Find the area of the following circles, given that :

1) radius = 14 mm

Solution:

Radius of the circle (r) = 14 mm

Area of the circle = Πr²

\( =\frac{22}{7} \times 14 \times 14=616 \mathrm{~mm}^2\)

2)diameter = 49 m

Solution:

Diameter = 49 m

Radius = \( \frac{49}{2} \mathrm{~m} \) \( r=\frac{d}{2} \)

Area of the circle = Πr²

\( \begin{aligned}
& =\frac{22}{7} \times \frac{49}{2} \times \frac{49}{2} \\
& =11 \times 7 \times \frac{49}{2}
\end{aligned} \) \( =\frac{3773}{2} \)

= 1886.5 m2

3) radius = 5 cm

Solution:

Radius ( r) 5 cm

Area of the circle = Πr²

\( =\frac{22}{7} \times 5 \times 5=\frac{550}{7} \mathrm{~cm}^2=78.5 \mathrm{~cm}^2 \)

3. if the circumference of a circular sheet is 154 m, find its radius. Also, find the area of the sheet.

Solution:

Circumference of the circular sheet 154 m

2Πr =154

\( 2 \times \frac{22}{7} \times r=154 \) \( \begin{aligned}
r=154 \times \frac{1}{2} \times \frac{7}{22} & \\
& =\frac{7 \times 7}{2} \\
& =\frac{49}{2}
\end{aligned} \)

Area of the circular sheet = Πr²

\( \begin{aligned}
& =\frac{22}{7} \times \frac{49}{2} \times \frac{49}{2} \\
& =\frac{11 \times 7 \times 49}{2}=\frac{3773}{2}=1886.5 \mathrm{~m}^2
\end{aligned} \)

4. A gardener wants to fence a circular garden of diameter 21m. Find the length of the rope he needs to purchase, if he makes 2 rounds of fence. Also find the cost of the rope, if it costs Rs. 4 per meter. \( \text { (Take } \pi=\frac{22}{7} \text { ) } \)

Solution:

Diameter of the circular garden = 21 m

Circumference of the garden = Πd

\( =\frac{22}{7} \times 21=66 \mathrm{~m}\)

Length of the rope to make one round of fence = 66 m

Length of the rope to make 2 rounds of fence = 66 x 2 = 132 m

Cost of 1 m rope = Rs. 4

Cost of 132 m rope = Rs, 4 x 132= Rs. 528

HBSE 7th Class Rational Number Word Problems

5. From a circular sheet of radius 4 cm, a circle of radius 3 cm is removed. Find the area of the remaining sheet (Take π = 3.14)

Find the area of the remaining sheet

Solution:

Radius of a circular sheet = 4 cm

Area of the outer circle = Πr³

=> 3.14 x 4 x 4 = 3.14 X 16 = 50.24 cm³

Radius of the inner circle =3 cm

Area of the inner circle = Πr²

= 3.14 x 3 x 3 = 28.26 cm³

Area of the remaining sheet

= Area of outer circle – Area of inner circle

= 50.24 – 28.26 = 21.98 cm²

Important Questions for Class 7 Maths Chapter 9 Haryana Board

6. Saima wants to put a lace on the edge of a circular tabic cover of diameter 1.5 m. Find the length of the lace required and also find its cost if one meter of the lace costs Rs. 15. (Take π = 3.14)

Solution:

Diameter of the circular table cover = 1.5 m

Circumference of the table cover =πd

=3.14 x 1.5 = 4.71 m

Length of the lace required

Circumference of the circular table cover = 4.71 m

Cost of lm of lace = Rs. 15

Cost of 4.71 m of lace = Rs. 15 x 4.71 = Rs. 70.65

7. Find the perimeter of the adjoining figure, which is a semicircle including its diameter.

Find the perimeter of the adjoining figure

Solution:

Diameter of the semicircle ‘d’= 10 cm

Circumference of the semicircle

\( \begin{aligned}
& =\frac{1}{2} \text { (circumference of the circle) } \\
& =\frac{1}{2} \pi \mathrm{~d} \\
& =\frac{1}{2} \times 3.14 \times 10=3.14 \times 5=15.7 \mathrm{~cm}
\end{aligned} \)

Perimeter of semicircle

=Circumference of semicircle + diameter

= 15.7 + 10 = 25.7 cm

8. Find the cost of polishing a circular table – top of diameter 1.6 m, if the rate of polishing is Rs. 15/mJ. (Take Π = 3.14)

Solution:

Diameter of the circular table-top = 1.6m

Radius = \( \frac{1.6}{2} \) = 0.8m

Area of the table-top

= Πr²

= 3.14 x (0.8)²

= 3.14 x 0.8 x 0.8

= 3.14 x 0.64 = 20096 m²

Cost of l m² area = Rs. 15

Cost of 2.0096 m²area = Rs 15 x 2.0096

= Rs 30 .144

= Rs 30. 14 (approx)

9. Shazli took a wire of length 44 cm and bentit into the shape of a circle. Find the radius of that circle. Also find its area. If the same wire is bent into the shape of a square, what will be the length of each of its sides ? Which figure encloses more area, the circle or the square? \( \text { (Take } \pi=\frac{22}{7} \text { ) } \)

Solution:

Length of the wire = 44 cm

Length of the wire = 44 cm

Itis bent into the shape of a circle.

Circumference of the circle = 44 cm

27 πr = 44

Circumference of the circle = 44 cm

\( 2 \times \frac{22}{7} \times r=44 \) \( \mathrm{r}=44 \times \frac{1}{2} \times \frac{7}{22}=7 \mathrm{~cm} \)

Radius of that circle = 7 cm

Area of the circle = Πr²

\( =\frac{22}{7} \times\left(7^2\right)=\frac{22}{7} \times 7 \times 7=154 \mathrm{~cm}^2 \)

The wire is bent into the shape of a square.

shape of a square

Perimeter of the square = Length of the wire 4 X side = 44

Side =\( \frac{44}{4} \) =11 cm

Area of the square = side x side = 11 X 11 = 121 cm²

The circle encloses more area than the square.

Important Concepts Rational Numbers Class 7 HBSE Chapter 9

10. From a circular card sheet of radius 14 cm, two circles of radius 3.5 cm, and a rectangle of length 3 cm and breadth 1 cm are removed (as shown in the adjoining figure), find the area of the 22 remaining sheet.

Solution:

Radius of the circular card sheet=14 cm

Area of the circular card sheet

=πr²

Radius of the circular card sheet=14 cm

\( =\frac{22}{7}(14)^2=\frac{22}{7} \times 14 \times 14 \)

=22 x 2 x 14 =616 cm²

Area of two circles of radius = 3.5 cms.

\( \begin{aligned}
& =2\left[\pi \mathrm{r}^2\right]=2\left[\frac{22}{7} \times(3.5)^2\right] \\
& =2\left[\frac{2 \dot{2}}{7} \times 3.5 \times 3.5\right]
\end{aligned} \)

= 2 X 38.5 = 77 cm²

Length of the rectangle = 3 cm

Breadth =1 cm

Area of this rectangle = l x b

=3 x 1=3 cm²

Area of the sheet = Area of the circular card sheet- Area of the two circles Area of the rectangle.

= 616-77-3

= 616-80

= 536 cm²

Step-by-Step Solutions for Perimeter and Area Class 7 Haryana Board

11. A circle of radius 2 cm is cut out from a square piece of an aluminium sheet of side 6 cm. What is the area of the leftover aluminium sheet? (Take π= 3.14)

Solution: Side of the square piece of an aluminium sheet = 6 cm

Side of the square piece of an aluminium sheet

Area of this sheet = side x side = 6 x 6 = 36 cm²

Radius of the circle = 2 cm

Area of the circle

= Πr²

= 3.14 x(2)² = 3.14x2x2

= 3.14×4 = 12.56 cm²

Area of the aluminium sheet left over

= Area of the square- Area of the circle

= 36 -12.56 = 23.44 cm²

12. The circumference of a circleis 31.4 cm. Find the radius and area of the circle? (Take Π = 3.14)

Solution:

Circumference of a circle = 31.4 cm

2Πr= 31.4

2 X3.14 x r = 31.4

\( \mathrm{r}=\frac{31.4}{2 \times 3.14}=5 \mathrm{~cm} \)

Area of the circle = Πr²

= 3.14 X (5)²

= 3.14x5x5

= 3.14 X 25 = 78.5 cm²

13. A circular flowerbed is surrounded by a path 4 m wide. The diameter of the flower bed is 66 m. What is the area of this path? (Take π= 3.14)

Solution:

Diameter of the flower bed = 66 m

Radius of the flower bed = \( \frac{66}{2}=33 \mathrm{~m} \)

Area of the flower bed

Diameter of the flower bed = 66 m

= πr²

= 3.14 x (33)²

= 3.14 X 33 X 33

= 3.14 X 1089 = 3419.46 m²

Width of the path = 4 m

Radius of the flower bed with path = 33 + 4 = 37 m

Area of the flower bed with path

= Π²

= 3.14 x(37)²

= 3.14x37x37

= 3.14×1369 = 4298.66 m²

Area of the path = Area of flowerbed with path- Area of the flower bed = 4298.66- 3419. 46 = 879.20 m²

14. A circular flower garden has an area of 314 m2. A sprinkler at the centre of the garden can cover an area that has a radius of 12 m. Will the sprinkler water the entire garden? (Take Π – 3.14)

Solution:

Area of the circular flower garden = 314 m²

Πr²= 314

3.14 xr² = 314

\( \frac{314}{100} \times r^2=314 \) \( \begin{aligned}
& 3.14 \times r^2=314 \\
& \frac{314}{100} \times r^2=314 \\
& r^2=314 \times \frac{100}{314}
\end{aligned} \)

r² = 100

r = 100 = 10×10 = 10 m

Given that the sprinkler can cover the area that has a radius 12m.

12 m > 10 m

The sprinkler will water the entire garden.

HBSE Class 7 Maths Chapter 9 Guide Rational Numbers

5. Find the circumference of the inner and outer circles, shown in the adjoining figure. (Take π= 3.14)

Radius of the outer circle = 19m

Solution:

Radius of the outer circle = 19m

Circumference of the outer circle

= 2Πr

= 2×3.14×19

= 38×3.14 = 119.32 m

Radius of the inner circle =19m-10m = 9m

Circumference of the inner circle

= 2Πr

= 2×3.14×9

= 18X3.14 = 56.52 m

16. How many times a wheel of radius 28 cm must rotate to go 352 m?

Solution:

Radius of the wheel = 28 cm

Circumference of the wheel = 27Πr

\( =2 \times \frac{22}{7} \times 28 \)

Distance covered by the wheel in one rotation = Circumference of the wheel 1 m = 100 cm

352 m = 352 x 100 = 35200 cm

= 2 X 22 x 4 = 176 cm

Number of times the wheel must rotate to go 352 m.

\( =\frac{35200}{176}=200 \text { times } \)

17. The minute hand of a circular clock is 15 cm long. How far does the tip of the minute hand move in1 horn? (Take Π = 3.14)

Solution:

Length of the minute hand of a circular clock =15 cm

Radius of the circular clock ‘r’=15 cm

Circumference of the circle = 27Πr

=2X3.14X15 = 94.2 cm

In one hour i.e., in 60 minutes; the minute hand of the clock completes 1 rotation.

The tip of the minute hand moves 94.2 cm in 1 hour.

Haryana Board Class 7 Maths Solutions For Chapter 9 Very Short Answer Questions

1. Define ‘Area’.

Solution:

The amount of surface enclosed by a closed figure is called its ‘area’.

2. Define ‘Perimeter’.

Solution:

Perimeter is the distance around a closed figure.

3. The area of a rectangular sheet is 500 cm². If the length of the sheet is 25 cm. What is its width?

Solution:

Length of the rectangular sheet (Z)=25 cm

Area of the rectangular sheet = 500 cm²

l x b = 500 => 25 x b = 500

\( b=\frac{500}{25}=20 \mathrm{~cm} \)

4. The perimeter of a rectangle is 180 cm. If the breadth of the rectangle is 40 cm. Find its length.

Solution:

The breadth of a rectangle = 40 cm

Perimeter of the rectangle = 180 cm

2 ( l + b ) = 180

2 ( l + 40) = 180

\( l+40=\frac{180}{2} \text { or } l+40=90 \)

l = 90 – 40 = 50 cm

5. Find the area of the parallelogram whose base is 10 cm and the height is 4 cm.

Solution:

Base of the parallelogram = 10 cm

Height = 4 cm

Area of the parallelogram = Base x Height = 10 X 4 = 40 cm²

Rational Numbers on the Number Line Class 7 Haryana Board

6. Find the area of the triangle whose base is 6 cm and height 3 cm:

Solution:

Area of triangle

\( \begin{aligned}
& =\frac{1}{2} \mathrm{bh} \\
& =\frac{1}{2} \times 6 \times 3=9 \mathrm{~cm}^2
\end{aligned} \)

7. What is the circumference of a circular 22 disc of radius 14 cm?\( \)

\( \text { (Take } \pi=\frac{22}{7} \text { ) } \)

Solution:

Radius of circular disc = 14 cm

Circumference of disc = 2Πr

\( =2 \times \frac{22}{7} \times 14=88 \mathrm{~cm} \)

8. Diameter of a circular garden is 9.8 m. Find its area.

Solution:

Diameter d = 9.8 m

Radius (r) = \(\frac{9.8}{2} \) = 4.9 m

Area of the cirle = Πr²

\( =\frac{22}{7} \times 4.9 \times 4.9=75.46 \mathrm{~m}^2 \)

9. Find the base of a triangle whose area is 220 cm2 and height is 11 cm.

Solution:

Given area of triangle = 220 cm²

\( \begin{aligned}
& \Rightarrow \frac{1}{2} \times \text { base } \times \text { height }=220 \mathrm{~cm}^2 \\
& \quad \text { (height }=11 \mathrm{~cm} \text { ) } \\
& \Rightarrow \frac{1}{2} \times \text { base } \times 11=220 \\
& \text { base }=\frac{220 \times 2}{11}=40 \mathrm{~cm}
\end{aligned} \)

10. Find the circumference of a circle whose radius is (1) 35 cm (2) 4.2 cm (3) 15.4 cm

Solution:

Circumference of a circle = 2Πr

1) r = 35 cm; circumference

= 2 x \( \frac{22}{7} \) x 35 cm = 220 cm

2) r = 4.2 cm; circumference

= 2 x \( \frac{22}{7} \) x 4.2 = 26.4 cm

3) r = 15.4 cm; circumference

= 2 x \( \frac{22}{7} \) x 15.4 = 26.4 cm

11. If the circumference of a circle is 264 cm, find its radius. \( \text { Take } \pi=\frac{22}{7} \) .

Solution:

Circumference of a circle = 2rcr = 264cm

Given

\( 2 \times \frac{22}{7} \times r=264 \)

r = \( \frac{264 x 7}{2 x 22} \) = 42 cm

12. If the circumference of a circle is 33 cm, find its diameter.

Solution:

Given

Circumference of a circle = Πd = 33 cm

i.e, \( \frac{22}{7} \times d=33 \)

d = \( \frac{33×7}{22} [latex] = [latex] \frac{21}{2} \) = 10.5 cm

Haryana Board Class 7 Maths Solutions For Chapter 9 Short Answer Questions

13. Find the area of each of the following triangles.

1)

Find the area of each of the following triangles. 13 1

Solution:

1) Area of triangle = \( \frac{1}{2} \mathrm{bh} =

= [latex] \frac{1}{2} \) x 5 x 8 = 20 cm²

2)

Find the area of each of the following triangles. 13 2

Solution:

2) Area of triangle = \( \frac{1}{2} \mathrm{bh}

= [latex] \frac{1}{2} \) x 6 x 4 = 12 cm²

3)

Find the area of each of the following triangles. 13 3

Solution:

3) Area of triangle = \( \frac{1}{2} \) x 5.4 x 7.5 = 20.25 cm²

4)

Find the area of each of the following triangles. 13 4

Solution:

4) Area of triangle = \( \frac{1}{2} \) x 6 x 4 = 12 cm²

14. ΔABC is right-angled at A. AD is perpendicular to BC AB = 5 cm, BC = 13 cm, and AC =12 cm. Find the area of ΔABC. Also, find the length of AD.

Find the area of triangle ABC. Also, find the length of AD

Solution:

ABC is a right-angled triangle, either AB or AC can be considered as base or height

Take AC (Base 12 cm); height (AB) = 5 cm

Area of triangle = \( \frac{1}{2} \mathrm{bh}=\frac{1}{2} \times 12 \times 5=30 \mathrm{~cm}^2 \) …….(1)

Now take BC (base) = 13 cm and AD =h

1/2 bh = area; substituting, 13 cm for base

\( \begin{aligned}
&\text { we get } \frac{1}{2} \times 13 \times \mathrm{h}=30 \mathrm{~cm}^2\\
&\text { height }=\frac{30 \times 2}{13}=\frac{60}{13}=4.6 \mathrm{~cm} \text { (nearly) }
\end{aligned} \)

15. APQRis isosceles withPQ = PR = 7.5 cm and QR = 9 cm. The height PS from P to QR is 6 cm. Find the area of ΔPQR. What will be the height from R to PQ i.e. RT?

Solution:

By Question, base QR = 9 cm; height PS = 6 cm

Area of triangle \( =\frac{1}{2} \mathrm{bh} \)

= \(\frac{1}{2}\)x9x6 = 27cm² (1)………

What will be the height from R to PQ i.e. RT

Again, take PQ as base = PR = 7.5 cm (It is a triangle) & Height (TR) =h cm

\( \begin{aligned}
& \frac{1}{2} \mathrm{bh}=\text { Area ; from } \\
& \frac{1}{2} \times 7.5 \times \mathrm{h}=27
\end{aligned} \)

Height = \( \frac{27×2}{75} \) x 10 (decimal removed)

\( =\frac{36}{5}=7.2 \mathrm{~cm} \)

16. Find the area of the following rhombuses.

Find the area of the following rhombuses

Solution:

Area of rhombus = \( \frac{1}{2} \mathrm{~d}_1 \mathrm{~d}_2 \)

\( \begin{aligned}
& =\frac{1}{2} \times 10 \times 4 \mathrm{~cm}^2 \\
& =20 \mathrm{~cm}^2
\end{aligned} \)

Solution:

Area of rhombus = \( \frac{1}{2} \mathrm{~d}_1 \mathrm{~d}_2 \)

\( =\frac{1}{2} \times 8 \times 6 \mathrm{~cm}^2 \)

= 24 cm²

Haryana Board Class 7 Maths Solutions For Chapter 9 Long Answer Questions

17. Find the circumference of circle whose diameter is

(1) 17.5 cm (2) 5.6 cm (3) 4.9 cm

Note: take \( \pi=\frac{22}{7}\) in the above two questions.

Solution:

Circumference of circle = Πd

1) Circumference of circle = \( \frac{22}{7} \times 17.5=55.0 \mathrm{~cm} \)

2) Circumference of circle =\( \frac{22}{7} \times 5.6=17.6 \mathrm{~cm} \)

3) Circumference of circle =\( \frac{22}{7} \times 4.9=15.4 \mathrm{~cm} \)

18. 1) Taking Π = 3.14, find the circumference of a circle whose radius is

(1) 8 cm (2) 15 cm (3) 20 cm

2) Calculate the radius of a circle whose circumference is 44 cm.

Solution:

1) Circumference of circle = 2Πr

1) given r = 8 cm and Π= 3.14, circumference = 2 x 3.14 x 8 = 50.24 cm

2) given r = 15 cm and Π = 3.14, circumference = 2 x 3.14 x 15 = 94.20 cm

3) given r=20 cm and Π= 3.14, circumference = 2 x 3.14 x 20 = 125.60 cm

2) Given circumference = 44 cm. To find radius, 27Πr = 44

i.e \( 2 \times \frac{22}{7} \times r=44 \)

r = \( \frac{44×7}{2×22} \)

= 7

Finding Rational Numbers Between Two Numbers Class 7 HBSE

19. Arectangle ABCD with AB = 8 cm, BC = 16 cm and AE = 4 cm.Find the area of ΔBCE. Is the area of ΔBEC equal to the sum of the area of ΔBAE and ΔCDE. Why?

Arectangle ABCD with AB = 8 cm, BC = 16 cm and AE = 4 cm

Solution:

Area of a rectangle =l xb = 16 x 8 cm² = 128 cm²

Area of a triangle (BEC) = \( \frac{1}{2} \mathrm{bh} \) = \( \frac{1}{2} \) x 16 x 8 = 64 cm²

Area of a triangle (BAE) =\( \frac{1}{2} \mathrm{bh} \) = \( \frac{1}{2} \) x 4 x 8 = 16 cm²

Area of a triangle (CDE) =\( \frac{1}{2} \mathrm{bh} \) = \( \frac{1}{2} \) x 8 x 12 = 48 cm²

Observation:

1) Area of triangle BEC = \( \frac{1}{2} \) (area of rectangle)

2)Area of A BAE +A CDE = Area of A BEC because area of A BEC = Area of A BEC = \( \frac{1}{2} \) (Area of rectangle) the remaining portion of rectangle containing two triangles

BAE and CDE = BEC = 64 cm²

So, the area of ΔBEC is equal to the sum of the area of ΔBAE and ΔCDE.

Haryana Board Class 7 Maths Solutions For Chapter 9 Multiple Choice Question and Answers

1. 1 Hectare =

  1. 100 m²
  2. 1000 m²
  3. 1000 mm²
  4. 10 m²

Answer: 3

2. Side of a square is 50 cm find its area

  1. 2500 cm²
  2. 2000 cm²
  3. 2500 cm
  4. 200 cm

Answer: 1

3. The perpendicular dropped on that side from the opposite vertex is known as

  1. median
  2. height
  3. side
  4. length

Answer: 2

4. If the area of a parallelogram is 24 cm2 and the base is 4 cm then its height is

  1. 4 cm
  2. 6 cm
  3. 48 cm
  4. 96 cm

Answer: 2

5. Diameter of a circle is 10 cm. Its radius is

  1. 20 cm
  2. 10 cm
  3. 5 cm
  4. 40 cm

Answer: 3

6. If the area of a circle is 2464 m² find its diameter.

  1. 14 m
  2. 28 m
  3. 56m
  4. 45 m

Answer: 3

7. The circumference of a circle is 44 m. What is its area?

  1. 44 cm²
  2. 154 cm²
  3. 164 cm²
  4. 144 cm²

Answer: 2

8. Two sides of a right-angled triangle are 100 cm and 8.6 cm find its area.

  1. 340 cm²
  2. 530 cm²
  3. 430 cm²
  4. 240 cm²

Answer: 3

9. Find the altitude of a triangle whose base is 24 m and area 672 cm².

  1. 56 cm
  2. 4 cm
  3. 26 cm
  4. 36 cm

Answer: 1

10. Find the area of the triangle whose base is 14 cm and height is 650 cm

  1. 3550 cm²
  2. 4550 cm²
  3. 2550 cm²
  4. 5550 cm²

Answer: 2

11. If the perimeter of a semi-circle is 144 cm. What is its area?

  1. 1132 cm²
  2. 1432 cm²
  3. 1232 cm²
  4. 1332 cm²

Answer: 3

12. If each side of a square is 1 m which of the following is its area?

  1. 100 cm²
  2. 1000 cm²
  3. 10000 cm²
  4. 100000 cm²

Answer: 3

13. The sides of a triangle are 3 cm, 4 cm, and 5 cm respectively then the perimeter is

  1. 10 cm
  2. 12 cm
  3. 20 cm
  4. 15 cm

Answer: 2

14. If the side of an equilateral triangle is 6 cm then its perimeter is

  1. 15 cm
  2. 12 cm
  3. 18 cm
  4. 24 cm

Answer: 3

15. If the side of a right-angled isosceles triangle is 2 m then its area is

  1. 4 m²
  2. 6 m²
  3. 5 m²
  4. 2 m²

Answer: 4

16. The diagonals of a rhombus are 8 cm and 12 cm then its area is

  1. 64 sq.cm
  2. 40 sq.cm
  3. 48 sq.cm
  4. 70 sq.cm

Answer: 3

17. If the base of an isosceles right triangle is 30 cm then its area is

  1. 300 sq.cm
  2. 400 sq.cm
  3. 450 sq.cm
  4. 500 sq.cm

Answer: 3

18. If the diameter of the circle is 52 cm then its radius is

  1. 26 cm
  2. 27 cm
  3. 28 cm
  4. 29 cm

Answer: 1

19. If the radius of a circle is 12 m then the circumference of a circle is

  1. 20πm
  2. 24πm
  3. 48πm
  4. 42πm

Answer: 2

20. Choose the correct matching.

1) Circumference of circle              (  )    1/2 bh

2) Area of circle                              (  )      l x b

3) Area of triangle                          (  )     2Πr

4) Area of rectangle                       (  )     Πr2

                                                        (  )  \( \frac{\pi r^2}{2} \)

  1. 1 – c,2 – d,3 – a,4- b
  2. 1 – e,2 – a,3 – b,4- d
  3. 1 – a,2 – c,3 – b,4- d
  4. 1 – e,2 – b,3 – c,4- a

Answer: 1

21. The area of a rhombus is 60cm2 and one of its diagonals is 8 cm find the other diagonal.

  1. 10 cm
  2. 15 cm
  3. 20 cm
  4. 25 cm

Answer: 2

22. If the area of a rectangle is 144 m2 then its length and breadth are

  1. 18 m, 8 m
  2. 16 m, 9m
  3. 24 m, 6m
  4. All of them

Answer: 4

23. The perimeter of a square is 1 meter then its side is…..

  1. 10 cm
  2. 25 cm
  3. 20 cm
  4. 50 cm

Answer: 2

24. The product of length of two diagonals of a rhombus is 90 cm2 then its area is……

The product of length of two diagonals of a rhombus is 90 cm² then its area is

  1. 90 cm²
  2. 45 cm²
  3. 180 cm²
  4. 135 cm²

Answer: 2

25. Choose the correct matching.

Choose the correct matching

  1. i-c,ii-b,iii-a
  2. i-a,ii-c,iii-b
  3. i-a,ii-b,iii-c
  4. i-c,ii-a,iii-b

Answer: 2

26. ABCD is a rectangle. The ratio of the areas of ABCD and AED is………..

ABCD is a rectangle

  1. √2:1
  2. 2:1
  3. 3:1
  4. 3:√2

Answer: 2

27. If the outer radius of a circular path is R and its width is W, then its area in sq. units is

  1. Π(2R-W) W
  2. Π(R-W)W
  3. Π(R+W)W
  4. Π(R+W)(R-W)

Answer: 1

28. If the radii of two circles are in the ratio 9:16 then the ratio of their areas is

  1. 81:256
  2. 265:84
  3. 3:4
  4. 4:3

Answer: 1

29. If the perimeter of a circle is 16 times to the perimeter of square then the ratio of radius to the side is……….

  1. 112:11
  2. 11:112
  3. 11: 128
  4. 256: 11

Answer: 1

30. Statement – 1: If r is a radius of a circle then 2itr is the circumference of the cird.

Statement -II: If the side of the square is 14cm then the ratio of the square and circle perimeter is 14: 11

If r is a radius of a circle then 2r is the circumference of the circle

  1. Both statements are true.
  2. Both statements are false.
  3. Statement – 1 is true statement – II is false.
  4. Statement – 1 is false statement -II Is true.

Answer: 1

31. Statement – 1: If the inner and outer radii of circular rings arc 3.5 m and 7 m then area of the ring is 1185 m².

Statement -II: Area of the ring whose inner and outer radii are r, R is Π(R + r)(R – r) sq. units.

  1. Both statements are true. Statement – II is the correct explanation of statement -I.
  2. Both statements are true. Statement -II is not correct explanation statement-I.
  3. Statement – 1 is true, statement – II is false.
  4. Statement – I Is false, statement II is true.

Answer: 4

32 Base and height of a parallelogram are 6 cm and 13 cm respectively. Its area is

  1. 39 sq.m
  2. 78 sq.m
  3. 82 sq.m
  4. 38 sq.m

Answer: 2

33. Area of the parallelogram is 1470 cm2. Its base is 49 cm then the corresponding height is

  1. 42 cm
  2. 21cm
  3. 30 cm
  4. 32 m

Answer: 3

34. If the inner and outer radii of circular rings are 4m and 8m then the area of the ring is……..m².

  1. 148
  2. 150.86
  3. 160.81
  4. 140.72

Answer: 2

35. In ΔABC, ∠A = 90. AB=5 cm; AC= 12 cm; BC=13 cm, then area of ΔABC is

  1. 30 cm²
  2. 31.5 cm²
  3. 78 cm²
  4. 60 cm²

Answer: 1

36. The diagonals of a rhombus are li2 cm and 16 cm then its area is……square cms.

  1. 192
  2. 96
  3. 126
  4. 108

Answer: 2

37. The circumference of a circle whose radius is 4.2 cm

  1. 27.2 cm
  2. 26.4 cm
  3. 18.6 cm
  4. 13.2 cm

Answer: 2

38. If the circumference of a circle is 264 cm, then its radius is

  1. 36 cm
  2. 40 cm
  3. 42 cm
  4. 38 cm

Answer: 3

39. A road roller makes 200 rotations in covering 2200m then the radius of the road roller is

  1. 1.75 m
  2. 1.25 m
  3. 2.15 m
  4. 2.25 m

Answer: 1

40. In a rhombus ABCD, ∠A = 60°, AB = 6 cm then the length of the diagonal BD in cm is

  1. 12
  2. 9
  3. 6
  4. 3

Answer: 3

41. In the adjacent figure side of a square is 7cm. A circle is inscribed in the square. The perimeter of the circle is

The perimeter of the circle is

  1. 11 cm
  2. 44 cm.
  3. 22 cm
  4. 28 cm

Answer: 3

Observe the table and answer the following questions. (42 – 44)

Observe the table and answer the following questions

42. What is the formula for area of a rectangle?

  1. 2(l +b)
  2. l x b
  3. l ÷ b
  4. l + b

Answer: 2

43. What are the dimensions here?

  1. length
  2. breadth
  3. A or B
  4. A and B

Answer: 4

44. What information does this table give us?

  1. It gives formula for area of rectangle
  2. It gives formula for perimeter of rectangle
  3. A and B
  4. None

Answer: 3

45, What In the length of QS?

What In (he length of Qs

  1. 4 cm
  2. 3 cm
  3. 2 cm
  4. 6 cm

Answer: 2

46. Find the bane of n triangle whose area is 220 cm2 and height is 11 cm.

  1. 40 cm
  2. 50 cm
  3. 60 cm
  4. 70 cm

Answer: 1

Read the above table and answer the following questions

Read the above table and answer the following questions (47 – 49)

47. About which area this table tells us

  1. Parallelogram
  2. Rectangle
  3. Square
  4. Rhombus

Answer: 4

48. What are the values of a and x?

  1. 96 cm², 6 cm
  2. 6 cm, 96 cm²
  3. 30 cm², 6 cm
  4. 36 cm², 6 cm

Answer: 1

49. What is the formula for area of a rhombus?

  1. bh
  2. \( \frac{1}{2} \mathrm{bh} \)
  3. \( \frac{1}{2} \mathrm{~d}_1 \mathrm{~d}_2 \)
  4. l x b

Answer: 3

50. If the circumference is 30cm more than the diameter of the circle find the radius of the circle.

  1. 7 cm
  2. 8 cm
  3. 9 cm
  4. 10 cm

Answer: 1

51. Area of a semi-circle is 77cm². Its perimeter is equal to

  1. 35 cm
  2. 44 cm
  3. 42 cm
  4. 36 cm

Answer: 4

52. Area of the parallelogram is 1470 cm².Its base is 30 cm then the corresponding height

  1. 42 cm
  2. 21cm
  3. 49 cm
  4. 32 cm

Answer: 3

53. What is the length of DL if AB = 13 cm and area of parallelogram is 156 cm² ?

What is the length of DLif AB = 13 cm and area of parallelogram is 156 cm²

  1. 13 cm
  2. 12 cm
  3. 14 cm
  4. 15 cm

Answer: 2

54. What is the area of quadrilateral?

What is the area of quadrilateral

  1. 25 m²
  2. 35 m²
  3. 45 m²
  4. 55 m²

Answer: 3

55. What is the difference of circumferences of the circles shown ?

What is the difference of circumferences of the circles shown

  1. 22 cm
  2. 33 cm
  3. 44 cm
  4. 66 cm

Answer: 1

Haryana Board Class 7 Maths Solutions For Chapter 9 Fill in the blanks:

56. Perimeter of a regular polygon is……………..

Answer: Number of sides x Length of one side

57. All congruent triangles are equal in…………

Answer: area

58. The distance around a circular region is known as its ………

Answer: circumference

59. The perimeter of a parallelogram whose base is 5 units and height 3 units is………

Answer: 16 units

60. The area of a triangle is 36 cm² and the height is 3 cm. Its base is………..

Answer: 24 cm

Haryana Board Class 7 Maths Solutions For Chapter 9 Match the following:

61. Figure                             Area

1. Square                (   ) A) Length x Breadth

2. Rectangle           (   ) B) Base x Height

3. Triangle              (   ) C)r²

4. Parallelogram    (   ) D) (Side)²

5. Circle                  (   ) E) 1/2 x base X height

Answer: 1. D 2. A 3. E . 4. B 5. C

62.

1. Perimeter of a square                                                                                    (  ) A) π x diameter

2. Perimeter of a rectangle                                                                               (  ) B) 20 cm

3. Circumference of a circle                                                                              (  ) C) 4 X Side

4. Area of a rectangular sheet is 500 cm2, length is 25 cm its breadth is    (  ) D) 6m²

5. Base of a triangle is 3 cm, height is 4 cm its area is                                   (  ) E) 2 (Length + Breadth)

Answer: 1. C 2. E 3. A 4. B 5. D

Haryana Board Class 7 Maths Solutions For Chapter 7 Comparing Quantities

Haryana Board Class 7 Maths Solutions For Chapter 7 Comparing Quantities

1. Introduction: In our daily life, there are many occasions when we compare two quantities.

Example:

  1. Heena is 150 cm toll; Amir is 75 cm tall.
    1. Heena is two times taller than Amir.
    2. Amir’s height is 1/2 of Heena’s.
  2. Speed of cheetah is 120 km per hour. Speed of man is 20 km per hour.
    1. The speed of cheetah is 6 times the speed of man.
    2. The speed of a man is 1/6 of the speed of a cheetah.

In example: 1

1. We write the ratio of their heights as Heena’s height: Amir’s height – 150: 75 or 2: 1

In example: 2

2. We write the ratio of their speeds as

Speed of man: Speed of cheetah : 20: 120 or 1: 6

To compare two quantities, the units must be the same.

2. Ratio :

We compare two quantities of some kind by division. We use Y this symbol to express the ratio.

For any non-zero numbers a and b, a is to b is a ratio.

a/b is written as a: b.

‘a’ is known as the first term or antecedent.

‘b’ is known as the second term or consequent.

3. Equivalent ratio:

A ratio does not change if its first and second terms are multiplied or divided by the same non-zero number.

Example: 15: 25 (Multiplying by 2)

15X2: 25×2=30: 50

15:25 (Dividing by 5)

15 +5: 25 +5 = 3:5

HBSE Class 7 Comparing Quantities Solutions

4. Ratio in the Simplest Form:

A ratio a: b is said to be in the simplest form if its antecedent ‘a’ and consequent ‘b’ have no common factors except 1. A ratio in the simplest
form is also called the ratio in the lowest terms.

Example: 24: 72

24: 72=\frac{24}{72}=\frac{1}{3}=1: 3

5. Comparison of ratios:

Steps :

  1. Write each of the ratios in the form of a fraction in the simplest form.
  2. Find the LCM ofdenominators of the two fractions.
  3. Make each of the fractions its respective equivalent fractions in such way that the denominators should be equal to the LCM.
  4. Compare the numerators of each of the equivalent fractions. The fraction having the larger numerator will be larger than the other.

Example:

In 5: 6; 6:7 which is bigger ?

Solution:

\( 5: 6=\frac{5}{6}: 6: 7=\frac{6}{7} \)

LCM of 6 and 7 is 6×7=42

\( \frac{5}{6}=\frac{5 \times 7}{6 \times 7}=\frac{35}{42} ; \frac{6}{7}=\frac{6 \times 6}{7 \times 6}=\frac{36}{42} \) \( \text { In } \frac{35}{42}, \frac{36}{42} \text { the bigger fraction is } \frac{36}{42} \)

The ratio 6: 7 is bigger than 5: 6.

6. Proportion:

The ratios which are equivalent are said to be in proportion.

Four numbers a, b, c, d are in proportion ad=bc

Product ofmeans = Product of extremes;

Proportions are also used in the making of National flags

7. Unitary method:

The method offinding the value ofone article first from the value of the given number of articles and then the value of the required number of articles is called the unitary method.

Example: 6 bowls cost Rs. 90/-. What would be the cost of 10 such bowls ?

Solution:

Cost of 6 bowls = Rs. 90

Cost of 1 bowl = Rs \( \frac{90}{6} \)

Hence the cost of 10 bowls = Rs.\( \frac{90}{6} \times 10\)

= Rs. 15X10 = Rs. 150

Ratios also appear in the form of percentages.

We use percentages to express profit, loss,discount and interest.

Expressing them in percentages makes comparisons easy.

\( \begin{aligned}
& \text { gain/loss } \%=\frac{\text { gain } / l o s s ~}{\times 100} \\
& \text { Cost price } \\
& \text { Discount } \%=\frac{\text { Discount } \times 100}{\text { Marked price }}
\end{aligned} \)

The money borrowed or lent outfor a certain period is called the Principal. This money would be used by the borrowerfor some time before it is returned. For keeping this money for some time the borrower has to pay some extra money to the bank. This is known as Interest.

The amount that is to be repayed back is equal to the sum of the borrowed principal and the interest

That is Amount = Principal + Interest.

Interest is generally expressed as percent of the principal for a period of one year. It is written as say 10% peryear orper annum or in short as 10% p.a

Haryana Board Class 7 Maths Comparing Quantities solutions

1. Find the percentage of children of different heights for the following data.

Solution:

 

Find the percentage of children of different heights for the following data

2. A shop has the following number of shoe pairs of different sizes.
Size 2: 20; Size 3: 30; Size 4: 28; Size 5: 14; Size 6: 8

Write this information in tabular form as done earlier and find the Percentage of each shoe size available in the shop

Solution:

Write this information in tabular form as done earlier and find the Percentage of each shoe size available in the shop

1. A collection of 10 chips with different colours is given.

A collection of 10 chips with different colours is given

A collection of 10 chips with different colours is given 1

Fill the table and find the percentage of chips of each colour

Solution:

Fill the table and find the percentage of chips of each colour

Key Questions in Comparing Quantities for Class 7 HBSE

2. Mala has a collection of bangles. She has 20 gold bangles and10 silver bangles. What is the percentage of bangles of each type? Can you put it in the tabular form as donein the above example?

Solution:

Can you put it in the tabular form as done in the above example

1. Look at the examples below and in each of them, discuss which is better for comparison. In the atmosphere,1 g of air contains:

In the atmosphere,1 g of air contains

Solution: In the atmosphere the quantity of air contained in percent is better for comparison at a glance..

The second one is better for comparison.

2. A shirt has:

A shirt has

Solution:

Quantity of blending of cotton and polysterin a shirtis easy to understand in terms of percentage.

The second oneisbetter for comparison

1) Can you eat 50% of a cake ? Can you eat 100% of a cake ? Can you eat 150% of a cake ?

Solution:

Yes, we can eat 50% of a cake.

Yes, we can eat 100% of a cake.

No, we cannot eat 150% of a cake.

2) Can a price of an item go up by 50% ?

Can a price of an item go up by 100% ?

Can a price of an item go up by 150% ?

Solution:

Yes, theprice of anitemcan goupby 50%.

Yes, the price of an item can go up by 100%.

Yes, the price of an item can go up by 150%.

1. Convert the following to percents:

\( \frac{12}{16} \)

3.5

\( \frac{49}{50} \)

\( \frac{2}{2} \)

0.05

Solution:

1) \( \frac{12}{16}=\frac{12}{16} \times 100=75 \% \)

2) 3.5 = 3.5×100

\( =\frac{35}{10} \times 100=350 \% \)

3) \( \frac{49}{50}=\frac{49}{50} \times 100=98 \% \)

4) \( \frac{2}{2}=\frac{2}{2} \times 100=1 \times 100=100 \% \)

5) 0.05 = 0.05 x 100

\( =\frac{5}{100} \times 100=5 \% \)

2.1) Out of 32 students, 8 are absent. What present of the students are absent?

Solution:

Out of 32 students 8 are absent. Writing

this as a fraction we get \( \frac{8}{32} \).

\( \frac{8}{32} \times 100=25 \)

25% students are absent.

How to calculate percentage Class 7 HBSE

2) There are 25 radios, 16 of them are put of order. What percent of radios are out of order?

Solution:

Total number of radios = 25

Number of radios which are out of order = 16

Writing this as a fraction we get

\( \frac{16}{25} \) \( \frac{16}{25}=\frac{16}{25} \times 100=64 \% \)

64% of radios are out of order.

3) A shop has 500 items, out of which 5 are defective. What percent are defective ?

Solution:

Total number of items = 500

Number of items defective = 5

Writing this as a fraction = \( \frac{5}{500} \)

l% of items are defective

Practice Problems Comparing Quantities Class 7 Haryana Board

4) There are 120 voters, 90ofthem voted yes. What percent voted yes ?

Solution:

Total number of voters = 120

Numbers of voters voted yes = 90

Writing this as a fraction = \( \frac{90}{120} \)

\( \frac{90}{120}=\frac{90}{120} \times 100 \%=75 \% \)

75% voters voted yes.

Look at the table, observe, and complete it:

Look at the table, observe and completeit

1. 35% +________% = 100%;
64% + 20% +____________% = 100%
45% = 100% -____________%;
70% = _____________%-30%

Solution:

35% + 65% = 100%;
64% + 20% + 16% = 100%
45% = 100% -55%;
70% =100% -30%

2. If 65% of students in a class have a bicycle, what percent of the students do not have bicycles ?

Solution:

Number of students having bicycle in a class = 65%

Out of 100 students 65 of them have a bicycle.

Number of students do not have a bicycle = 100% – 65%. = 35%.

35% of students do not have a bicycle.

3. We have a basket full of apples, oranges and mangoes.If 50% are apples, 30% are oranges, then whatpercent are mangoes?

Solution:

Out of 1,00 given fruits, apples are 50,oranges are 30 and the remaining are mangoes.

Percentage of mangoes

= 100% -50% -30%
= 100%’-80% =20%

Consider the expenditure made on a dress. 20% on embroidery, 50%pn cloth, 30% on stitching. Can you think of more such examples ?

Solution:

Yes, some more examples are as follows:

1) Maths examination was conducted for 100 marks and was observed that 35% of the students,got marks below 50. 29% of the students gotmarks between 60
and 75. 46% of the students gotmarks above 80%.

2) An alloy is made of following compositions :

Copper: 35%; Nickel: 40%; Zinc: 25%

What percent of these figures are shaded?

Look at the table, observe and complete it

You can make some more figures yourself and ask your friends to estimate the shaded parts.

Solution:

1) Fractions which are,shaded

\( =\frac{1}{4}+\frac{1}{4}+\frac{1}{4}=\frac{3}{4} \)

Shaded triangular partin the fraction

\( =\frac{3}{4} \)

Percentage of shaded triangular part

\( =\frac{3}{4} \times 100 \%=75 \% \)

2) Fraction of tangram which is shaded

\( =\frac{1}{4}+\frac{1}{8}+\frac{1}{8}=\frac{2+1+1}{8}=\frac{4}{8}=\frac{1}{2} \)

Percentage of total shaded parts

\( =\frac{1}{2} \times 100 \%=50 \% \)

1. Find

1) 50% of 164

Solution: 50% of 164

\( =\frac{50}{100} \times 164=82 \)

2) 75% of 12

Solution: 75% of 12

\( =\frac{75}{100} \times 12=9 \)

3) \( 12 \frac{1}{2} \% \text { of } 64 \)

Solution:

\( \begin{aligned}
& 12 \frac{1}{2} \% \text { of } 64 \\
& \frac{25}{2} \% \text { of } 64=\frac{25}{2} \times \frac{1}{100} \times 64=8
\end{aligned} \)

2. 8% children of a class of 25 like getting wet in the rain. How many children like getting wet in the rain ?

Solution.

Children who like getting wet in the rain = 8% of 25

\( =\frac{8}{100} \times 25=2 \)

1. 9 is 25% of what number ?

Solution:

Let the required number be P

25% of P = 9

\( \begin{aligned}
& \frac{25}{100} \times P=9 \\
& \frac{P}{4}=9
\end{aligned} \)

P= 9 x 4 = 36

Required number is 36

Important Concepts Comparing Quantities Class 7 HBSE

2. 75% at what number is 15?

Salution:

let the number be P

75% of P=15

\( \begin{aligned}
& \frac{75}{100} \times P=15 \\
& \frac{3 P}{4}=15 \\
& P=15 \times \frac{4}{3}=20
\end{aligned} \)

Required number is 20

Haryana Board Class 7 Maths Solutions For Chapter 7 Exercise-7.1

1. Convert the given fractional numbers to percents.

1) \( \frac{1}{8} \)

Solution:

\( \begin{aligned}
& \frac{1}{8}=\frac{1}{8} \times 100 \% \\
& =\frac{25}{2} \%=12.5 \%
\end{aligned} \)

2. \( \frac{5}{4} \)

Solution:

\( \frac{5}{4}=\frac{5}{4} \times 100 \%=\frac{500}{4} \% \)

= 125%

Profit and loss formula Class 7 Haryana Board

3. \( \frac{3}{40} \)

Solution:

\( \begin{aligned}
& \frac{3}{40}=\frac{3}{40} \times 100 \% \\
& =\frac{15}{2} \%=7.5 \%
\end{aligned} \)

4. \( \frac{2}{7} \)

Solution:

\( \begin{aligned}
& \frac{2}{7}=\frac{2}{7} \times 100 \% \\
& =\frac{200}{7} \%=28 \frac{4}{7} \%
\end{aligned} \)

2. Convert the given decimal fractions to percents

1) 0.65

Solution:

\( \begin{aligned}
0.65 & =\frac{65}{100} \\
& =\frac{65}{100} \times 100 \% \\
& =65 \%
\end{aligned} \)

2) 2.1

Solution:

\( \begin{aligned}
2.1 & =\frac{21}{10} \\
& =\frac{21}{10} \times 100 \% \\
& =210 \%
\end{aligned} \)

3) 0.02

Solution:

\( \begin{aligned}
0.02 & =\frac{2}{100} \\
& =\frac{2}{100} \times 100 \% \\
& =2 \%
\end{aligned} \)

4) 12.35

Solution:

\( \begin{aligned}
12.35 & =\frac{1235}{100} \\
& =\frac{1235}{100} \times 100 \% \\
& =1235 \%
\end{aligned} \)

HBSE Class 7 Maths Chapter 7 Guide Comparing Quantities

3. Estimate what part of the figures are coloured and hence find the percent which is coloured.

Solution:

1) \( \begin{aligned}
&\frac{1}{4} \text { part is coloured }\\
&\frac{1}{4}=\frac{1}{4} \times 100 \%=25 \%
\end{aligned} \)

one fourth part is coloured

2) \( \begin{aligned}
&\frac{3}{5} \text { part is coloured }\\
&\frac{3}{5}=\frac{3}{5} \times 100 \%=60 \%
\end{aligned} \)

3 fifth part is coloured

3)

3 /8th part is coloured

\( \begin{aligned}
& \frac{3}{8} \text { part is coloured } \\
& \frac{3}{8}=\frac{3}{8} \times 100 \% \\
& \quad=\frac{75}{2} \%=37.5 \%
\end{aligned} \)

4. Find:

1) 15% of 250

Solution: 15% of 250

\( \begin{aligned}
&=\frac{15}{100} \times 250\\
&=\frac{75}{2}=37.5
\end{aligned} \)

2) 1% of 1 hour

Solution:

1% of 1 hour

\( \begin{aligned}
& =\frac{1}{100} \times 1 \text { hour }=\frac{1}{100} \text { hour } \\
& =\frac{1}{100} \times 60=\frac{3}{5} \text { minute } \\
& =\frac{3}{5} \times 60=36 \mathrm{sec}
\end{aligned} \)

3) 20% of Rs. 2500

Solution:

20% of 2500 \( =\frac{20}{100} \times \text { Rs. } 2500 \)

= Rs. 500

4) 75% of 1 kg

Solution:

75% of 1 kg.

\( \begin{aligned}
& =\frac{75}{100} \times 1 \mathrm{~kg}=\frac{3}{4} \mathrm{~kg} \\
& =\frac{3}{4} \times 1000=750 \mathrm{grams}
\end{aligned} \)

5. Find the whole quantity if

1) 5% of it is 600

Solution:

Let the whole quantity be P

5% of P is 600

\( \begin{aligned}
&\frac{5}{100} \times P=600\\
&\frac{P}{20}=600
\end{aligned} \)

P = 600 x 20 = 12000

The whole quantity is 12000.

2) 12% of it is Rs. 1080.

Solution:

Let the whole quantity he Rs. P

12% of Rs. P = Rs. 1060

\( \begin{aligned}
& \frac{12}{100} \times P=1080 \\
& \frac{3 P}{25}=1080
\end{aligned} \)

3P = 1060 x 25

\( P=\frac{1080 \times 25}{3}=\text { Rs. } 9000 \)

The whole quantity is Rs. 9000.

3) 40% of it is 500 km.

Solution:

Let the whole quantity be P km 40% of P km = 500 km

\( \frac{40}{100} \times P=500 \) \( \frac{2 P}{5}=500 \)

2P = 500 x 5

\( P=\frac{500 \times 5}{2}=1250 \mathrm{~km} \)

The whole quantity is 1230 km

4) 70% of it is 14 minutes

Solution:

Let the whole quantity be P minutes

70% of P minutes = 14 minutes

\( \begin{aligned}
& \frac{70}{100} \times P=14 \\
& \frac{7 \mathrm{P}}{10}=14
\end{aligned} \)

7P =14 x 10

\( P=\frac{14 \times 10}{7}=20 \)

The whole quantity is 20 minutes

3) 8% of it is 40 litres.

Solution:

let the whole quantity be P litres.

8% of P litres = 40 litres

\( \begin{aligned}
& \frac{8}{100} \times P=40 \\
& \frac{2 P}{25}=40 \\
& 2 P=40 \times 25 \\
& P=\frac{40 \times 25}{2}=500
\end{aligned} \)

The whole quantity is 500 litres

6. Convert given percents to decimal fractions and also to fractions in simplest forms:

1) 25%

Solution:

\( \begin{aligned}
& 25 \%=\frac{25}{100}=0.25 \\
& 25 \%=\frac{25}{100}=\frac{1}{4}
\end{aligned} \)

2) 150%

Solution:

\( \begin{aligned}
& 150 \%=\frac{150}{100}=1.50 \\
& 150 \%=\frac{150}{100}=\frac{15}{10}=\frac{3}{2}
\end{aligned} \)

3) 20%

Solution:

\( \begin{aligned}
& 20 \%=\frac{20}{100}=0.20 \\
& 20 \%=\frac{20}{100}=\frac{1}{5}
\end{aligned} \)

4) 5%

Solution:

\( \begin{aligned}
& 5 \%=\frac{5}{100}=0.05 \\
& 5 \%=\frac{5}{100}=\frac{1}{20}
\end{aligned} \)

Ratio and Proportion Class 7 Haryana Board

7. In a city, 30% are females, 40% are males and remaining are children. What percent are children?
Solution:

Total population of the city = 100%

Females – 30%

Males = 40%

Children = 100- (30 + 40)

=100-70 = 30%

8. Out of 15,000 voters in a constituency,60% voted. Find the percentage of voters who didnot vote. Can you now find how many actually didnot vote ?

Solution:

Number of voters in the constituency = 15,000

Voters who voted = 60%

Voters who did not vote = 100% -60% = 40%.

Actual number of voters who did not vote = 40% of 15,000

\( =\frac{40}{100} \times 15000=40 \times 150=6000 \text { voters } \)

9. Meeta saves Rs. 4000 from her salary.If this is 10% of her salary. What is her salary?

Solution: Let the salary of Meeta be Rs. P

Given Rs 4000 is 10% of her salary

10% of P = 4000

\( \begin{aligned}
& \frac{10}{100} \times P=4000 \\
& \frac{P}{10}=4000
\end{aligned} \)

P = 4000x 10 = 40000

The salary of Meetais Rs. 4000.

10. A local cricket team played 20 matches in one season.It won 25% of them. How many matches did they win?

Solution:

Let the total number of matches won be P.

Given 25% of matches were won.

25% of 20 = P

\( \frac{25}{100} \times 20=P \Rightarrow 5=P \)

Number of matches won by the team = 5

Solutions To Try These

1. Divide 15 sweets between Manu and Sonu so that they get 20% and 80% of them respectively.

Solution: Total number of sweets = 15

No. of sweets Manu gets = 20% of 15

\( =\frac{20}{100} \times 15=3 \)

No. of sweets Sonu gets = 80% of15

\( =\frac{80}{100} \times 15=12 \)

2. If angles of a triangle are in the ratio.2:3:4. Find the value of each angle.

Solution:

Angles of a triangle are in the ratio 2:3:4

Total of the parts = 2 + 3 + 4 = 9

Sum of the measures of the three angles of a triangle = 180°

\( \begin{aligned}
& \text { First angle }=\frac{2}{9} \times 180^{\circ}=40^{\circ} \\
& \text { Second angle }=\frac{3}{9} \times 180^{\circ}=60^{\circ} \\
& \text { Third angle }=\frac{4}{9} \times 180^{\circ}=80^{\circ}
\end{aligned} \)

The value of each angle = 40°, 60°, 80°

Solutions To Try These

1. Find percentage of increase or decrease:

1) Price of shirt decreased from Rs. 280 to Rs. 210.

Solution:

Decreased price of shirt- Rs. 280-Rs. 210

=Rs. 70

Percent decrease

\( \begin{aligned}
& =\frac{\text { Amount of decrease }}{\text { Original price }} \times 100 \% \\
& =\frac{70}{280} \times 100 \%=25 \%
\end{aligned} \)

2) Marks in a test increased from 20 to 30.

Solution:

Increased marks- 30 – 20 = 10

Percent increase

\( \begin{aligned}
& =\frac{\text { Increased marks }}{\text { Original marks }} \times 100 \% \\
& =\frac{10}{20} \times 100 \%=50 \%
\end{aligned} \)

HBSE 7th Class Comparing Quantities Word Problems

2. My mother says, in her childhood petrol was Rs.1 a litre.It is Rs. 52 per litre today.By what percentage has the prife gone up?

Solution:

Cost of 1 litre petrol in her childhood = Rs.1

Cost of 1 litre petrol today is = Rs. 52

Increase,in the price of 1 litre petrol = Rs. 52 – Rs. 1 = Rs. 51

Percentage of price increase

\( \begin{aligned}
& =\frac{\text { Amount increase }}{\text { Original cost }} \times 100 \% \\
& =\frac{51}{1} \times 100 \%=5100 \%
\end{aligned} \)

Solutions To Try These

1. A shopkeeper bought a chair for Rs. 375 and sold it for Rs 400. Find the gain percentage

Solution: C.P. of a chair = Rs. 375

S.P. of a chair = Rs. 400

SP>CP

Gain = S.P. – C.P

= Rs. 400 – Rs. 375- Rs. 25

= \( \begin{aligned}
& \frac{\text { Gain }}{\text { C.P. }} \times 100 \% \\
& \quad=\frac{25}{375} \times 100 \% \\
& =\frac{20}{3} \%=6 \frac{2}{3} \%
\end{aligned} \)

2. Cost of an item is Rs. 50.It was sold with a profit of 12%. Find the selling price.

Solution:

Cost price of an item Rs. 50

Gain = 12% of Rs. 50

\( =\frac{12}{100} \times 50=\text { Rs. } 6 \)

S.P.= C.P. + Gain = Rs. 50 + Rs. 6 = Rs. 56

The selling price of the item Rs. 56

3. An article was sold for Rs 250 with a profit of 5%. What was its cost price?

Solution:

Selling price of an article = Rs. 250

Gain = 5% of C.P.

\( \begin{aligned}
& =\frac{5}{100} \times \text { C.P. } \\
& =\frac{1}{20} \text { C.P. }
\end{aligned} \)

S.P. = C.P. + Gain

\( 250=\text { C.P. }+\frac{1}{20} \text { C.P. } \) \( \begin{aligned}
& 250=\text { C.P. }\left(1+\frac{1}{20}\right) \\
& 250=\text { C.P. } \times \frac{21}{20} \\
& \text { C.P. }=\frac{250 \times 20}{21}=\frac{5000}{21}=\text { Rs. } 238 \frac{2}{21}
\end{aligned} \)

The cost price of the article

\( =\text { Rs. } 238 \frac{2}{21} \)

4. An item was sold for Rs. 540 at a loss of 5%. What was its cost price?

Solution:

Selling price of an item = Rs. 540

Loss = 5%

Actual loss =5% of C.P

\( =\frac{5}{100} \times C . P .=\frac{1}{20} \text { C.P. } \)

S.P. = CP- Loss

\( \begin{aligned}
& 540=C P-\frac{1}{20} C . P . \\
& 540=C P\left(1-\frac{1}{20}\right) \\
& 540=C P \times \frac{19}{20}
\end{aligned} \) \( C P=540 \times \frac{20}{19}=\frac{10800}{19} \) \( C P=\text { Rs. } 568 \frac{8}{19} \)

The cost price of the item

\( =\text { Rs. } 568 \frac{8}{19} \)

Solutions To Try These

1. Rs. 10,000 is invested at 5% interest rate p.a. Find the interest at the end of one year.

Solution:

Principal (P) = Rs. 10,000

Rate of interest (R) = 5%

Interest at the end of one year \( I=\frac{P R T}{100} \)

\( =\frac{10.000 \times 5 \times 1}{100}=\text { Rs. } 500 \)

Interest at the end of one year = Rs.500

Sample Problems Comparing Quantities Haryana Board Class 7

2. Rs. 3,500 is given at 7%. p.a. rate of interest. Find the interest which will be received at the end of two years.

Solution:

Principal (P) = Rs. 3,500

Rate of interest (R) = 7%

Time (T) = 2 years

Interest at the end of two years

\( \begin{aligned}
I & =\frac{P \times R \times T}{100} \\
& =\frac{3500 \times 7 \times 2}{100}=\text { Rs. } 490
\end{aligned} \)

Interest at the end of two years is Rs. 490

3. Rs. 6,050 is borrowed at 6.5% rate of interest p.a. Find the interest and the amount to be paid at the end of 3 years.

Solution:

Principal (P) = Rs, 6,050

Rate of interest (R) =6.5%

Time (T) =3 years

Interest at the end of 3 years

\( \begin{aligned}
& I=\frac{P \times R \times T}{100}=\frac{6050 \times 6.5 \times 3}{100} \\
& =\frac{6050 \times 65 \times 3}{100 \times 10}=\frac{1179750}{1000}
\end{aligned} \)

= Rs. 1179.75

Amount (A) = Principal + Interest

Amount to be paid at tire end of 3 yearsis

= Rs. 6050 + Rs. 1179.75

= Rs. 7229.75

4. Rs. 7,000 is borrowed at 3.5% rate of interest p.a. borrowed for 2 years. Find the amount to be paid at the end of the second year.

Solution:

Principal (P) = Rs. 7,000

Rate of Interest (R) = 3.5%

Time (T) = 2 years

Interest to be paid at the end of second year

\( \begin{aligned}
& I=\frac{P \times R \times T}{100} \\
& =\frac{7000 \times 3.5 \times 2}{100}=\text { Rs. } 490
\end{aligned} \)

Amount (A) = Principal + Interest

Amount to be paid at the end of the second year is = Rs. 7000 + Rs. 490 = Rs. 7490

Solutions To Try These

1. You have Rs. 2400 in your account and the interest rate is 5% After how many years would you earn Rs. 240 as interest.

Solution:

Principal (P) = Rs. 2,400

Rate of interest (R) = 5%

Interest (I) = Rs. 240

Time (T) =?

\( \begin{aligned}
& \text { Interest } \mathrm{I}=\frac{\mathrm{P} \times \mathrm{R} \times \mathrm{T}}{100} \\
& 240=\frac{2400 \times 5 \times \mathrm{T}}{100} \\
& 240=24 \times 5 \times \mathrm{T} \\
& \mathrm{~T}=\frac{240}{24 \times 5}=2
\end{aligned} \)

After 2 years you would earn Rs. 240 as interest.

2. On a certain sum the interest paid after 3 years is Rs. 450 at 5% rate of interest per annum. Find the sum

Solution:

Let the sum be Rs. P

Rate of interest (R) = 5%

Time (T) = 3 years

Interest (I) = Rs. 450

\( I=\frac{P \times R \times T}{100} \) \( 450=\frac{P \times 5 \times 3}{100} \) \( P=\frac{450 \times 100}{5 \times 3}=\text { Rs. } 3000 \)

The required sum = Rs. 3000

Haryana Board Class 7 Maths Solutions For Chapter 7 Exercise-7.2

1. Tell what is the profit or loss in the following transactions. Also find profit percent (or) loss per centin each case.

1) Gardening shears bought forRs. 250 and sold for Rs. 325.

Solution:

CP of gardening shears = Rs. 250

SP of gardening shears = Rs. 325

SP>CP

Profit = SP- CP = Rs. 325- Rs. 250

= Rs.75

\( \begin{aligned}
\text { Profit \% } & =\frac{\text { Profit }}{C P} \times 100 \% \\
& =\frac{75}{250} \times 100 \%=30 \%
\end{aligned} \)

2) A refrigerator bought for Rs. 12,000 and sold at Rs. 13,500

Solution: CP of refrigerator = Rs. 12,000.

SP of refrigerator = Rs. 13,500

SP>CP

Profit = SP- CP

= Rs. 13,500- Rs. 12,000 = Rs. 1,500

\( \begin{aligned}
& \text { Profit } \%=\frac{\text { Profit }}{\text { CP }} \times 100 \% \\
& =\frac{1500}{12000} \times 100 \%=12.5 \%
\end{aligned} \)

3) A cupboard bought for Rs. 2500 and sold at Rs. 3,000

Solution:

CP of cupboard = Rs. 2,500

SP of cupboard = Rs. 3,000

SP>CP

Profit = SP- CP

= Rs. 3000- Rs. 2500

= Rs. 500

\( \begin{aligned}
& \text { Profit } \%=\frac{\text { Profit }}{C P} \times 100 \% \\
& =\frac{500}{2500} \times 100 \%=20 \%
\end{aligned} \)

Word problems on profit and loss Class 7 HBSE Maths

4) A skirt bought for Rs. 250 and sold at Rs. 150.
Solution:

CP of skirt = Rs. 250.

SP of a skirt = Rs. 150

CP>SP

Loss = CP- SP

= Rs. 250- Rs. 150

= Rs. 100

\( \begin{aligned}
& \text { Loss % }=\frac{\text { Loss }}{\text { CP }} \times 100 \% \\
= & \frac{100}{250} \times 100 \%=40 \%
\end{aligned} \)

2. Convert each part of the ratio to percentage:

1) 3:1

Solution: Given ratio 3:1

Total parts =3 +1=4

Percentage of first part

\( =\frac{3}{4} \times 100 \%=75 \% \)

Percentage of second part

\( =\frac{1}{4} \times 100 \%=25 \% \)

2) 2:3:5

Solution:

Given ratio = 2:3:5

Total parts = 2 + 3 + 5 = 10

Percentage of first part

\( =\frac{2}{10} \times 100 \%=20 \% \)

Percentage of second part

\( =\frac{3}{10} \times 100 \%=30 \% \)

Percentage of third part

\( =\frac{5}{10} \times 100 \%=50 \% \)

3) 1:4

Solution:

Given ratio 1:4

Total parts =1+4 = 5

Percentage of first part

\( =\frac{1}{5} \times 100 \%=20 \% \)

Percentage of second part

\( =\frac{4}{5} \times 100 \%=80 \% \)

4) 1:2:5

Solution: Given ratio 1:2:5

Total parts =l+2+5=8

Percentage of firstpart

\( =\frac{1}{8} \times 100 \%=\frac{25}{2} \%=12.5 \% \)

Percentage of second part

\( =\frac{2}{8} \times 100 \%=25 \% \)

Percentage of third part

\( =\frac{5}{8} \times 100 \%=\frac{125}{2} \% \)

= 62.5%

3. The population of a city decreased from 25,000 to 24,500. Find the percentage of decrease.

Solution:

Population at the begining = 25,000

Population after = 24,500

Actual decrease = 500

Percent decrease =

\( \begin{aligned}
& \frac{\text { changein population }}{\text { population at the begining }} \times 100 \% \\
& \quad=\frac{500}{25000} \times 100 \%=2 \%
\end{aligned} \)

4. Arun bought a car for Rs. 3,50,000 The next year, theprice wentupto Rs. 3,70,000. What was the percentage of price increase?

Solution:

Price of the carin the first year = Rs. 3,50,000

Price of the carin the next year = Rs. 3,70,000

Increased price of the car
= Rs. 3,70,000 -Rs. 3,50,000
= Rs. 20,000

Percent increase

\( \begin{array}{r}
=\frac{\text { Increased price }}{\text { Pricein the first year }} \times 100 \% \\
=\frac{20,000}{3,50,000} \times 100 \%=\frac{40}{7} \%=5 \frac{5}{7} \%
\end{array} \)

5. I buy a TV for Rs. 10,000 and sell it at a profit of 20%. How much money do I get for it ?

Solution:

C.P. of a TV= Rs. 10,000

Profit = 20% of C.P

\( =\frac{20}{100} \times 10,000=\text { Rs. } 2000 \)

SP of TV = CP +’Profit
= Rs. 10,000 + Rs. 2,000
= Rs. 12,000
I will get Rs. 12,000 for it.

6. Juhi sells a washing machine for Rs. 13,500. She loses 20% in the bargain. What was the price at which she bought it?

Solution: SP of washing machine = Rs. 13,500

Loss =20% of CP
Now SP = CP- Loss
13,500 = CP -20% of CP

\( \begin{aligned}
& 13,500=C P-\frac{20}{100} C P \\
& 13,500=C P-\frac{1}{5} C P \\
& 13,500=C P\left(1-\frac{1}{5}\right) \\
& 13,500=\frac{4}{5} C P \\
& \frac{13,500 \times 5}{4}=C P
\end{aligned} \)

CP =3375 X 5 = Rs. 16,875

Juhi bought the washing machine at Rs. 16,875

7.

1) Chalk contains calcium, carbon and oxygen in the ratio10:3: 12. Find the percentage of carbon in chalk.

Solution:

Given ratio =10:3:12

Total parts = 10 + 3 + 12 = 25

Percent of carbon = \( \frac{3}{25} \times 100 \%=12 \% \)

2) If in a stick of chalk,carbon is 3 g, what is the weight of the chalk stick?

Solution:

Let the weight of the chalk stick be

P grams.

12% of P = 3g

\( \frac{12}{100} \times P=3 \) \( P=\frac{3 \times 100}{12}=25 \mathrm{~g} \)

The weight of the chalk stick = 25 g

8. Amina buys a book for Rs. 275 and sells it at a loss of 15%. How much does she sell it for ?

Solution:

CP of book = Rs. 275

Loss = 15% of CP

\( \begin{aligned}
& =\frac{15}{100} \times 275 \\
& =\frac{4125}{100}=\text { Rs. } 41.25
\end{aligned} \)

SP of the book = CP – Loss

= Rs. 275 -Rs. 41.25

= Rs. 233.75

Amina sells the book for Rs. 233.75

Percentage Problems Class 7 HBSE

9. Find the amount to be paid at the end of 3 years in each case:

1) Principal = Rs. 1,200 at 12% p.a.

Solution:

Principal (P) = Rs. 1,200

Rate of interest (R) =12% pa.

Time (T) = 3 years

\( \begin{aligned}
& \text { Interest } \mathrm{I}=\frac{\mathrm{P} \times \mathrm{R} \times \mathrm{T}}{100} \\
& =\frac{1200 \times 12 \times 3}{100}=\text { Rs. } 432
\end{aligned} \)

Amount (A) = P +1

= Rs. 1200 + Rs. 432

= Rs. 1632

Amount to be paid at the end of 3 years, is Rs. 1632.

2) Principal = Rs. 7,500 at 5%pa.

Solution:

Principal (P) = Rs. 7500

Rate of interest (R) = 5% p.a.

Time (T) = 3 years

\( \text { Interest } \mathrm{I}=\frac{\mathrm{P} \times \mathrm{R} \times \mathrm{T}}{100}=\frac{7500 \times 5 \times 3}{100} \)

= Rs.1125

Amount(A)= P+I

= Rs. 7300 + Rs 1125

= Rs, 8625

Amount to be paid at the end of 3 years is Rs. 8625.

10. What rate gives Rs. 280 as interest on a sum of Rs. 56,000in 2 years ?

Solution:

Principal (P) = Ps. 56,000

Interest (T) = Rs. 280

Time (T) =2 years

Rate of interest (R) =?

\( \begin{aligned}
& I=\frac{P R T}{100} \Rightarrow R=\frac{I \times 100}{P T} \\
& R=\frac{280 \times 100}{56000 \times 2}=\frac{1}{4}=0.25 \%
\end{aligned} \)

The rate of interest is 0.25 percent per annum.

11. If Meena gives an interest of Rs. 45 for one year at 9% rate pa. What is the sum she has borrowed?

Solution:

Let the sum Meena borrowed be Rs. P.

Rate of interest (R) = 9% per annum

Time (T) =1 year

Interest (I) = Rs. 45

\( \begin{aligned}
& I=\frac{P \times R \times T}{100} \\
& 45=\frac{P \times 9 \times 1}{100}
\end{aligned} \) \( P=\frac{45 \times 100}{9} \)

= 5 x 100 = Rs. 500

Meena has borrowed a sum of Rs.500

Haryana Board Class 7 Maths Solutions For Chapter 7 Very Short Answer Questions

1. What is meant by ‘ratio’?

Solution:

The comparison of two quantities is known as ratio.

2. Find the ratio of 3 km to 300 m.

Solution:

3 km = 3 X 1000 m = 3000 m

The required ratio = 3 km: 300 m

= 3000:300 =10:1.

3. Write the equivalent ratio of 2: 3

Solution:

\( 2: 3=\frac{2}{3}=\frac{2 \times 2}{3 \times 2}=\frac{4}{6}=4: 6 \)

4. What is meant by ‘Unitary method’?

Solution:

The method in which we find the value of one unit first and then the value of the required number of units is known as unitary method.

5. What is a ‘proportion’?

Solution:

If two ratios are equivalent then the four quantities are said to be in proportion.

The ratios 8: 2 and 16: 4 are equivalent.

8, 2, 16, 4 arein proportion

6. Find 25% of 40

Solution:

\( 25 \% \text { of } 40=\frac{25}{.100} \times 40=10 \)

7. If Manohar pays an.interest of Rs 750 for 2 years on a sum of Rs 4,500 find the rate of interest.

Solution:

Principal (P)=Rs 4500

Interest (I) = Rs 750

Time (T) = 2 years

Rate (R) =?

\( I=\frac{P \times T \times R}{100} \) \( 750=\frac{4500 \times 2 \times \mathrm{R}}{100} \) \( R=\frac{750 \times 100}{4500 \times 2}=\frac{25}{3}=8 \frac{1}{3} \% \)

8. Last year the cost of 1000 articles was 5000 /-. This year, it went down to 4000/-. Find the percentage decrease in price.

Solution:

Original cost = 5000/-

Present cost = 4000/-

decreasein cost = 1000

% decrease in price = \( \frac{1000}{5000} \) x 100

= 20 %

9. Out,of 12,000 voters in a constituency, 60% voted. Find the No. of people voted in the constituency?

Solution:

By Question, 60% voted means \( \frac{60}{100} \)

Persons voted =\( \frac{12000×60}{100} \) = 7200

Simple Interest formula examples Class 7 HBSE

10. 40% of a number is 800 then find the number.

Solution:

40% of the number is 800

1 % of the number is \( \frac{800}{40} \)

and 100% (Actual No) = \( \frac{800}{40} \) x 100

= 2000

11. Tamarind was soldlast year at? 75/-perkg. This year it is sold at 125 per kg. Find the percentage increase in price?

Solution:

Percentage increase = \( \frac{\text { dif.in price }}{\text { original price }} \times 100 \)

The increase in tamarind price

= \( \frac{125-75}{75} \) x 100

= \( \frac{50}{75} \) x 100

\( =66 \frac{2}{3} \% \)

12. Suppose a person buys an article for 650/- and gains 6% on selling it. Find the selling price?

Solution:

CP = 650

Profit % = 6%

Thus, profit = 6% of 650

= \( \frac{6}{100} \) x 650 = 39

We know that SP = CP + Profit

=650+39=689

Thus, the SP = 689

13. Ajay bought a TV for 15,000 and sold it for 14,100. Find the loss percent.

Solution:

By Question, loss =? (15,000.- 14,100) = 900

% loss = \( \frac{900 \times 100}{C P} \)

\( \frac{900 \times 100}{15000} \)

= 6%

14. The marked Price of a book is 225. The publisher allows a discount of 10% onit. Find the selling price of it.

Solution:

MP = 225; discount = 10%

SP (100 down) = \( \frac{100 \text {-discount }}{100} \times \text { M.P } \)

\( \frac{225}{100} \times { 90 } \)

= 202.50

15. A dealer allows a discount of 10% and still gains by 10%. Find the marked price of an article which cost him 900.

Solution:

Given CP = 900

gain =10%

SP (100 down) =

\( =\frac{100+\text { gain } \times C . P}{100} \)

\(\frac{900}{100} \times { 110 } \) = 990

again, discount = 10%

MP (100 up) = \( \frac{\mathrm{SP} \times 100}{100-\text { discount }} \)

\(\frac{990×100}{90} \) = 1100

16. Find the interest on a sum of? 8250 for 3 years at the rate of 8% per annum.

Solution:

Principal (P) = 8250

Rate of interest (R) = 8%

Interest per 3 years (I)

\( \begin{aligned}
& =\mathrm{P} \times \frac{\mathrm{R}}{100} \times \mathrm{T} \\
& =8250 \times \frac{8}{100} \times 3
\end{aligned} \)

17. 3000 is lent out at 9% rate of interest. Find the interest which will be received at the end of 2½ years.

Solution:

Amount (P) = 3000,

Rate of Interest (R) = 9%.

\( \mathrm{T}=2 \frac{1}{2} \mathrm{yr} \) \( I=\frac{P \times R \times T}{100}=3000 \times \frac{9}{100} \times \frac{5}{2} \)

= 675

18. A child-friendly bank annnounces a , savings scheme for school children. Theywill give kiddybanks to children. Children have to keep their savings in it and the bank collects all the money once in a year. To encourage children savings, they give 6% interest if the amount exceeds 10,000, and otherwise 5%. Find the interest received by the school if they deposit 9000 for one year.

Solution:

Since the deposit is only 9,000 the children get 5% interest only.

Interest = \( \frac{\text { PTR }}{100}=\frac{9000 \times 1 \times 5}{100} \)

= 450

Haryana Board Class 7 Maths Solutions For Chapter 7 Short Answer Questions

19. Shyam’smonthlyincomeis?10,000.He spends 60% of it on family expenses 10% onmedical expenses, 5% on donations and saves by 25%. Find the
amount he spends on each item.

Solution:

Amount spent on family expenses

= 60% of total income

= 60% of 10000

\( =\frac{60}{100} \times 10000 \) = 6000

Similarly, amount spent on medical expenses

\( =\frac{10}{100} \times 10000 \) = 1000

Amount spent on donations

\( =\frac{5}{100} \times 10000 \) = 500

Amount saved = \( =\frac{25}{100} \times 10000 \) = 2500

20. Ramu sold a plot of land for 2,40,000 gaining 20%. Find the costprice ofplot.

Solution:

Method -1:

SP = 2, 40,000 gain 20%

\( \mathrm{CP}=\frac{\mathrm{SP} \times 100}{100+\operatorname{gain} \%} \)

CP (100 up) = \( \frac{2,40,000 x 100}{120} \)

= 2,00,000

Method – 2: Since Ramu gained 20% his CP : SP = 100 : 120

By question 100: 120 = x : 2,40,000

x = \( \frac{2,40,000 x 100}{120} \)

= 2,00,000

21. A shopkeeper gives successive discounts of10% and5% onhis articles. Find the net discount on the whole.

Solution:

Let his MP be 100

by two successive discounts,his SP (100

down) = 100x \( \frac{90}{100} \) x \( \frac{95}{100} \)= \( \frac{171}{2} \)

= 85.50

Net discountis (100- 85.5) = 14.5,%

22. In what time will? 6880 amount to 7224, if simple interest is calculated at 10% per annum?

Solution:

Amount = ? 7224

Principle = ? 6880

S.I = Amount – Principle

= 7224-6880 = 344

R% = 10%

\( \begin{aligned}
& \text { Now } I=P \times \frac{R}{100} \times T \\
& 344=6880 \times \frac{10}{100} \times T
\end{aligned} \)

344 x 100 = 6880 x 10 x T

Therefore, T = \( \frac{344×100}{6880×10} \)

= \( \frac{1}{2} \) years

= 6 months

23. What sum will yield an interest of 3927 in 2 years and 4 months at 8% per annum?

Solution:

S.I =3927,

R% = 8%

T = 2 year + 4 months

\( \begin{aligned}
& \left(2+\frac{4}{12}\right) \mathrm{Yrs} \\
& \left(2+\frac{1}{3}\right) \mathrm{Yrs}=\frac{7}{3} Y \mathrm{rs}
\end{aligned} \) \( \text { Substituting in } \mathrm{I}=\mathrm{P} \times \frac{\mathrm{R}}{100} \times \mathrm{T} \) \( 3927=\mathrm{P} \times \frac{8}{100} \times \frac{7}{3} \)

3927 x 100 x 3 = P x 8 x 7

Therefore,P =\( \frac{3927x100x3}{8×7} /latex]

Thus, P = 21037.50

Therefore, Principal. = 21037.50

Haryana Board Class 7 Maths Solutions For Chapter 7 Long Answer Questions

24. A shopkeeper bought a TV for 9000 and he sold it for 10,000. Find the profit or loss? calculate percentage.

Solution:

Method-1:

Cost price (CP) of the TV = 9000

Selling price (SP) of the TV = 10,000

As SPis greater than CP, the shopkeeper makes a profit:

Profit (P) = 10000 -9000 = 1000

Thus, when the CP is 9000, the shopkeeper makes a profit of 1000

The ratio of profit and cost price is [latex] \frac{1000}{9000} \)

To find the profit percentage we multiply this ratio with 100%

i.e, \( \frac{1000}{9000} \) x 100%

\( =\frac{100}{9} \%=11 \frac{1}{9} \% \)

Method -II:

When the CP is 9000, the profit is  1000 Now, when CP is? 100, let the profit be x.

We know that the CP and profit are directly proportional thus, ratio of profit and the ratio of cost price (CP) will be same in both cases.

Therefore, x: 1000 = 100: 9000

\( \frac{x}{1000}=\frac{100}{9000} \)

9000 x x = 1000 x 100

\( \frac{1000 x 100}{9000} \) \( =11 \frac{1}{9} \% \)

Thus, the profit % \( =11 \frac{1}{9} \% \)

25. In 4 years ? 6,500 amounts to ? 8840 at a certain rate of interest.In what time will 1600 amount to 1816 at the same rate?

Solution:

Case (1):

P = 6500

T = 4 yrs

A = 8840 such that I = (8840- 6500)

= 2340

we have to find rate %

we know \( \frac{\mathrm{PTR}}{100}=\mathrm{I} \)

\( \frac{6500 \times 4 \times \mathrm{R}}{100}=2340 \)

R = \( \frac{2340×100}{6500×4} \) = 9 %

Case (2):

given P = 1600 A =1816

T = ? (such that I=(1816-1600)=216)

R = 9%

Substituting these values in \( \frac{\mathrm{PTR}}{100}=\mathrm{I} \)

we get 1600 x T x \( \frac{9}{100} \) = 216

T = \( \frac{216×100}{1600×9} \) = \( \frac{3}{2} \)

\( =1 \frac{1}{2} \mathrm{yrs} \)

Haryana Board Class 7 Maths Solutions For Chapter 7 Multiple Choice Answer Questions

1. Ratio of Rs 20 to Rs 50 is

  1. 2: 5
  2. 5: 2
  3. 2: 3
  4. 3: 5

Answer: 1

2. Equivalent ratio of 6 : 4 is

  1. 2:8
  2. 8:2
  3. 8:12
  4. 12:8

Answer: 4

3. Percentages are numerators of fractions with denominator

  1. 10
  2. 100
  3. 1000
  4. 50

Answer: 2

4. If the cost of 10 cans of juice is Rs 200 then the cost of 6 cans of juice is

  1. Rs 180
  2. Rs 160
  3. Rs 120
  4. Rs 140

Answer: 3

5. \( \mathrm{I}=\frac{\mathrm{P} \times \mathrm{R} \times \mathrm{T}}{100} \) in this formula P is called

  1. Interest
  2. Amount
  3. Principal
  4. Rate of interest

Answer: 3

6.Convert \( \frac{1}{3} \) as percent.

  1. \( \frac{3}{100} \% \)
  2. \( \frac{100}{3} \% \)
  3. \( \frac{50}{3} \% \)
  4. \( \frac{3}{50} \% \)

Answer: 2

7. Choose the correct matching.

1) 0.2                     (  )           a) 75%

2) 0.09                  (  )            b) 20%

3) 0.75                  (  )            c) 9%

  1. 1-a,2-b,3-c
  2. 1-b,2-a,3-c
  3. 1-a,2-c,3-b
  4. 1-b,2-c,3-a

Answer: 4

8. If 2: 5 and 6 : x are equal then find fourth proportion.

  1. 20
  2. 25
  3. 15
  4. 30

Answer: 3

9. lire ratio of two angles is 3: 2 and the larger angle is 60. What is the smaller one ?

  1. 40
  2. 50
  3. 60
  4. 70

Answer: 1

10. Choose the correct matching

1) %                (   )         a) Rate

2) /                 (   )         b) Proportion

3) ::                 (   )         c) Percent

  1. 1- a,2-b,3-c
  2. 1- c,2-a,3-b
  3. 1- b,2-a,3-c
  4. 1- c,2-b,3-a

Answer: 2

11. 72% + 15% + X = 100% => X = ………

  1. 15%
  2. 20%
  3. 18%
  4. 13%

Answer: 4

12. If x and y arein direct proportion, then which of the following is true ?

  1. \( \frac{x}{y} \text { is constant } \)
  2. xy is constant
  3. \( \frac{1}{x} \text { is constant } \)
  4. y is constant

Answer: 1

13. If C.P of 12 mangoes is equal to the selling price of 15 mangoes, find the loss percentage.

  1. 10%
  2. 20%
  3. 30%
  4. 40%

Answer: 2

14. If 25% ofa number is 300 then the number is………..

  1. 1200
  2. 1250
  3. 1300
  4. 1350

Answer: 1

15. If the cost of 10 pens is Rs. 300/- then the cost of one pen is…………..

  1. Rs. 30
  2. Rs. 25
  3. Rs. 50
  4. Rs. 17.50

Answer: 1

16. If 3, 8, x and 16 are in proportion. The value of x is…….

  1. 12
  2. 4
  3. 8
  4. 6

Answer: 4

17. The ratio of 8 books to 20 books.

  1. 2:5
  2. 5:2
  3. 4: 5
  4. 5: 4

Answer: 1

18. What sum will yield an interest of Rs. 1000/-in 2 years at 5% (in Rs) ?

  1. 10,000
  2. 12,000
  3. 15,000
  4. 20,000

Answer: 1

19. At what rate will ?10,000 amount to ?16,000in 3 yrs ?

  1. 10%
  2. 15%
  3. 20%
  4. 25%

Answer: 3

20. An amount becomes double in 5 yrs at some SI rate. Find the rate %.

  1. 25%
  2. 20%
  3. 15%
  4. 10%

Answer: 2

21.What sum will amount to ? 880in 2 years at 5% SI (in Rs)

  1. 600
  2. 660
  3. 800
  4. 500

Answer: 3

22. Certain sum amounts to ? 7000in 4 yrs and? 6000 in 2 years, the rate % is

  1. 10%
  2. 5%
  3. \( 12 \frac{1}{2} \% \)
  4. \( 6 \frac{1}{4} \% \)

Answer: 1

23. If the compound ratio of 5: 8 and 3: 7 is 45: x, then x is

  1. 128
  2. 72
  3. 168
  4. 105

Answer: 3

24. By selling an article for 1100 which is marked at 1200, discount percent is

  1. \( 6 \frac{1}{4} \% \)
  2. \( 12 \frac{1}{2} \% \)
  3. \( 8 \frac{1}{3} \% \)
  4. 10%

Answer: 3

25. The 30% of 40% of a number is 69 then the number is

  1. 557
  2. 575
  3. 757
  4. 775

Answer: 2

26. At what rate per annum will X 6360 yield an interest of 1378 in \( 2 \frac{1}{2} \) years ?

  1. \( 6 \frac{2}{3} \% \)
  2. \( 8 \frac{1}{3} \% \)
  3. \( 8 \frac{2}{3} \% \)
  4. \( 7 \frac{1}{4} \% \)

Answer: 3

27. Ratio’s of sellingprice and costprice are 4: 5, then percentage is

  1. 10%
  2. 20%
  3. 19%
  4. 25%

Answer: 2

28. Profit is calculated on

  1. selling price
  2. cost price
  3. marked price
  4. above allAnswer: 2

29. Discount is calculated on

  1. marked price
  2. selling price
  3. cost price
  4. none of the above

Answer: 1

30. 21% of 85 =?

  1. 16.85
  2. 17.05
  3. 18.05
  4. 17.85

Answer: 4

31. If 3: 5 = 4.5: x then x – 5 = ?

  1. 7.5
  2. 5
  3. 4.5
  4. 2.5

Answer: 4

32. 8% of 400 – 4% of 800 + 1% of 500 = ?

  1. 10
  2. 20
  3. 4
  4. 5

Answer: 4

33. What is the mean proportion of 25: 20:: 20: 16 ?

  1. 25
  2. 20
  3. 16
  4. 100

Answer: 2

34. If 25% of a number is 40 then the number is

  1. 100
  2. 140
  3. 160
  4. none

Answer: 3

35. While doing the problem “whatis the cost of 9 bananasif the cost of a dozen bananas is 20 ?” Write the steps in order.

1) Cost of 9 bananas =\( \frac{20}{12} \times 9 \) = 15

2) Cost of 12 bananas = 20

3) Cost of 1 banana = \( \frac{20}{12} \)

  1. 1,2,3
  2. 2,1,3
  3. 2,3,1
  4. 1,3,2

Answer: 3

36. 40% of a numberis 800 then find the number.

  1. 1000
  2. 2000
  3. 3000
  4. 4000

Answer: 2

37. If the price of an item goes up by 100%, then the cost of article will be

  1. doubled
  2. tripled
  3. five times
  4. eight times

Answer: 1

38. When 35% is expressedin decimalit is equal to

  1. 0.35
  2. 0.7
  3. 0.14
  4. 0.21

Answer: 1

39. \( \frac{4}{16} \) is equal to……..%

  1. 30%
  2. 25%
  3. 35%
  4. 40%

Answer: 2

40. Arectangleis enlarged fromits originalposition. Which conceptis usedin this aspect?

  1. Profit
  2. Percentage
  3. Proportion
  4. Simple interest

Answer: 3

41. Two is 10% of x and 20% of y, what is x-y ?

  1. 1
  2. 2
  3. 5
  4. 10

Answer: 4

42. The markedprice of a machineis Rs. 18,000.Ifitis sold at 20% discount, there is a loss of 4%. Then its costin Rs. is

  1. 15,000
  2. 16,000
  3. 14,000
  4. 13,000

Answer: 1

43. A is 20% less than B. At what percent B is more than A?

  1. 25%
  2. 20%
  3. 18%
  4. 22%

Answer: 1

44. What percentage of numbers from1 to 70 has1 or 9in its unit’s digit ?

  1. 1%
  2. 14%
  3. 20%
  4. 21%

Answer: 3

45. Rani’s salary is increased from 630 to 700, find the increase percent.

  1. \( 11 \frac{1}{9} \% \)
  2. \( 8 \frac{1}{2} \% \)
  3. \( 13 \frac{2}{9} \% \)
  4. \( 9 \frac{1}{9} \% \)

Answer: 1

46. 12% of a certain sum of money is 43.5. Find the sum.

  1. 360
  2. 362.50
  3. 340.50
  4. 352.50

Answer: 2

47. Out of 40 students 5 are absent. What percent of the students are present ?

  1. \( 12 \frac{1}{2} \% \)
  2. 40%
  3. \( 87 \frac{1}{2} \% \)
  4. 4%

Answer: 3

48. The C.P of 25 articles is equal to the S.P of 20 articles. Whatis the gain% ?

  1. 0.25%
  2. 2.5%’
  3. 25%
  4. 0.025%

Answer: 3

49. A man buys aradio for ? 500 and sellsit at a gain of 25%. What is the S.P of the radio?

  1. 600
  2. 575
  3. 625
  4. 675

Answer: 3

50. At what rate of interest per annum will a sum double itselfin 8 years ?

  1. \( 6 \frac{1}{4} \% \)
  2. \( 12 \frac{1}{2} \% \)
  3. \( 11 \frac{1}{2} \% \)
  4. \( 11 \frac{1}{4} \% \)

Answer: 2

51. Suppose aperson buys an article for? 650 and gains 6% on sellingit. The sellingprice is

  1. 700
  2. 650
  3. 698
  4. 689

Answer: 4

52. Ramu sold a plot of land for? 24000 gaining 20%. The cost price of the plot is

  1. 28,000
  2. 2,800
  3. 20,000
  4. 19,200

Answer: 3

53. 3000 is lent out at 9% rate of interest. The interest which will be received at the end of \( 2 \frac{1}{2} \) years is

  1. 675
  2. 725
  3. 756
  4. 657

Answer: 1

54. At what rate per annum will the principal triples in 16 years ?

  1. 25%
  2. 24%
  3. \( 12 \frac{1}{2} \% \)
  4. 20%

Answer: 3

55. At what rate per annus will ? 6360 yield an interest of 1378in 2 \( [/2 \frac{1}{2}latex] years.

  1. [latex] 6 \frac{2}{3} \% \)
  2. \( 8 \frac{1}{3} \% \)
  3. \( 8 \frac{2}{3} \% \)
  4. \( 7 \frac{1}{4} \% \)

Answer: 4

56. What is the decimal form of this figure?

What is the decimal form of this figure ?

  1. \( \frac{17}{10} \)
  2. 0.17
  3. 0.18
  4. 1.8

Answer: 3

Fill in the blanks:

57. To compare two quantities the units must be the ………

Answer:

58. 1 Dozen =………..items.

Answer:

59. Percentage is derived from the Latin word…………

Answer:

60. The buying price of any item is known as its …………….

Answer:

61. Amount = ………+………..

Answer:

62. Match the following:

1. Out of 25 children in a class 15 are girls. The percentage of girls is items.                                         (  ) A) 20%

2. Write – as a percent                                                                                                                                  (  ) B) 50%

3. Convert 0.2 as a percent                                                                                                                           (  ) \( 33 \frac{1}{3} \% \)

4. A school team won 6 games this year against 4 games won last year. Write the percent increase. (  ) D) 0.25%

5. What rate gives Rs 280 as interest on a sum of Rs 56000 in 2 years?                                                  (  ) E) 60%

Answer:

1. E 2. C 3. A 4. B 5. D

Haryana Board Class 7 Maths Solutions For Chapter 8 Rational Numbers

Haryana Board Class 7 Maths Solutions For Chapter 8 Rational Numbers

Key Concepts

  1. 1. Introduction :
    1. The numbers used for counting objects around us are called Counting numbers (or) Natural numbers They are 1, 2, 3, 4,……,…
    2. Including ‘0’ to natural numbers we get the whole numbers i.e. 0,1,2,3,4, ………
    3. The negatives of natural numbers were put together with whole numbers to make up integers. They are……..- 3,-2,-1, 0,1,2,3,……….
    4. The numbers of the form \( \frac{\text { numerator }}{\text { denominator }} [latex] where the numerator is either 0 or a positive integer and the denominator, a positive integer are called fractions.
      Need for rational numbers: We know that integers can be. used to denote opposite situations involving numbers.
  2. Definition of a rational number:
    A rational number is defined as number that can be expressed in the form of [latex] \frac{p}{q} \) where p and q are integers and q ≠ 0.

Example :

\( \frac{4}{5} ; \frac{-3}{4} ; \frac{3}{8} ; 1 \frac{2}{3} \text { etc. } \) \( 0.5=\frac{5}{10} ; 0.333=\frac{333}{1000} \text { etc. } \)

1. Is the number \( \frac{2}{-3} \) rational ? Think about it.

Solution: Yes; \( \frac{2}{-3} \) is a rational number.

It is in the form of latex] \frac{p}{q} [/latex], where p = 2; q = -3 both are integers.

2. List ten rational numbers.

Solution:

\( \frac{3}{8}, \frac{2}{3}, \frac{-3}{2}, \frac{4}{-9}, \frac{1}{2}, \frac{3}{4}, \frac{-6}{11}, 2 \frac{3}{5}, \) \( 0.75=\frac{75}{100}, \frac{17}{79} . \)

HBSE Class 7 Rational Numbers Solutions

Fill in the boxes:

1)

Fill in the boxes

Solution:

\( \begin{aligned}
& \frac{5}{4}=\frac{5 \times 4}{4 \times 4}=\frac{20}{16} \\
& \frac{5}{4}=\frac{5 \times 5}{4 \times 5}=\frac{25}{20} \\
& \frac{5}{4}=\frac{5 \times(-3)}{4 \times(-3)}=\frac{-15}{-12} \\
& \frac{5}{4}=\frac{20}{16}=\frac{25}{20}=\frac{-15}{-12}
\end{aligned} \)

2)

Fill in the boxes 2

Solution:

\( \begin{aligned}
& \frac{-3}{7}=\frac{-3 \times 2}{7 \times 2}=\frac{-6}{14} \\
& \frac{-3}{7}=\frac{-3 \times(-3)}{7 \times(-3)}=\frac{9}{-21} \\
& \frac{-3}{7}=\frac{-3 \times 2}{7 \times 2}=\frac{-6}{14} \\
& \frac{-3}{7}=\frac{-6}{14}=\frac{9}{-21}=\frac{-6}{14}
\end{aligned} \)

Solutions To Try These

1. Is 5 a positive rational number?

Solution:

Yes,5 is a positive rational number. It can be written as \( \frac{5}{1} \) . The numerator is 5 and denominator is 1.

2. List five more positive rational numbers.

Solution:

\( \frac{3}{7}, \frac{5}{12}, \frac{4}{19}, \frac{6}{13}, \frac{17}{9} \)

Solutions To Try These

1. Is – 8 a negative rational number?

Solution:

Yes,- 8 is a negative rational number. It can be written as \( \frac{-8}{1} \) . The numerator is a negative integer and the denominator is
a positive integer.

2. List five more negative rational numbers.

Solution:

\( \frac{-4}{9}, \frac{-7}{11}, \frac{-5}{11}, \frac{-15}{22}, \frac{-3}{10} \)

Solutions To Try These

Which of these are negative rational numbers?

  1. \( \frac{-2}{3} \)
  2. \( \frac{5}{7} \)
  3. \( \frac{3}{-5} \)
  4. 0
  5. \( \frac{6}{11} \)
  6. \( \frac{-2}{-9} \)

Solution:

1) \( \frac{-2}{3} \) and 3. \( \frac{3}{-5} \) are negative rational numbers.

Solutions To Try These

Find the standard form of

1) \( \frac{-18}{45} \)

Solution: The HCF of 18 and 45 is 9.

\( \frac{-18}{45}=\frac{-18 \div 9}{45 \div 9}=\frac{-2}{5} \)

2) \( \frac{-12}{18} \)

Solution: The HCF of 12 and 18 is 6.

\( \frac{-12}{18}=\frac{-12 \div 6}{18 \div 6}=\frac{-2}{3} \)

Solutions To Try These

Find five rational numbers between

\( \frac{-5}{7}\) and \( \frac{-3}{8}\).

Solution:

\( \frac{-5}{7}=\frac{-5 \times 8}{7 \times 8}=\frac{-40}{56} \) \( \frac{-3}{8}=\frac{-3 \times 7}{8 \times 7}=\frac{-21}{56} \)

\( \frac{-40}{56}<\frac{-39}{56}<\frac{-38}{56}<\frac{-29}{56} \) \( <\frac{-27}{56}<\frac{-22}{56}<\frac{-21}{56} \)

\( \frac{-5}{7}<\frac{-39}{56}<\frac{-38}{56}<\frac{-29}{56}<\frac{-27}{56}<\frac{-22}{56} \) \( <\frac{-3}{8} \)

The five rational numbers between \( <\frac{-5}{7} \) and \( <\frac{-3}{8} \) are

\( \frac{-39}{56}, \frac{-38}{56}, \frac{-29}{56}, \frac{-27}{56}, \frac{-22}{56} . \)

Haryana Board Class 7 Maths Solutions For Chapter 8  Exercise-8.1

1. List five rational numbers between: (1) -1 and 0 . (2) -2 and -1

(3) \( \frac{-4}{5} \text { and } \frac{-2}{3} \)

4. \( \frac{-1}{2} \text { and } \frac{2}{3} \)

Solution:

First we find equivalent rational numbers having same denominator.

1) \( -1=\frac{-1}{1}=\frac{-1 \times 10}{1 \times 10}=\frac{-10}{10} \)

\( 0=\frac{0}{1}=\frac{0 \times 10}{1 \times 10}=\frac{0}{10} \) \( \Rightarrow \frac{-10}{10}<\frac{-9}{10}<\frac{-8}{10}<\frac{-7}{10}<\frac{-6}{10}<\frac{-5}{10}<\frac{0}{10} \) \( \Rightarrow-1<\frac{-9}{10}<\frac{-8}{10}<\frac{-7}{10}<\frac{-6}{10}<\frac{-5}{10}<0 \)

The five rational numbers between -1 and 0 are

\( \frac{-9}{10}, \frac{-8}{10}, \frac{-7}{10}, \frac{-6}{10}, \frac{-5}{10} \)

Haryana Board Class 7 Maths Rational Numbers Solutions

2) – 2 and -1

Solution:

\( -2=\frac{-2}{1}=\frac{-2 \times 10}{1 \times 10}=\frac{-20}{10} \) \( -1=\frac{-1}{1}=\frac{-1 \times 10}{1 \times 10}=\frac{-10}{10} \) \( \begin{aligned}
\Rightarrow \frac{-20}{10} & <\frac{-19}{10}<\frac{-18}{10}<\frac{-17}{10} \\
& <\frac{-16}{10}<\frac{-15}{10}<\frac{-10}{10}
\end{aligned} \) \( \begin{aligned}
\Rightarrow-2<\frac{-19}{10}<\frac{-18}{10}<\frac{-17}{10} & <\frac{-16}{10} \\
& <\frac{-15}{10}<-1
\end{aligned} \)

The five rational numbers between -2 and -1 are

\( \frac{-19}{10}, \frac{-18}{10}, \frac{-17}{10}, \frac{-16}{10}, \frac{-15}{10} \)

HBSE 7th Class Rational Number Word Problems

3) \( \frac{-4}{5} \text { and } \frac{-2}{3} \)

Solution:

\( \frac{-4}{5}=\frac{-4 \times 9}{5 \times 9}=\frac{-36}{45} \) \( \frac{-2}{3}=\frac{-2 \times 15}{3 \times 15}=\frac{-30}{45} \) \( \begin{aligned}
\Rightarrow \frac{-36}{45}<\frac{-35}{45} & <\frac{-34}{45}<\frac{-33}{45} \\
& <\frac{-32}{45}<\frac{-31}{45}<\frac{-30}{45}
\end{aligned} \) \( \begin{aligned}
\Rightarrow \frac{-4}{5}<\frac{-35}{45}<\frac{-34}{45}<\frac{-33}{45} & <\frac{-32}{45} \\
& <\frac{-31}{45}<\frac{-2}{3}
\end{aligned} \)

The five rational numbers between \( \frac{-4}{5} \) and \( \frac{-2}{3} \text { are } \) are

\( \frac{-35}{45}<\frac{-34}{45}<\frac{-33}{45}<\frac{-32}{45}<\frac{-31}{45} \)

Class 7 Maths Chapter 8 Rational Numbers Haryana Board

4) \( \frac{-1}{2} \text { and } \frac{2}{3} \)

Solution:

\( \begin{aligned}
& \frac{-1}{2}=\frac{-1 \times 3}{2 \times 3}=\frac{-3}{6} \\
& \frac{2}{3}=\frac{2 \times 2}{3 \times 2}=\frac{4}{6}
\end{aligned} \) \( \begin{aligned}
& \Rightarrow \frac{-3}{6}<\frac{-2}{6}<\frac{-1}{6}<0<\frac{1}{6}<\frac{2}{6}<\frac{4}{6} \\
& \Rightarrow-\frac{1}{2}<\frac{-2}{6}<\frac{-1}{6}<0<\frac{1}{6}<\frac{2}{6}<\frac{2}{3}
\end{aligned} \)

The five rational numbers between \( \frac{-1}{2} \text { and } \frac{2}{3} \text { are } \)

\( \frac{-2}{6}, \frac{-1}{6}, 0, \frac{1}{6}, \frac{2}{6} \)

2. Write four more rational numbers in each of the following patterns :

1) \( \frac{-3}{5}, \frac{-6}{10}, \frac{-9}{15}, \frac{-12}{20} \ldots . . \)

Solution:

\( \frac{-3}{5}=\frac{-3 \times 1}{5 \times 1} ; \frac{-6}{10}=\frac{-3 \times 2}{5 \times 2} \) \( \frac{-9}{15}=\frac{-3 \times 3}{5 \times 3} ; \frac{-12}{20}=\frac{-3 \times 4}{5 \times 4} \)

Thus, we observe a pattern in these numbers.

The next four numbers would be

\( \frac{-3 \times 5}{5 \times 5}=\frac{-15}{25} ; \frac{-3 \times 6}{5 \times 6}=\frac{-18}{30} \) \( \frac{-3 \times 7}{5 \times 7}=\frac{-21}{35} ; \frac{-3 \times 8}{5 \times 8}=\frac{-24}{40} \)

The required four rational numbers are

\( \frac{-15}{25}, \frac{-18}{30}, \frac{-21}{35}, \frac{-24}{40} \)

2) \( \frac{-1}{4}, \frac{-2}{8}, \frac{-3}{12}, \ldots . \)

Solution:

\( \frac{-1}{4}=\frac{-1 \times 1}{4 \times 1} ; \frac{-2}{8}=\frac{-1 \times 2}{4 \times 2} \) \( \frac{-3}{12}=\frac{-1 \times 3}{4 \times 3} \)

Thus we observe a pattern in these numbers.

The next four numbers would be

\( \begin{aligned}
& \frac{-1 \times 4}{4 \times 4}=\frac{-4}{16} ; \frac{-1 \times 5}{4 \times 5}=\frac{-5}{20} \\
& \frac{-1 \times 6}{4 \times 6}=\frac{-6}{24} ; \frac{-1 \times 7}{4 \times 7}=\frac{-7}{28}
\end{aligned} \)

The required four rational numbers are

\( \frac{-4}{16}, \frac{-5}{20}, \frac{-6}{24}, \frac{-7}{28} \)

Haryana Board 7th Class Maths Rational Numbers Questions and Answers

3) \( \frac{-1}{6}, \frac{2}{-12}, \frac{3}{-18}, \frac{4}{-24}, \ldots \ldots \)

Solution:

\( \frac{2}{-12}=\frac{-1 \times(-2)}{6 \times(-2)} ; \frac{3}{-18}=\frac{-1 \times(-3)}{6 \times(-3)} \) \( \frac{4}{-24}=\frac{-1 \times(-4)}{6 \times(-4)} \)

Thus we observe a pattem in these numbers.

\( \begin{aligned}
&\frac{-1 \times(-5)}{6 \times(-5)}=\frac{5}{-30} ; \frac{(-1) \times(-6)}{6 \times(-6)}=\frac{6}{-36} ;\\
&\frac{(-1) \times(-7)}{6 \times(-7)}=\frac{7}{-42} ; \frac{(-1) \times(-8)}{6 \times(-8)}=\frac{8}{-48}
\end{aligned} \)

The required four numbers are \( \frac{5}{-30} ; \frac{6}{-36} \)

\( \frac{7}{-42}, \frac{8}{-48} \)

4) \( \frac{-2}{3}, \frac{2}{-3}, \frac{4}{-6}, \frac{6}{-9} \ldots . \)

Solution:

\( \begin{aligned}
&\frac{2}{-3}=\frac{-2 \times(-1)}{(-3) \times(-1)} ; \frac{4}{-6}=\frac{(-2) \times(-2)}{3 \times(-2)}\\
&\frac{6}{-9}=\frac{(-2) \times(-3)}{(3) \times(-3)}
\end{aligned} \)

Thus we observe a pattern in these numbers.

The next four numbers would be

\( \begin{aligned}
&\frac{-2 \times(-4)}{3 \times(-4)}=\frac{8}{-12} ; \frac{(-2) \times(-5)}{3 \times(-5)}=\frac{10}{-15}\\
&\frac{(-2) \times(-6)}{3 \times(-6)}=\frac{12}{-18} ; \frac{(-2) \times(-7)}{3 \times(-7)}=\frac{14}{-21}
\end{aligned} \)

The required four numbers are

\( \frac{8}{-12} ; \frac{10}{-15} ; \frac{12}{-18} ; \frac{14}{-21} \)

3. Give four rational numbers equivalent to:

(1)\( \frac{-2}{7} \)
(2)\( \frac{5}{-3} \)
(3)\( \frac{4}{9} \)

Solution:

(1)

\( \begin{aligned}
& \frac{-2}{7}=\frac{-2 \times 2}{7 \times 2}=\frac{-4}{14} \\
& \frac{-2}{7}=\frac{-2 \times 3}{7 \times 3}=\frac{-6}{21} \\
& \frac{-2}{7}=\frac{-2 \times 4}{7 \times 4}=\frac{-8}{28} \\
& \frac{-2}{7}=\frac{-2 \times 5}{7 \times 5}=\frac{-10}{35}
\end{aligned} \)

Thefourrational numbers equivalent to \( \frac{-2}{7} \text { are } \frac{-4}{14}, \frac{-6}{21}, \frac{-8}{28}, \frac{-10}{35} \)

(2)\( \frac{5}{-3} \)

Solution:

\( \frac{5}{-3}=\frac{5 \times 2}{-3 \times 2}=\frac{10}{-6} \) \( \begin{aligned}
& \frac{5}{-3}=\frac{5 \times 3}{-3 \times 3}=\frac{15}{-9} \\
& \frac{5}{-3}=\frac{5 \times 4}{-3 \times 4}=\frac{20}{-12} \\
& \frac{5}{-3}=\frac{5 \times 5}{-3 \times 5}=\frac{25}{-15}
\end{aligned} \)

The four rational numbers equivalent

\( \text { to } \frac{5}{-3} \text { are } \frac{10}{-6}, \frac{15}{-9}, \frac{20}{-12}, \frac{25}{-15} \)

Sample Problems Rational Numbers Haryana Board Class 7

(3)\( \frac{4}{9} \)

Solution:

\( \begin{aligned}
& \frac{4}{9}=\frac{4 \times 2}{9 \times 2}=\frac{8}{18} \\
& \frac{4}{9}=\frac{4 \times 3}{9 \times 3}=\frac{12}{27} \\
& \frac{4}{9}=\frac{4 \times 4}{9 \times 4}=\frac{16}{36} \\
& \frac{4}{9}=\frac{4 \times 5}{9 \times 5}=\frac{20}{45}
\end{aligned} \)

The four rational numbers equivalent to \( \frac{4}{9} \) are

\( \frac{8}{18}, \frac{12}{27}, \frac{16}{36}, \frac{20}{45} \)

4. Draw the number line and represent the following rational numbers on it:

1) \( \frac{3}{4} \)

Solution:

Draw the number line and represent the following rational numbers on it

2) \( \frac{-5}{8} \)

Solution:

Draw the number line and represent the following rational numbers on it 2

3) \( \frac{-7}{4} \)

Solution:

Draw the number line and represent the following rational numbers on it 3

\( \frac{-7}{4}=\frac{-4-3}{4}=\frac{-4}{4}-\frac{3}{4}=-1-\frac{3}{4} \)

4) \( \frac{7}{8} \)

Solution:

Draw the number line and represent the following rational numbers on it 4

5. The points P, Q, R, S, T, U, A, and B on the number line are such that, TR = RS = SU and AP= PQ = QB. Name the rational numbers represented by P, Q, R, and S.

Solution:

Name the rational numbers represented by P, Q, R and S

The points P, Q, R,S on the number line such that TR = RS = SU

\( \mathrm{TR}=\mathrm{RS}=\mathrm{SU}=\frac{1}{3} \mathrm{TU} \) \( \begin{aligned}
& \mathrm{TR}=\frac{1}{3} \text { unit } \\
& \text { and } \mathrm{AP}=\mathrm{PQ}=\mathrm{QB} \\
& \mathrm{AP}=\mathrm{PQ}=\mathrm{QB}=\frac{1}{3} \mathrm{AB}
\end{aligned} \) \( \mathrm{AP}=\frac{1}{3} \text { unit. } \)

The rational number represented by P

\( P=2+\frac{1}{3}=\frac{6+1}{3}=\frac{7}{3} \)

The rational number represented by Q,

\( \mathrm{Q}=2+\frac{1}{3}+\frac{1}{3}=\frac{6+1+1}{3}=\frac{8}{3} \)

The rational number represented by R,

\( R=(-1)+\left(\frac{-1}{3}\right)=\frac{-1-3}{3}=\frac{-4}{3} \)

The rational number represented by S,

\( S=(-1)+\left(\frac{-1}{3}\right)+\left(\frac{-1}{3}\right)=\frac{-3-1-1}{3}=\frac{-5}{3} \)

6. Which of the following pairs represent the same rational number ?

1) \( \frac{-7}{21} \text { and } \frac{3}{9} \)

Solution:

\( \frac{-7}{21} \) is a negative rational number.

\( \frac{3}{9} \) is a positive rational number.

The given pair does not represent the same rational number.

2)

\( \frac{-16}{20} \text { and } \frac{20}{-25} \)

Solution:

\( \frac{-16}{20}=\frac{-16 \div 4}{20 \div 4}=\frac{-4}{5}=\frac{(-4) \times(-1)}{5 \times(-1)}=\frac{4}{-5} \) \( \frac{20}{-25}=\frac{20 \div 5}{-25 \div 5}=\frac{4}{-5} \)

The given pair represents the same rational number

3) \( \frac{-2}{-3} \text { and } \frac{2}{3} \)

Solution:

\( \frac{-2}{-3}=\frac{(-2) \times(-1)}{(-3) \times(-1)}=\frac{2}{3} \)

The given pair represents the same rational number.

4) \( \frac{-3}{5} \text { and } \frac{-12}{20} \)

Solution:

\( \frac{-3}{5}=\frac{-3 \times 4}{5 \times 4}=\frac{-12}{20} \)

The given pair represents the same rational number.

5) \( \frac{8}{-5} \text { and } \frac{-24}{15} \)

\(  \frac{8}{-5} \text { and } \frac{-24}{15} \) \( \frac{8}{-5}=\frac{8 \times 3}{-5 \times 3}=\frac{24}{-15}=\frac{24 \times(-1)}{(-15) \times(-1)}=\frac{-24}{15} \)

The given pair represents the same rational number.

6) \( \frac{1}{3} \text { and } \frac{-1}{9} \)

Solution:

\( \frac{1}{3} \) is a positive rational number.

\( \frac{-1}{9} \) is a negative rational number.

The given pair does not represent the same rational number.

7) \( \frac{-5}{-9} \text { and } \frac{5}{-9} \)

Solution:

\( \frac{-5}{-9}=\frac{-5 \times(-1)}{-9 \times(-1)}=\frac{5}{9} \)

\( \frac{5}{9} \) is a positive rational number.

\( \frac{5}{-9} \) is a negative rational number.

The given pair does not represent the same rational number.

Operations on Rational Numbers Class 7 HBSE

7. Rewrite the following rational numbers in the simplest form:

1) \( \frac{-8}{6} \)

Solution:

\( \frac{-8}{6} \)

HCF of 8 and 6 is 2.

\( \frac{-8}{6}=\frac{-8 \div 2}{6 \div 2}=\frac{-4}{3} \)

2) \( \frac{25}{45} \)

Solution:

\( \frac{25}{45} \)

HCF of 25 and 45 is 5.

\( \frac{25}{45}=\frac{25 \div 5}{45 \div 5}=\frac{5}{9} \)

Chapter 8 Rational Numbers Class 7 Solutions in Hindi Haryana Board

3) \( \frac{-44}{7 \cdot 2} \)

Solution:

\( \frac{-44}{7 \cdot 2} \)

HCF of 44 and 72 is 4.

\( \frac{-44}{72}=\frac{-44 \div 4}{72 \div 4}=\frac{-11}{18} \)

4) \( \frac{-8}{10} \)

Solution:

\( \frac{-8}{10} \)

HCF of 8 and 10 is 2.

\( \frac{-8}{10}=\frac{-8 \div 2}{10 \div 2}=\frac{-4}{5} \)

8. Fill in the boxes with the correct symbol out of >, < and =

1) \( \frac{-5}{7}\) Fill in the boxes with the correct symbol out of \( \frac{2}{3}\)

Solution:

LCM of 7 and 3 is 21.

\( \begin{aligned}
& \frac{-5}{7}=\frac{-5 \times 3}{7 \times 3}=\frac{-15}{21} \\
& \frac{2}{3}=\frac{2 \times 7}{3 \times 7}=\frac{14}{21}
\end{aligned} \) \( \text { Hence } \frac{-5}{7}<\frac{2}{3} \)

2) \( \frac{-4}{5} \) Fill in the boxes with the correct symbol out of \( \frac{-5}{7} \)

Solution:

LCM of 5 and 7 is 35

\( \frac{-4}{5}=\frac{-4 \times 7}{5 \times 7}=\frac{-28}{35} \) \( \begin{aligned}
&\frac{-5}{7}=\frac{-5 \times 5}{7 \times 5}=\frac{-25}{35}\\
&\text { Hence } \frac{-4}{5}<\frac{-5}{7}
\end{aligned} \)

3) \( \frac{-7}{8} \) Fill in the boxes with the correct symbol out of \( \frac{14}{-16} \)

Solution:

\( \begin{aligned}
&\frac{-7}{8}=\frac{-7 \times(-2)}{8 \times(-2)}=\frac{14}{-16}\\
&\text { Hence } \frac{-7}{8}=\frac{14}{-16}
\end{aligned} \)

4) \( \frac{-8}{5} \) Fill in the boxes with the correct symbol out of \( \frac{-7}{4} \)

Solution:

LCM of 5 and 4 is 20

\( \begin{aligned}
&\begin{aligned}
& \frac{-8}{5}=\frac{-8 \times 4}{5 \times 4}=\frac{-32}{20} \\
& \frac{-7}{4}=\frac{-7 \times 5}{4 \times 5}=\frac{-35}{20}
\end{aligned}\\
&\text { Hence } \frac{-8}{5}>\frac{-7}{4}
\end{aligned} \)

5) \( \frac{1}{-3} \) Fill in the boxes with the correct symbol out of \( \frac{-1}{4} \)

Solution:

LCM of3 and 4 is 12

\( \begin{aligned}
&\begin{aligned}
& \frac{1}{-3}=\frac{1 \times 4}{-3 \times 4}=\frac{4}{-12}=\frac{4 \times(-1)}{(-12) \times(-1)}=\frac{-4}{12} \\
& \frac{-1}{4}=\frac{-1 \times 3}{4 \times 3}=\frac{-3}{12}
\end{aligned}\\
&\text { Hence } \frac{1}{-3}<\frac{-1}{4}
\end{aligned} \)

6) \( \frac{5}{-11} \) Fill in the boxes with the correct symbol out of \( \frac{-5}{11} \)

Solution:

\( \begin{aligned}
&\frac{5}{-11}=\frac{5 \times(-1)}{(-11) \times(-1)}=\frac{-5}{11}\\
&\text { Hence } \frac{5}{-11}=\frac{-5}{11}
\end{aligned} \)

Equivalent Rational Numbers Class 7 Haryana Board

7) 0 rec \( \frac{-7}{6} \)

Solution: \begin{aligned}
& 0=\frac{0}{6} \\
& \text { Hence } 0>\frac{-7}{6}
\end{aligned}

9. Which is greater in each of the following:

1) \( \frac{2}{3}, \frac{5}{2} \)

Solution:

LCM of 3 and 2 is 6

\( \begin{aligned}
& \frac{2}{3}=\frac{2 \times 2}{3 \times 2}=\frac{4}{6} \\
& \frac{5}{2}=\frac{5 \times 3}{2 \times 3}=\frac{15}{6} \\
& \frac{15}{6}>\frac{4}{6}
\end{aligned} \) \( \frac{5}{2}>\frac{2}{3} \)

Important Questions for Class 7 Maths Chapter 8 Haryana Board

2) \( \frac{-5}{6}, \frac{-4}{3} \)

Solution: LCM of 6 and 3 is 6.

\( \begin{aligned}
& \frac{-5}{6}=\frac{-5 \times 1}{6 \times 1}=\frac{-5}{6} \\
& \frac{-4}{3}=\frac{-4 \times 2}{3 \times 2}=\frac{-8}{6} \\
& \frac{-5}{6}>\frac{-8}{6}
\end{aligned} \) \( \frac{-5}{6}>\frac{-4}{3} . \)

3) \( \frac{-3}{4}, \frac{2}{-3} \)

LCM of 4 and 3 is 12.

Solution:

\( \begin{aligned}
& \frac{-3}{4}=\frac{-3 \times 3}{4 \times 3}=\frac{-9}{12} \\
& \frac{2}{-3}=\frac{2 \times 4}{-3 \times 4}=\frac{8}{-12}=\frac{8 \times(-1)}{(-12) \times(-1)}=\frac{-8}{12}
\end{aligned} \) \( \begin{aligned}
&\frac{-8}{12}>\frac{-9}{12}\\
&\frac{-2}{3}>\frac{-3}{4}
\end{aligned} \) \( \frac{2}{-3}>\frac{-3}{4} \)

4) \( \frac{-1}{4}, \frac{1}{4} \)

Solution: \( \frac{1}{4}>\frac{-1}{4} \)

5) \( -3 \frac{2}{7},-3 \frac{4}{5} \)

Solution:

\( -3 \frac{2}{7}=\frac{-23}{7} \quad ; \quad-3 \frac{4}{5}=\frac{-19}{5} \)

LCM of 7 and 5 is 35.

\( \begin{aligned}
& \frac{-23}{7}=\frac{-23 \times 5}{7 \times 5}=\frac{-115}{35} \\
& \frac{-19}{5}=\frac{-19 \times 7}{5 \times 7}=\frac{-133}{35}
\end{aligned} \) \( \begin{aligned}
\frac{-115}{35} & >\frac{-133}{35} \\
\frac{-23}{7} & >\frac{-19}{5}
\end{aligned} \) \( \text { Hence }-3 \frac{2}{7}>-3 \frac{4}{5} \)

10. Write the following rational numbers in ascending order :

1) \( \frac{-3}{5}, \frac{-2}{5}, \frac{-1}{5} \)

Solution:

Denominators of each rational number is 5.

-3<-2<-l

\( \frac{-3}{5}<\frac{-2}{5}<\frac{-1}{5} \) \( \text { Ascending order is } \frac{-3}{5}, \frac{-2}{5}, \frac{-1}{5} \)

2) \( \frac{-1}{3}, \frac{-2}{9}, \frac{-4}{3} \)

Solution: LCM of 3, 9, 3 is 9.

\( \frac{-1}{3}=\frac{-1 \times 3}{3 \times 3}=\frac{-3}{9} ; \frac{-2 \times 1}{9 \times 1}=\frac{-2}{9} \) \( \frac{-4}{3}=\frac{-4 \times 3}{3 \times 3}=\frac{-12}{9} \)

-12 < -3 < -2

LCM of 3, 9, 3 is 9

\( \frac{-12}{9}<\frac{-3}{9}<\frac{-2}{9} \) \( \frac{-4}{3}<\frac{-1}{3}<\frac{-2}{9} . \) \( \text { Ascending order is } \frac{-4}{3}, \frac{-1}{3}, \frac{-2}{9} \)

HBSE Class 7 Maths Chapter 8/9 Guide Rational Numbers

3) \( \frac{-3}{7}, \frac{-3}{2}, \frac{-3}{4} \)

Solution: LCM of 7, 2, 4 is 28.

\( \begin{aligned}
& \frac{-3}{7}=\frac{-3 \times 4}{7 \times 4}=\frac{-12}{28} \\
& \frac{-3}{2}=\frac{-3 \times 14}{2 \times 14}=\frac{-42}{28}
\end{aligned} \) \( \begin{aligned}
\frac{-3}{4} & =\frac{-3 \times 7}{4 \times 7} \\
& =\frac{-21}{28}
\end{aligned} \)

– 42 < -21 < -12

LCM of 7, 2, 4 is 28

\( \begin{aligned}
& \frac{-42}{28}<\frac{-21}{28}<\frac{-12}{28} \\
& \frac{-3}{2}<\frac{-3}{4}<\frac{-3}{7}
\end{aligned} \) \( \text { Ascending order is } \frac{-3}{2}, \frac{-3}{4}, \frac{-3}{7} \)

Solutions To Try These

Find (1) \( \frac{-13}{7}+\frac{6}{7} \)

Solution: \( \frac{-13}{7}+\frac{6}{7}=\frac{-13+6}{7}=\frac{-7}{7}=-1 \)

2) \( \frac{19}{5}+\left(\frac{-7}{5}\right) \)

Solution: \( \frac{19}{5}+\frac{-7}{5}=\frac{19+(-7)}{5}=\frac{19-7}{5}=\frac{12}{5} \)

Solutions To Try These

Find:

(1) \( \frac{-3}{7}+\frac{2}{3} \)

Solution: LCM of 7 and 3 is 21.

\( \begin{aligned}
& \frac{-3}{7}=\frac{-3 \times 3}{7 \times 3}=\frac{-9}{21} \\
& \text { and } \frac{2}{3}=\frac{2 \times 7}{3 \times 7}=\frac{14}{21}
\end{aligned} \) \( \frac{-3}{7}+\frac{2}{3}=\frac{-9}{21}+\frac{14}{21}=\frac{-9+14}{21}=\frac{5}{21} \)

2) \( \frac{-5}{6}+\frac{-3}{11} \)

Solution: LCM of 6 and 11 is 66

\( \frac{-5}{6}=\frac{-5 \times 11}{6 \times 11}=\frac{-55}{66} \text { and } \) \( \begin{aligned}
&\frac{-3}{11}=\frac{-3 \times 6}{11 \times 6}=\frac{-18}{66}\\
&\frac{-5}{6}+\frac{(-3)}{11}=\frac{-55}{66}+\frac{(-18)}{66}=\frac{-55-18}{66}=\frac{-73}{66}
\end{aligned} \)

Solutions To Try These

What will be the additive inverse of \( \frac{-3}{9}, \frac{-9}{11}, \frac{5}{7} ? \)

Solution:

\( \text { Additive inverse of } \frac{-3}{9} \text { is } \frac{3}{9} \) \( \text { Additive inverse of } \frac{-9}{11} \text { is } \frac{9}{11} \) \( \text { Additive inverse of } \frac{5}{7} \text { is } \frac{-5}{7} \)

Solutions To Try These

Find 1) \( \frac{7}{9}-\frac{2}{5} \)

Solution:

\( \frac{7}{9}+\frac{(-2)}{5}=\frac{35 \div(-18)}{45}=\frac{35-18}{45}=\frac{17}{45} \)

2) \( 2 \frac{1}{5}-\frac{(-1)}{3} \)

Solution:

\( \begin{aligned}
&2 \frac{1}{5}-\frac{(-1)}{3}=\frac{11}{5}+\text { Additive inverse of } \frac{-1}{3}\\
&\frac{11}{5}+\frac{1}{3}=\frac{33 \div 5}{15}=\frac{38}{15}=2 \frac{8}{15}
\end{aligned} \)

Solutions To Try These

What will be

1) \( \frac{-3}{5} \times 7 ? \)

Solution: \( \frac{-3}{5} \times 7=\frac{(-3) \times 7}{5}=\frac{-21}{5}=-4 \frac{1}{5} \)

2) \( \frac{-6}{5} \times(-2) ? \)

Solution: \( \frac{-6}{5} \times(-2)=\frac{(-6) \times(-2)}{5}=\frac{12}{5}=2 \frac{2}{5} \)

Important Concepts Rational Numbers Class 7 HBSE

Solutions To Try These

Find:

(1) \( \frac{-3}{4} \times \frac{1}{7} \)

Solution: \( \frac{-3}{4} \times \frac{1}{7}=\frac{(-3) \times 1}{4 \times 7}=\frac{-3}{28} \)

(2) \( \frac{2}{3} \times \frac{-5}{9} \)

Solution: \( \frac{2}{3} \times \frac{-5}{9}=\frac{2 \times(-5)}{3 \times 9}=\frac{-10}{27} \)

Solutions To Try These

What will be the reciprocal of \( \frac{-6}{11} \) and \( \frac{-8}{5} \) ?

Solution:

\( \text { The reciprocal of } \frac{-6}{11} \text { is } \frac{-11}{6} \) \( \text { The reciprocal of } \frac{-8}{5} \text { is } \frac{-5}{8} \)

Solutions To Try These

Find:

(1) \( \frac{2}{3} \times \frac{-7}{8} \)

Solution:

\( \begin{aligned}
& \frac{2 \times(-7)}{3 \times 8}=\frac{-14}{24} \\
& =\frac{-14 \div 2}{24 \div 2}=\frac{-7}{12}
\end{aligned} \)

2) \( \frac{-6}{7} \times \frac{5}{7} \)

Solution: \( \begin{aligned}
&\frac{-6}{7} \times \frac{5}{7}=\frac{(-6) \times 5}{7 \times 7}\\
&=\frac{-30}{49}
\end{aligned} \)

Step-by-Step Solutions for Rational Numbers Class 7 Haryana Board

Haryana Board Class 7 Maths Solutions For Chapter 8  Exercise-8.2

1. Find the sum:

1) \( \frac{5}{4}+\left(\frac{-11}{4}\right) \)

Solution:

\( \begin{aligned}
& \frac{5}{4}+\left(\frac{-11}{4}\right)=\frac{5+(-11)}{4} \\
& =\frac{5-11}{4}=\frac{-6}{4}=\frac{-6 \div 2}{4 \div 2}=\frac{-3}{2}
\end{aligned} \)

2) \( \frac{5}{3}+\frac{3}{5} \)

\(\)

Solution: \( \frac{5}{3}+\frac{3}{5} \)

LCM of 3 and 5 is 15.

\( \frac{5}{3}=\frac{5 \times 5}{3 \times 5}=\frac{25}{15} \) \( \frac{3}{5}=\frac{3 \times 3}{5 \times 3}=\frac{9}{15} \) \( \frac{5}{3}+\frac{3}{5}=\frac{25}{15}+\frac{9}{15}=\frac{25+9}{15} \) \( =\frac{34}{15}=2 \frac{4}{15} \)

3) \( \frac{-9}{10}+\frac{22}{15} \)

Solution: \( \frac{-9}{10}+\frac{22}{15} \)

LCM of 10 and 15 is 30.

\( \frac{-9}{10}=\frac{-9 \times 3}{10 \times 3}=\frac{-27}{30} \) \( \frac{22}{15}=\frac{22 \times 2}{15 \times 2}=\frac{44}{30} \) \( \frac{-9}{10}+\frac{22}{5}=\frac{-27}{30}+\frac{44}{30} \) \( =\frac{-27+44}{30}=\frac{17}{30} \)

4) \( \frac{-3}{-11}+\frac{5}{9} \)

Solution:

LCM of 11 and 9 is 99.

\( \frac{-3}{-11}=\frac{3}{11}=\frac{3 \times 9}{11 \times 9}=\frac{27}{99} \) \( \frac{5}{9}=\frac{5 \times 11}{9 \times 11}=\frac{55}{99} \) \( \frac{-3}{-11}+\frac{5}{9}=\frac{27}{99}+\frac{55}{99}=\frac{27+55}{99}=\frac{82}{99} \)

5) \( \frac{-8}{19}+\frac{(-2)}{57} \)

Solution:

LCM of 19 and 57 is 57

\( \frac{-8}{19}=\frac{-8 \times 3}{19 \times 3}=\frac{-24}{57} \) \( \begin{aligned}
& \text { and } \frac{-2}{57}=\frac{-2 \times 1}{57 \times 1}=\frac{-2}{57} \\
& \frac{-8}{19}+\frac{(-2)}{57}=\frac{-24}{57}+\frac{(-2)}{57} \\
& =\frac{-24-2}{57}=\frac{-26}{57}
\end{aligned} \)

6) \( \frac{-2}{3}+0 \)

Solution: \( \frac{-2}{3}+\frac{0}{3}=\frac{-2+0}{3}=\frac{-2}{3} \)

7) \( -2 \frac{1}{3}+4 \frac{3}{5} \)

Solution: \( -2 \frac{1}{3}+4 \frac{3}{5}=\frac{-7}{3}+\frac{23}{5} \)

LCM of 3 and 5 is 15.

\( \begin{aligned}
& \frac{-7}{3}=\frac{-7 \times 5}{3 \times 5}=\frac{-35}{15} \\
& \frac{23}{5}=\frac{23 \times 3}{5 \times 3}=\frac{69}{15} \\
& \frac{-7}{3}+\frac{23}{5}=\frac{-35}{15}+\frac{69}{15}
\end{aligned} \) \( =\frac{-35+69}{15}=\frac{34}{15}=2 \frac{4}{15} \)

2. Find:

1) \( \frac{7}{24}-\frac{17}{36} \)

Solution: \( \frac{7}{24}-\frac{17}{36}=\frac{7}{24}=\left(\frac{-17}{36}\right) \)

LCM of 24 and 36 is 72.

\( \frac{7}{24}=\frac{7 \times 3}{24 \times 3}=\frac{21}{72} \text { and } \frac{17}{36}=\frac{17 \times 2}{36 \times 2}=\frac{34}{72} \) \( \begin{aligned}
\frac{7}{24}+\frac{(-17)}{36} & =\frac{21}{72}+\frac{(-34)}{72} \\
& =\frac{21+(-34)}{72}=\frac{-13}{72}
\end{aligned} \)

(2) \( \frac{5}{63}-\left(\frac{-6}{21}\right) \)

Solution:

\( \frac{5}{63}-\left(\frac{-6}{21}\right)=\frac{5}{63}+\frac{6}{21} \)

LCM of 63 and 21 is 63.

\( \begin{aligned}
\frac{5}{63} & =\frac{5 \times 1}{63 \times 1}=\frac{5}{63} \\
\frac{6}{21} & =\frac{6 \times 3}{21 \times 3}=\frac{18}{63} \\
& =\frac{5+18}{63}=\frac{23}{63}
\end{aligned} \)

Practice Problems Rational Numbers Class 7 Haryana Board

3) \( \frac{-6}{13}-\frac{(-7)}{15} \)

Solution: \( \frac{-6}{13}-\frac{(-7)}{15}=\frac{-6}{13}+\frac{7}{15} \)

LCM of 13 and 15 is 195.

\( \begin{aligned}
& \frac{-6}{13}=\frac{-6 \times 15}{13 \times 15}=\frac{-90}{195} \\
& \frac{7}{15}=\frac{7 \times 13}{15 \times 13}=\frac{91}{195} \\
& \frac{-6}{13}+\frac{7}{15}=\frac{-90}{195}+\frac{91}{195}
\end{aligned} \) \( =\frac{-90+91}{195}=\frac{1}{195} \)

4) \( \frac{-3}{8}-\frac{7}{11} \)

Solution: \( \frac{-3}{8}-\frac{7}{11}=\frac{-3}{11}+\left(\frac{-7}{11}\right) \)

LCM of 8 and 11 is 88.

\( \begin{aligned}
& \frac{3}{8}=\frac{3 \times 11}{8 \times 11}=\frac{33}{88} \text { and } \frac{7}{11}=\frac{7 \times 8}{11 \times 8}=\frac{56}{88} \\
& \frac{-3}{8}+\left(\frac{-7}{11}\right)=\frac{-33}{88}+\left(\frac{-56}{88}\right)
\end{aligned} \) \( \begin{aligned}
& =\frac{-33+(-56)}{88}=\frac{-89}{88} \\
& =-1 \frac{1}{88}
\end{aligned} \)

5) \( -2 \frac{1}{9}-6 \)

Solution: \( -2 \frac{1}{9}-6=\frac{-19}{9}-6=\frac{-19}{9}+\frac{(-6)}{1} \)

LCM of 9 and 1 is 9.

\( \begin{aligned}
& \frac{19}{9}=\frac{19 \times 1}{9 \times 1}=\frac{19}{9} \text { and } \frac{6}{1}=\frac{6 \times 9}{1 \times 9}=\frac{54}{9} \\
& -2 \frac{1}{9}-6=\frac{-19}{9}+\frac{(-6)}{1}
\end{aligned} \) \( \begin{aligned}
& =\frac{-19}{9}+\left(\frac{-54}{9}\right)=\frac{-19+(-54)}{9} \\
& =\frac{-73}{9}=-8 \frac{1}{9}
\end{aligned} \)

3) Find the product

1) \( \frac{9}{2} \times\left(\frac{-7}{4}\right) \)

Solution: \( \frac{9}{2} \times\left(\frac{-7}{4}\right) \)

\( \begin{aligned}
& =\frac{9 \times(-7)}{2 \times 4} \\
& =\frac{-63}{8} \\
& =-7 \frac{7}{8}
\end{aligned} \)

Haryana Board Class 7 Maths Exercise 8.1 Solutions

2) \( \frac{3}{10} \times(-9) \)

Solution: \( \frac{3}{10} \times(-9) \)

\( \begin{aligned}
& =\frac{3 \times(-9)}{10} \\
& =\frac{-27}{10} \\
& =-2 \frac{7}{10}
\end{aligned} \)

3) \( \frac{-6}{5} \times \frac{9}{11} \)

Solution: \( \frac{-6}{5} \times \frac{9}{11} \)

\( \begin{aligned}
& =\frac{-6 \times 9}{5 \times 11} \\
& =\frac{-54}{55}
\end{aligned} \)

4) \( \frac{3}{7} \times\left(\frac{-2}{5}\right) \)

Solution: \( \frac{3}{7} \times\left(\frac{-2}{5}\right) \)

\( \begin{aligned}
& =\frac{3 \times(-2)}{7 \times 5} \\
& =\frac{-6}{35}
\end{aligned} \)

5) \( \frac{3}{11} \times \frac{2}{5} \)

Solution: \( \frac{3}{11} \times \frac{2}{5} \)

\( \begin{aligned}
& =\frac{3 \times 2}{11 \times 5} \\
& =\frac{6}{55}
\end{aligned} \)

Key Questions in Rational Numbers for Class 7 HBSE

6) \( \frac{3}{-5} \times \frac{-5}{3} \)

Solution: \( \frac{3}{-5} \times \frac{-5}{3} \)

\( \begin{aligned}
& =\frac{3 \times(-5)}{(-5) \times 3} \\
& =\frac{-15}{-15}=1
\end{aligned} \)

4. Find the value of:

1) \( (-4) \div \frac{2}{3} \)

Solution: \( (-4) \div \frac{2}{3} \)

\( =\frac{-4}{1} \div \frac{2}{3} \) \( \begin{aligned}
& =\frac{-4}{1} \times \frac{3}{2} \\
& =\frac{(-4) \times 3}{1 \times 2} \\
& =\frac{-12}{2}
\end{aligned} \)

=-6

2) \( \frac{-3}{5} \div 2 \)

Solution:

\( \begin{aligned}
& \frac{-3}{5} \div 2=\frac{-3}{5} \div \frac{2}{1} \\
& =\frac{-3}{5} \times \frac{1}{2} \\
& =\frac{-3 \times 1}{5 \times 2} \\
& =\frac{-3}{10}
\end{aligned} \)

3) \( \frac{-4}{5} \div(-3) \)

Solution:

\( \begin{aligned}
& \frac{-4}{5} \div(-3)=\frac{-4}{5} \div \frac{(-3)}{1} \\
& =\frac{-4}{5} \times \frac{1}{-3}=\frac{-4 \times(-1)}{5 \times 3}=\frac{4}{15}
\end{aligned} \)

4) \( \frac{-1}{8} \div \frac{3}{4} \)

Solution:

\( \begin{aligned}
& \frac{-1}{8} \div \frac{3}{4}=\frac{-1}{8} \times \frac{4}{3} \\
& =\frac{-1 \times 4}{8 \times 3} \\
& =\frac{-4}{24} \\
& =\frac{-4 \div 4}{24 \div 4} \\
& =\frac{-1}{6}
\end{aligned} \)

5) \( \frac{-2}{13} \div \frac{1}{7} \)

Solution:

\( \begin{aligned}
& \frac{-2}{13} \div \frac{1}{7}=\frac{-2}{13} \times \frac{7}{1} \\
& =\frac{-2 \times 7}{13 \times 1} \\
& =\frac{-14}{13} \\
& =-1 \frac{1}{13}
\end{aligned} \)

6) \( \frac{-7}{12} \div\left(\frac{-2}{13}\right) \)

Solution:

\( \begin{aligned}
& \frac{-7}{12} \div\left(\frac{-2}{13}\right) \\
& =\frac{-7}{12} \times\left(\frac{-13}{2}\right) . \\
& =\frac{-7 \times 13}{12 \times(-2)} \\
& =\frac{-91}{-24} \\
& =\frac{91}{24}=3 \frac{19}{24}
\end{aligned} \)

7) \( \frac{3}{13} \div\left(\frac{-4}{65}\right) \)

Solution:

\( \begin{aligned}
& \frac{3}{13} \div\left(\frac{-4}{65}\right) \\
& =\frac{3}{13} \times \frac{65}{-4} \\
& =\frac{3 \times 65}{13 \times(-4)}=\frac{3 \times 5}{-4} \\
& =\frac{-15}{4}=-3 \frac{3}{4}
\end{aligned} \)

Additional Questions

Very Short Answer Questions

1. What is meant by a rational number?

Solution:

A number that can be expressed in the form of \( \frac{p}{q} \) where p and q are integers and q ≠ 0 is called a rational number.

2. How’ to write equivalent rational numbers?

Solution: If the numerator and denominator of a rational number are multiplied or divided by a non- zero integer we get a rational number which is said to be equivalent to the given rational number.

3. How to write rational numbers in the standard form?

Solution:

A rational number is said to be in the standard form if its denominator is a I positive integer and the numerator and denominator have no common factor
other than 1.

4) Reduce \( \frac{-75}{120}\) to the standard form.

Solution:

We have \( \begin{aligned}
& \frac{-75}{120}=\frac{-75+3}{120+3} \\
& =\frac{-25}{40}=\frac{-25+5}{40 \div 5}=\frac{-5}{8}
\end{aligned} \)

5. Compare \( \frac{-3}{5} \text { and } \frac{-1}{3} \)

Solution:

\( \begin{aligned}
& \frac{-3}{5}=\frac{-3 \times 3}{5 \times 3}=\frac{-9}{15} \\
& \frac{-1}{3}=\frac{-1 \times 5}{3 \times 5}=\frac{-5}{15}
\end{aligned} \) \( \begin{aligned}
& \text { we have } \frac{-9}{15}<\frac{-8}{15}<\frac{-7}{15}<\frac{-6}{15}<\frac{-5}{15} \\
& \frac{-3}{5}<\frac{-8}{15}<\frac{-7}{15}<\frac{-6}{15}<\frac{-1}{3}
\end{aligned} \) \( \frac{-3}{5}<\frac{-1}{3} \)

6. \( \text { Add } \frac{-7}{5} \text { and } \frac{-2}{3} \text {. } \)

Solution:

LCM of 5 and 3 to 15

\( \begin{aligned}
& \frac{-7}{5}=\frac{-7 \times 3}{5 \times 3}=\frac{-21}{15} \\
& \frac{-2}{3}=\frac{-2 \times 5}{3 \times 5}=\frac{-10}{15} \\
& \frac{-7}{5}+\frac{(-2)}{3}=\frac{-21}{15}+\frac{(-10)}{15} \\
& =\frac{-21 \cdot 10}{15}=\frac{-31}{15}
\end{aligned} \)

7. \( \text { Find } \frac{5}{7}-\frac{3}{8} \)

Solution: \( \frac{5}{7}-\frac{3}{8}=\frac{40-21}{56}=\frac{19}{56} \)

8. Find (1) \( \frac{-3}{5} \times 2 \)

Solution: \( \frac{-3 \times 2}{5}=\frac{-6}{5} \)

2) \( \frac{4}{9}+\frac{(-5)}{7} \)

Solution: \( \frac{4}{9}+\frac{(-5)}{7}=\frac{4}{9} \times \frac{7}{-5}=\frac{-28}{45} \)

9. Write five rational numbers which are smaller than \( \frac{5}{6} \).

Solution: \( \frac{5}{6}=\frac{50}{60} \)

We know that \( \frac{49}{60}, \frac{48}{60}, \frac{47}{60}, \frac{46}{60}, \frac{45}{60} \)………………… are smaller than \( \frac{50}{60} \).

\( \frac{49}{60}, \frac{48}{60}, \frac{47}{60}, \frac{46}{60}, \frac{45}{60} \)………… are any tive rational numbers smaller
than \( \frac{5}{6} \)

10. What number should \( \frac{-33}{16} \) by to get \( \frac{-11}{4} \)

Solution:

The number \( \frac{-33}{16} \) should be divided by to get \( \frac{-11}{4} \)

\( \begin{aligned}
& =\frac{-33}{16} \div \frac{-11}{4} \\
& =\frac{-33}{16} \times \frac{4}{-11} \\
& =\frac{3}{4}
\end{aligned} \)

Short Answer Questions

11. Subtract :

1) \( \frac{3}{4} \text { from } \frac{1}{3} \)

Solution: \( \frac{1}{3}-\frac{3}{4} \)

\( =\frac{(4 \times 1)-(3 \times 3)}{12}=\frac{4-9}{12}=\frac{-5}{12} \)

2) \( \frac{-32}{13} \text { from } 2 \)

Solution:

\( 2-\left(\frac{-32}{13}\right)=\frac{2}{1}+\frac{32}{13} \) \( =\frac{(13 \times 2)+(1 \times 32)}{13}=\frac{26+32}{13}=\frac{58}{13} \)

3) \( -7 \text { from } \frac{-4}{7} \)

Solution: \( \frac{-4}{7}-(-7)=\frac{-4}{7}+\frac{7}{1} \)

\( =\frac{(1 \times-4)+(7 \times 7)}{7}=\frac{-4+49}{7}=\frac{45}{7} \)

12. What numbers should be added to \( \frac{-5}{8} \) so as to get \( \frac{-3}{2} \) ?

Solution:

Suppose ‘x’ is the rational number to be

added to \( \frac{-5}{8} \text { to get } \frac{-3}{2} \)

Then, \( \frac{-5}{8}+x=\frac{-3}{2} \)

\( \Rightarrow x=\frac{-3}{2}-\left(\frac{-5}{8}\right) \) \( \begin{aligned}
& \Rightarrow x=\frac{-3}{2}+\frac{5}{8} \\
& \Rightarrow x=\frac{(4 \times-3) \times(1 \times 5)}{8} \\
& \Rightarrow x=\frac{-12+5}{8}=\frac{-7}{8}
\end{aligned} \) \( x=\frac{-7}{8} \)

13. The sum of two rational numbers is 8. If one of the numbers is \( \frac{-5}{6} \) then find the other.

Solution: It is given that

Sum of the two numbers = 8 and one of the numbers = \( \frac{-5}{6} \)

Suppose the other rational number is x. Since the sum is 8

\( \begin{aligned}
& \Rightarrow x+\left(\frac{-5}{6}\right)=8 \Rightarrow x=8-\left(\frac{-5}{6}\right) \\
& \Rightarrow x=\frac{8}{1}+\frac{5}{6} \\
& \Rightarrow x=\frac{(6 \times 8)+(1 \times 5)}{6} \\
& \Rightarrow x=\frac{48+5}{6}=\frac{53}{6}
\end{aligned} \)

The other number is \( \frac{53}{6} \)

14. Represent \( \frac{-13}{5} \) on the number line.

Solution:

Represent 13 of 5 on the number line

\( \frac{13}{5}=-2 \frac{3}{5}=-2-\frac{3}{5} \). This lies between – 2 and- 3 on the number line.

Divide the number line between- 2 and – 3 into 5 equal parts.

Mark 3rd part (numerator of rational part) counting from 2.

This is the place of the required rational number \( \frac{-13}{5} \)

15. Express each of the following decimal in the \( \frac{p}{q} \) form

  1. 0.57
  2. 0.176
  3. 1.00001
  4. 25.125

Solution:

1) \( 0.57=\frac{57}{100} \)

2) \( 0.176=\frac{176}{1000}=\frac{176 \div 8}{1000 \div 8}=\frac{22}{125} \)

3) \( 1.00001=\frac{100001}{100000} \)

4) \( \begin{aligned}
25.125 & =\frac{25125}{1000}=\frac{25125 \div 5}{1000 \div 5} \\
& =\frac{5025 \div 5}{200 \div 5}=\frac{1005 \div 5}{40 \div 5}=\frac{201}{8}
\end{aligned} \)

Long Answer Questions

16. Represent these numbers on the number line. (1) \( \frac{9}{7} \) (2) \( \frac{-7}{5} \)

Solution:

Represent these numbers on the number line,

(1) \( \frac{9}{7}=1 \frac{2}{7}=1+\frac{2}{7} \).This lies between 1 and 2 on the number line.

Divide the number line between1 and 2 into 7 equal parts. Mark 2nd part countingfrom1

This is the place of the required rational number \( \frac{9}{7} \) .

(2)

This is the place of the required rational number9 of 7

\( -\frac{7}{5}=-\left(1 \frac{2}{5}\right)=-\left(1+\frac{2}{5}\right)=-1+\left(\frac{-2}{5}\right) \)

This lies between -1 and -2 on the number line.

Divide the number line between -1 and -2 into 5 equal parts.. Mark 2nd part counting from -1.

This is the place of rational number \( \frac{-7}{5} \)

17. Find a rational number between \( \frac{2}{3} \text { and } \frac{3}{4} \)

Solution:

\( \frac{2}{3}=\frac{2 \times 4}{3 \times 4}=\frac{8}{12} \) [Hint: First write the rational numbers with equal denominators]

\( \frac{3}{4}=\frac{3 \times 3}{4 \times 3}=\frac{9}{12} \) (Converting them into rational numbers with same denominators)

Now

\( \frac{8}{12}=\frac{8 \times 5}{12 \times 5}=\frac{40}{60} \text { and } \quad \frac{9}{12}=\frac{9 \times 5}{12 \times 5}=\frac{45}{60} \)

Rational numbers between \( \frac{2}{3} \text { and } \frac{3}{4} \) may be taken as \( \frac{41}{60}, \frac{42}{60}, \frac{43}{60}, \frac{44}{60} \)

We can take any one of these.

(or)

We know that between two rational numbers x and y such that x < y, there is a rational \( \frac{x+y}{2} \)

i.e \( x<\frac{x+y}{2}<y \)

So, a rational number between \( \frac{2}{3} \text { and } \frac{3}{4} \text { is } \)

\( \frac{\frac{2}{3}+\frac{3}{4}}{2}=\frac{\frac{(4 \times 2)+(3 \times 3)}{12}}{2}=\frac{\frac{8+9}{12}}{2}=\frac{17}{12} \times \frac{1}{2}=\frac{17}{24} \)

Thus we have \( \frac{2}{3}<\frac{17}{24}<\frac{3}{4} \).

18. Find ten rational numbers between \( \frac{-3}{4} \text { and } \frac{5}{6} \).

Solution: \( \frac{-3}{4}=\frac{-3 \times 6}{4 \times 6}=\frac{-18}{24} \)

\( \frac{5}{6}=\frac{5 \times 4}{6 \times 4}=\frac{20}{24} \)

[Converting them to rational numbers with the same denominators]

Clearly -17, -16, -15, -14, -13, -12, -11, -10………………0,1,2,3………….are integers between numerators -18 and 20 of these equivalent rational numbers. Thus we have \( \frac{-17}{24}, \frac{-16}{24}, \frac{-15}{24}, \frac{-14}{24}, \frac{-13}{24}, \frac{-12}{24}, \frac{-11}{24}, \frac{-10}{24}, 0, \frac{1}{24} \) ……………………… as rational numbers between

\( \frac{-18}{24}\left(=\frac{-3}{4}\right) \text { and } \frac{20}{24}\left(=\frac{5}{6}\right) \)

We can take any ten of these as required rational numbers.

 

Workbook

Choose the correct answers :

1. Which of these is a negative rational number

  1. 0
  2. \( \frac{5}{7} \)
  3. \(\frac{-5}{7}\)
  4. \(\frac{-5}{-7}\)

Answer: 3

2. The HCF of 45 and 30 is 

  1. 15
  2. 30
  3. 45
  4. 1350

Answer: 1

3. \( \frac{3}{7}+\frac{(-6)}{7}= \)

  1. \( \frac{9}{7} \)
  2. \( \frac{-9}{7} \)
  3. \( \frac{3}{7} \)
  4. \( \frac{-3}{7} \)

Answer: 4

4. Additive inverse of \( \frac{-4}{7} \) is

  1. \( \frac{-7}{4} \)
  2. \( \frac{4}{7} \)
  3. \( \frac{-4}{7} \)
  4. \( \frac{-3}{7} \)

Answer: 2

5. LCM of 3 and 7 is

  1. 10
  2. 21
  3. 4
  4. 7

Answer: 2

6. How is \( \frac{7}{4} \) is expressed as a rational number with denominator 20?

  1. \( \frac{-70}{20} \)
  2. \( \frac{-35}{20} \)
  3. \( \frac{35}{20} \)
  4. B or C

Answer: 2

7. Express \( \frac{1}{4} \) and \( \frac{1}{3} \) with same denominator.

  1. \( \frac{4}{12} \text { and } \frac{3}{12} \)
  2. \( \frac{3}{12} \text { and } \frac{4}{12} \)
  3. \( \frac{4}{7} \text { and } \frac{3}{7} \)
  4. \( \frac{3}{7} \text { and } \frac{3}{7} \)

Answer: 2

8. \( -\frac{28}{84} \) can be expressed as a rational number as……..

  1. \( \frac{4}{7} \)
  2. \( \frac{-4}{12} \)
  3. \( \frac{4}{12} \)
  4. \( \frac{4}{-7} \)

Answer: 2

9. Which of the following is true?

Statement (1):

\( \frac{-9}{15}<\frac{-2}{3}<\frac{-4}{5} \)

Statement (2): \( \frac{-4}{5}<\frac{-2}{3}<\frac{-9}{15} \)

Statement (3): \( \frac{-2}{3}<\frac{-9}{15}<\frac{-4}{5} \)

  1. only (1)
  2. only (2)
  3. only (3)
  4. both (1) and (2)

Answer: 2

10. Which of the following are three rational numbers between -2 and -1?

  1. \( \frac{-1}{2}, \frac{-1}{3}, \frac{-1}{5} \)
  2. \( \frac{-3}{2}, \frac{-7}{4}, \frac{-5}{4} \)
  3. \( \frac{-12}{5}, \frac{-22}{5}, \frac{12}{5} \)
  4. \( \frac{3}{2}, \frac{7}{4}, \frac{5}{4} \)

Answer: 2

11. A rational number between \( \frac{-2}{3} \text { and } \frac{1}{4} \) is…………

  1. \( \frac{5}{12} \)
  2. \( \frac{-5}{12} \)
  3. \( \frac{5}{24} \)
  4. \( \frac{-5}{24} \)

Answer: 4

12. If \( \frac{p}{q} \) is the fractional form of 0.36 then p + q =…………..

  1. 15
  2. 17
  3. 19
  4. 21

Answer: 1

13. The denominator of afractionwhich equals to the decimal fraction of 0.125 is………….

  1. 900
  2. 1000
  3. 999
  4. 990

Answer: 3

14. 0.9 + 9.1 =……….

  1. 9.91
  2. 9.19
  3. 10.1
  4. 10.1

Answer: 3

15. The reciprocal of 9 lies in the number system…….

  1. N
  2. W
  3. Z
  4. N and W

Answer: 3

16. The sum of two rational numbers is 8 and one of them is \( \frac{-5}{6} \).Then the second number is……………….

Answer: 1

17. Which of the rational numbers

\( \frac{-11}{28}, \frac{-5}{7}, \frac{-9}{14}, \frac{-29}{42} \) is the greatest?

  1. \( \frac{-11}{28} \)
  2. \( \frac{-5}{7} \)
  3. \( \frac{-9}{14} \)
  4. \( \frac{-29}{42} \)

Answer: 1

18. \( \frac{7}{8}-\frac{2}{3}= \) = ……………..

Answer: 2

19. \( \text { If } \frac{x}{9}=\frac{4}{x} \text { then } x= \)…………..

Answer: 4

20. Which of the following is not a rational number ?

  1. \( \frac{-2}{3} \)
  2. -0.3
  3. π
  4. 0

Answer: 3

21. Rama : \( \frac{5}{3} \) is a rational number and 5 is only a natural number.

Shyama: Both \( \frac{5}{3} \) and 5 are rational numbers.

Which of the statements are true?

  1. Both Rama and Shyama
  2. Only Rama
  3. Only Shyama
  4. Neither Rama nor Shyama

Answer: 3

22. Which of the following is different among the following rationals ?

  1. \( \frac{1}{7} \)
  2. \( \frac{2}{3} \)
  3. \( \frac{27}{8} \)
  4. \( \frac{145}{6} \)

Answer: 3

23. 0.4 + 0.3 + 0.2 =………

  1. 0.432
  2. 0.432
  3. 0.1
  4. 1

Answer: 4

24. \( \frac{2 . \overline{9}}{4 . \overline{9}}=\)……….

  1. \( \frac{1}{2} \)
  2. \( \frac{3}{5} \)
  3. 1
  4. not defined

Answer: 2

25. A bus is moving at an average speed of \( 60 \frac{2}{5} \)km/hr.How much distance it will cover in \( 7 \frac{1}{2} \)

  1. 423 km
  2. 433 km
  3. 443 km
  4. 453 km

Answer: 4

26. The area of a rectangular park whose length is \( 36 \frac{3}{5} \)m and breadth is \( 16 \frac{2}{3} \)m………

  1. 1830 m²
  2. 1220 m²
  3. 610 m²
  4. 305 m²

Answer: 3

27.

  1. 10x = 157.3232………
  2. 1000 x = 15732.3232………
  3. Subtracting we get x = \( x=\frac{15575}{990} \)
  4. Let x = 15.732
    Arrange the steps in order to express 15.732 in \( x=\frac{p}{q} \)
  1. 2, 1, 3, 4
  2. 4, 2, 1, 3
  3. 3, 1, 2, 4
  4. 4, 2, 3, 1

Answer: 2

28. Identify the rational number A marked in the following number line.

  1. \( x=\frac{3}{7} \)
  2. \( x=\frac{4}{6} \)
  3. \( x=\frac{4}{7} \)
  4. \( x=\frac{5}{7} \)

Answer: 3

29. Write the rational numbers for the points labelled with letters P, Q, R, S in order on the number line

Write the rational numbers for the points labelled with letters P, Q, R, S in order on numberline

  1. \( \frac{-3}{2}, \frac{-5}{4}, \frac{-3}{4}, \frac{-1}{4} \)
  2. \( \frac{-1}{4}, \frac{-3}{4}, \frac{-5}{4}, \frac{-3}{2} \)
  3. \( \frac{6}{4}, \frac{5}{4}, \frac{3}{4}, \frac{1}{4} \)
  4. \( \frac{1}{4}, \frac{3}{4}, \frac{5}{4}, \frac{6}{4} \)

Answer: 1

30. Which letter of the number indicates \( \frac{17}{5} \)?

Which letter of the number indicates 17 of 5 in the folowing

  1. A
  2. B
  3. C
  4. D

Answer: 2

Fill in the blanks :

31. All integers and fractions are…………..

Answer: rational numbers

32. Equivalent rational number for \( \frac{-3}{7} \) is………..

Answer: \( \frac{-6}{14} \)

33. The number…………. is neither a positive nor a negative rational number

Answer: zero

34. There are……….. number of rational numbers between any two rational numbers.

Answer: infinite

35. Both the numerator and the denominator of a rational number are positive then it is called a……………..

Answer: positive rational number

36. Match the following:

1. Reduce to standard form \( \frac{-3}{-15} \)    (   ) A) \( \frac{1}{4} \)

2.Which is greater \( \frac{-1}{4}, \frac{1}{4} \)   (   ) B) \( \frac{10}{9} \)

3. The additive inverse of \( \frac{5}{7} \)            (   ) C) \( \frac{-5}{7} \)

4) \( \frac{-2}{9} \times(-5)= \)                             (   ) D) \( \frac{-15}{2} \)

5) \( (-5) \div \frac{2}{3}= \)                                  (   ) E) \( \frac{1}{5} \)

Answer:

1. E 2. A 3. C 4. B 5. D

Haryana Board Class 7 Maths Solutions For Chapter 6 The Triangle and its Properties

Haryana Board Class 7 Maths Solutions For Chapter 6 The Triangle and its Properties

Key Concepts

  • Triangle:
    A triangle is a simple dosed curve made of three line segments.lt has three vertices, three sides and three angles.

Triangle

Here is Δ ABC. It has

    1. Sides: AB’ BC’ CA
    2. Angles:∠BAC, ∠ABC, ∠BCA
    3. Vertices: A, B, C
  • The side opposite to the vertex A is BC.
    The angle opposite to the side AB is ∠BCA.
  • Classification of Triangles:
  1. Based on sides:
      1. A triangle having three unequal sides is called a scalene triangle.
      2. A triangle having two equal sides is called an isosceles triangle.
      3. A triangle having three equal sides is called an equilateral triangle.
    1. Based on angles:
      1. If each angle is less than 90°,’ then. the triangle is called an acute-angled triangle.
      2. If any one of the angle is a right angle, then the triangle is called a right-angled triangle.
      3. If any one angle is greater than 90°, then the triangle is called an obtuse angled triangle.
  • Parts of a Triangle: In a ΔABC,
    1. The points A, B, C are called “Vertices”
    2. The line segments AB, BC, CA are called “Sides”.
    3. ∠BAC, ∠ABC, ∠ACB or briefly∠A, ∠B, ∠C are called “angles” of the triangle. The sides AB, BC, CA and the angles ∠A, ∠B, ∠C are called the “Parts of the triangle ABC” or “elements of  ΔABC. “
  • In A ABC, we have
    1. ∠A is the angle opposite to the side BC
    2. Similarly, ∠B, ∠C are the angles opposite to the sides CA, AB respectively.
    3. A, B, C are the vertices opposite to the sides BC,CA, AB respectively. O The length of the side AB is denoted by AB. i.e., AB is a number.
  • Interior and Exterior of a Triangle:
    1. The Region of the plane enclosed by A ABC is called the interior of A ABC
    2. The total region of the plane not enclosed by A ABC is called the exterior of A ABC.

Triangular Region: The interior of a triangle together with its boundary is known as the “triangular region ” in a plane.

Haryana Board Class 7 Maths Solutions For Chapter 6 The Triangle and its Properties

1. Write the six elements (i.e., the 3 sides and the 3 angles) of ΔABC.

Solution:

Sides: AB, BC, CA

Angles: ∠A, ∠B,∠C

Write the six elements

2. Write the:

1. Side opposite to the’ vertex Q of ΔPQR

Solution:

PR

Side opposite to the’ vertex Q of PQR

HBSE Class 7 Triangle and Its Properties Solutions

2) Angle opposite to the side LM of ΔLMN

Solution:

∠MNL

Angle opposite to the side LM

Haryana Board Class 7 Maths Lines and Angles solutions

3) Vertex opposite to the side RT of ΔRST

Solution: S

Vertex opposite to the side RT of RST

3. Look at the following figure and classify each of the triangles according to its

1) Sides

2) Angles

1)

Look at the following figure

Solution:

Here ΔABC bas two equal sides.

Z It is an isosceles triangle.

Three angles are acute.

It is an acute-angled triangle.

2)

Look at the following figure 2

Solution:

In ΔPQR no two sides are equal.

It is a scalene triangle.

One angle is 90°.

It is a right- angled triangle.

Key Questions in Triangles for Class 7 HBSE

3)

Look at the following figure 3

Solution:

In MNL two sides are equal.

It is an isosceles triangle.

One angle is greater than 90°.

It is an obtuse-angled triangle

4)

Look at the following figure 4

Solution:

In ΔRST, all,three sides are equal.

It is an equilateral triangle.

Ali angles are acute.

It is an acute-angled triangle.

5)

Look at the following figure 5

Solution: AABC has two sides equal.

It is an isosceles triangle.

∠B is greater than 90°.

Itis an obtuse-angled triangle.

Haryana Board Class 7 Maths Triangle Properties solutions

6)

Look at the following figure 6

Solution:

ΔPQR has two sides equal.

It is an isosceles triangle.

∠Q is 90°.

It is a right-angled triangle.

1. How many medians can a triangle have?

Solution: A triangle can have three medians.

2. Does a median lie wholly in the interior of the triangle? (If you think that this is not true draw a figure to show such a case).

Solution: Yes, a median lies wholly in the interior of the triangle.

1. How many altitudes can a triangle have?

Solution: A triangle can have three altitudes.

2. Drawrough sketches of altitudes from A to BC for the following triangles.

 

Drawrough sketches of altitudes from A to BC

Acute – angled
Right-angled
Obtuse- angled

Solution:

Drawrough sketches of altitudes from A to BC for the following triangles

3. Will an altitude always lie in the in terior of a triangle? If you think that this need not be true, draw a rough sketch to show such a case.
Solution:

No, an altitude always does not lie in the interior of a triangle. In the case of an obtuse-angled triangle, the altitude lies in the exterior of the triangle. ABC is an obtuse-angled triangle, zcis obtuse angle. The altitude AL drawn from A on to the produced BC lies in the exterior of the triangle.

draw a rough sketch to show such a case

Practice Problems Triangles Class 7 Haryana Board

4. Can you think of a triangle in which two altitudes of the triangle are two of its sides?

Solution:

No, we cannot think of a triangle in which two altitudes of the triangle are the two sides of triangle in the case of an acute-angled triangle and obtuse-angled triangle. Butin the case of rightangled triangle the two altitudes of the triangle areits two sides forming aright angle.

Angle sum property of a triangle Class 7 HBSE

5. Can the altitude and median be same for a triangle?
Solution:

Yes, the altitude and median can be same for an equilateral triangle

Take several cut-outs of (1) an equilateral triangle (2) an isosceles triangle and (3) a scalene triangle. Find their altitudes and medians. Do you find anything special about them? Discuss it with your friends.

Solution: Students can arrangeit with the help of their teacher. At last they find the altitude and median of a triangle are samein an equilateral triangle.

Haryana Board Class 7 Maths Solutions For Chapter 6 Exercise-6.1

1. In ΔPQR, D is the mid-point of QR.

PM is_____.
PD is_______.
Is QM = MR ?

In ΔPQR,D is the mid -point of QR

PM is an altitude.

PD is median.

No, QM x MR because M is not the mid point of QR.

2. Draw rough sketches for the following:

(a) In ΔABC, BE is a median.

(b) In ΔPQR, PQ and PR are altitudes of the triangle.

(c) In ΔXYZ, YL is an altitude in the exterior of the triangle

Solution:

Draw rough sketches for the following

3. Verify by drawing a diagram if the median and altitude of an isosceles triangle can be same.

Solution:

ABC is an isosceles triangle.
AB = AC
P is the mid-point of BC.
AD is the median.
AL is perpendicular to BC.

ABC is an isosceles triangle

In an isosceles triangle, the median and the altitude are same. AD = AL

1. Exterior angles can be formed for a triangle in many ways. Three of them are shown here.

Exterior angles can be formed for a triagle in many ways

There are three more ways of getting exterior angles. Try to produce those rough sketches.

Solution: The three more ways of getting exterior angles are :

Exterior angles can be formed for a triagle in many ways 1

Important Concepts Triangles Class 7 HBSE

2. Are the exterior angles formed at each vertex of a triangle equal?

Solution:

Yes, the exterior angles formed at each vertex of a triangle are equal.

3. What can you say about the sum of an exterior angle of a triangle and its adjacent interior angle?

Solution:

The sumof an.exterior angle ofa triangle and its adjacent interior angle is 180°.

1. What can you say about each of the interior opposite angles, when the exterior angle is

1) aright angle?
2) an obtuse angle?
3) an acute angle?

Solution:

1) If the exterior angle is a right angle, then each of theinterior opposite angle is acute.

2)If the exterior angleis an obtuse angle,then at least one of the interior opposite angle is acute.

3)If the exterior angleis an acute angle,then each of the interior opposite angle is acute.

Exterior angle theorem for triangles Class 7 HBSE

2. Can the exterior angle of a triangle be a straight angle?

Solution:

No, the exteriorangle ofa triangle cannot be a straight angle

1. An exterior angle of a triangle is a measure of 70° and one of its interior opposite angles is of measure 25°. Find the measure of the otherinterior opposite angle.

Solution:

Let the measure of the other interior angle be x.

By exterior angle property of a triangle,

25° + x = 70°

x= 70°- 25°

x = 45°

the measure of the otherinterior opposite angle

The measure of the other interior opposite angle = 45°.

2. The two interior opposite angles of an exterior angle of a triangle are 60° and 80°. Find the measure of the exterior angle.

Solution:

Given:

The two interior opposite angles of a triangle are 60° and 80°.

Measure of the exterior angle = Sum of its two interior opposite angles.

= 60° +80°=140°

3. Is something wrong in this diagram? Comment.

Is something wrong in this diagram

Solution. We know that

Measure of the exterior angle = Sum of its two interior opposite angles

= 50° + 50° = 100°

Given exterior angle of a triangle = 50°

Such a triangle cannot be drawn.

Haryana Board Class 7 Maths Solutions For Chapter 6 Exercise-6.2 :

1. Find the value of the unknown exterior angle x in the following diagrams:
Solution:

1) Exterior angle = Sum of the interior opposite angles

x = 50° + 70°

x=120°

Find the value of the unknown exterior angle x in figure

2) Exterior angle = Sum of the interior opposite angles

x = 65°+ 45°

x= 110°

x= 110°

3) Exterior angle = Sum of the interior opposite angles

x = 30° + 40°

x = 70°

x = 70°

4) Exterior angle = Sum of the interior opposite angles

x = 60° + 60°

x = 120°

x = 120°

Important questions on triangles Class 7 HBSE Maths

5) Exterior angle = Sum of the interior opposite angles

x = 50° + 50°

x = 100°

x = 100°

6) Exterior angle = Sum of the interior opposite angles

x = 30° + 60°

x = 90°

x = 90°

2. Find the value of the unknown interior angle x in the following figures:

Solution:

1) Sum of interior opposite angles = Exterior angle

x +50° = 115°

x = 115°-50°

x = 65°

x = 65

2) Sum of interior opposite angles = Exterior angle

70° + x =100°

x = 100°-70°

x=30°

x=30°

3) Sum of interior opposite angles = Exterior angle

x + 90° = 125°

x =125° -90°

x = 35°

x = 35

4) Sum of interior opposite angle = Exterior angle

x + 60° = 120°

x = 120°- 60°

x = 60°

x‘= 60°

5) Sum of interior opposite angles = Exterior angle

x + 30° = 80°

x = 80°- 30°

x = 50°

x = 50°

6) Sum of interior opposite angles = Exterior angle

x + 35° = 75°

x =75° -35°

x = 40°

x = 40°

Haryana Board Class 7 Maths Solutions For Chapter 6 Exercise-6.3 :

1. Find the value of the unknown x in the following diagrams:

Solution:

1) By angle sum property,

x + 50°+60° = 180°

x + 110° = 180°

x = 180°-110°

x = 70°

By angle sum property x = 70°

2) By angle sum property,

90° + x + 30° = 180°

120° + x= 180°

x= 180°- 120°’

x=60°

By angle sum property x=60°

3) By angle sum property

30° + 1109 + x = 180°

140°+x =180°

By angle sum property x = 40°

x=180°- 140°

x = 40°

4) By angle sum property

50° + x + x = 180°

50° + 2x = 180°

2x =180°- 50°

2x = 130°

By angle sum property x = 65°

\(x=\frac{130^{\circ}}{2}=65^{\circ}\)

x = 65°.

5) By angle sum property

x+x+x= 180°

3x= 180°

By angle sum property x = 60°

\( x=\frac{180^{\circ}}{3} \)

x = 60°

6) By angle sum property,

90° + x + 2x = 180°

90° + 3x= 180°

3x =180°- 90°

3x = 90°

By angle sum property x=30°

\( x=\frac{90^{\circ}}{3}=30^{\circ} \)

x=30°

2. Find the values of the unknowns x and yin the following diagrams:

Solution:

1) Sum of interior opposite angles = exterior angle

x + 50° = 120°

x = 120°-50°

Find the values of the unknowns x and y in the following diagrams

x = 70°

 

By angle sum property of a triangle,

x + y + 50° = 180°

700 + y +50° = 180°

y + 120° = 180°

y= 180° -120° = 60°

y – 60°

2) y = 80°(vertically opposite angles are equal)

By angle sum property of a triangle,

The values of the unknowns x and y in the following diagrams 2

x + y + 50° = 180°

x + 80° + 50° = 180°

x + 130° = 180°

x = 180° – 130°

x = 50°

3) Exterior angle = Sum of interior opposite angles

x =50° + 60°

x = 110°

The values of the unknowns x and y in the following diagrams 3

By angle sum property of a triangle,

y + 50° + 60° = 180°

y+ 110°=180°

y= 180° -110°

y = 70°

4) x= 60°(vertically opposite angles are equal.)

By angle sum property of a triangle,

x + y + 30° = 180°

60°+ y + 30° = 180°

Write which of the following is true

y + 90° = 180°

y =180°- 90°

y = 90°

5) y = 90° (vertically opposite angles are equal.)

By angle sum property of a triangle,

x + x + y = 180°

2x + 90° = 180°

2x = 180° -90°

The values of the unknowns x and y in the following diagrams 5

2x = 90°

\( x=\frac{90^{\circ}}{2} \)

x=45°

6) x = y (vertically opposite angles are equal)

By angle sum property of a triangle.

x+x+x =180°

The values of the unknowns x and y in the following diagrams 6

3x = 180°

\( x=\frac{180^{\circ}}{3}=60^{\circ} \)

x = 60°; y = 60°

HBSE Class 7 Maths Chapter 6 Guide

1. Two angles of a triangle are 30° and 80°. Find the third angle.

Solution: Let the third angle be x

By angle sum property of a triangle,

x +30° + 80° = 180°

x+ 110° = 180°

x = 180°- 110°

x = 70°

v. The third angle is 70°.

2. One of the angles of a triangle is 80° and the other two angles are equal. Find the measure of each of the.equal angles.

Solution: Let the measure of the equal angles be x.

By angle sum property of a triangle,

x + x + 80° = 180°

2x + 180° = 80°

2x. =100°

\( x=\frac{100^{\circ}}{2}=50^{\circ} \)

The measure of eachof the equal angle is 50°.

3. The three angles of a triangle arein the ratio1:2:1. Find all the angles of the triangle. Classify the triangle in two different ways.

Solution: Let the three angles of a triangle be x, 2x, x,

By angle sum property of a triangle,

x + 2x + x = 180°

4x = 180°

\( x=\frac{180^{\circ}}{4}=45^{\circ} \)

Three angles of the given triangle are 45°, 45° x 2, 45° or 45°, 90°, 45°

Classification:

1) The triangleis aright-angled triangle.

2) The triangle is an isosceles triangle.

1. Can you have a triangle with two right angles?

Solution: No, we can never have a triangle with two right angles because in a triangle the sum of three angles is 180°.

2. Can you have a triangle with two obtuse angles?

Solution: No, we can never have a triangle with two obtuse angles because in a triangle the sum of three angles is 180°.

3. Canyouhave a triangle with two acute angles?

Solution: Yes, we can have a triangle with two acute angles.

4. Can you have a triangle with all the three angles greater than 60°?

Solution: No, we cannot have a triangle with all the three angles greater than 60°.

5. Can you have a triangle with all the three angles equal to 60°?

Solution: Yes, we can have a triangle with all the three angles equal to 60°.

6. Can you have a triangle with all the three angles less than 60°?

Solution: No, we cannot have a triangle with all the three angles less than 60°.

1. Find angle x in each figure:

Solution:

1) Given triangle is an isosceles triangle.

In this triangle, base angles opposite to the equal sides are equal.

x = 40°

Find angle x in each figure 1

2) In an isosceles triangle, base angles opposite to the equal sides are equal. Sum of the three angles of a triangle is 180°.

45° + 45° + x = 180°

90°+ x =180°

Find angle x in each figure 1

x =180°- 90°

x =90°

3) In an isosceles triangle, base angles 45° opposite to the equal sides are equal.

x = 50°

Find angle x in each figure 3

4) Base angles opposite to the equal sides of an isosceles triangle are equal.

The sum of the three angles of a triangle is 180°.

x+ x + 100° = 180°

2x = 180°- 100°

2x = 80°

Find angle x in each figure 4

\( x=\frac{80^{\circ}}{2}=40^{\circ} \)

x = 40°

5) Base angles opposite to the equal sides of an isosceles triangle are equal. The sum of the three angles of a triangle is 180°.

x + x + 90° =180°

2x = 180° – 90°

Find angle x in each figure 5

2x = 90°

\( x=\frac{90^{\circ}}{2}=45^{\circ} \)

x = 45°

6) Base angles opposite to the equal sides of an isosceles triangle are equal.

The sum of the three angles of a triangle is 180°.

x + x + 40° = 180°

 

Find angle x in each figure 6

2x = 180°- 40°

2x = 140°

\( x=\frac{88^0}{2}=70^{\circ} \)

x = 70°

7) Base angles opposite.to equal sides of an isosceles triangle are equal. The angles in a linear pair are supplementry.

x+ 120° = 180°

x = 180°- 120°

= 60°

Find angles x and yin each figure 1

8) Base angles opposite to equal sides of an isosceles, triangle are equal. The external angle of a triangle is equal to the sum ofits interior opposite angles.

x+x= 110°

Find angle x in each figure 8

2x = 110°

\( x=\frac{110^{\circ}}{2}=55^{\circ} \)

x = 55°

9) Base angles opposite to equal sides of an isosceles triangle are equal. If two lines intersect, the vertically opposite angles are equal.

= 30°

Find angle x in each figure 9

HBSE 7th Class Angle Sum Property Explained

2. Find angles x and y in each figure.

Solution:

1) Base angles opposite to equal sides of an isosceles triangle are equal.

y + 120°- 180° (Linear pair)

y= 180° -120°

y = 60°

Sum of interior opposite angles = Exterior angle

Find angles x and yin each figure 1

x + y = 120°

x + 60°= 120°

x = 120°- 60° = 60

2) Base angles opposite to the equal sides of an isosceles triangle are equal.

The sum of the measures of the three angles of a triangle is 180°.

x+x + 90° = 180°

2x =180° -90°

Find angles x and yin each figure 2

2x = 90°

\( x=\frac{90^{\circ}}{2}=45^{\circ} \)

x = 45°

Exterior angle=Sumof interior opposite angles

y = x + 90°

y = 45° + 90°

y = 135°

3) Base angles opposite to the equal sides of an isosceles triangle are equal. Sum of the measures of the three angles of a triangle is 180°

x + x+92° = 180°

2x = 180° – 92°

Find angles x and yin each figure 3

2x = 88°

\( x=\frac{88^{\circ}}{2}=44^{\circ} \)

44° + y = 180° (Linear pair)

y = 180°- 44°

Haryana Board Class 7 Maths Solutions For Chapter 6 Exercise-6.4

1. Is it possible to have a triangle with the following sides?

1) 2cm, 3cm, 5cm

Solution: Given sides are 2cm, 3cm, 5cm

Here 2 + 3 = 5

Sum of the lengths of two sides = Length of the third side

This is impossible.

2) 3cm, 6cm, 7cm

Solution:

Given sides of a triangle are 3cm, 6cm, and 7cm

3 + 6 > 7; 6 + 7 > 3;7 + 3 > 6

Sum of the lengths of any two sides is greater than the length of the third side.

It is possible to form a triangle.

3) 6cm, 3cm, 2cm

Solution:

Given sides of a triangle are 6cm, 3cm,2cm

6 + 3 = 9 > 2; 3 + 2 = 5 |> 6; 2 + 6 = 8 > 3

It is not possible to form a triangle

Sample Problems Triangles Haryana Board Class 7

2. Take any point O in the interior of a triangle PQR. Is

(1) OP + OQ > PQ ?
(2) OQ + OR > QR ?
(3) OR + OP > RP ?

OP + OQ greater thhan PQ

Solution:

1) Yes, OP + OQ > PQ because sum of the lengths of any two sides of ΔPOQ is always greater than the third side.

2) Yes, OQ + OR > QR because sum of the lengths of any two sides of ΔROQ is always greater than the third side.

3) Yes, OR + OP > RP because sum of the lengths of any two sides of ΔROP is always greater than the third side.

OR + OP greater than RP

Pythagoras theorem examples Class 7 HBSE

3. AM is a median of a triangle ABC. Is AB + BC + CA > 2 AM ? (Consider the sides of triangles AABM and AAMC)

Solution:

In AABM

AB + BM > AM

Sum of the lengths of any two sides of a triangle is always greater than the third side.

Is AB + BC + CA less than 2 AM

In ΔAMC

CA + CM > AM………(2)

Sum of the lengths of any two sides of a triangle is always greater than the third side.

(1) + (2)

=> (AB + BM) + (CA + CM) > AM + AM

=> AB + (BM + CM) + CA > 2AM

AB + BC + CA > 2 AM

Pythagoras Theorem Class 7 Haryana Board

4. ABCD is a quadrilateral.

Is AB + BC + CD + DA > AC + BD?

Solution: In ΔABC

AB +BC>AC………..(1)

Sum of the lengths of any two sides of a triangle is greater than the length of the third side.

Is AB + BC + CD + DA greater than AC + BD

In AACD

CD + DA > AC……….(2)

Sum of the lengths of any two sides of a triangle is greater than the length of the third side.

(1) + (2)

=> AB + BC + CD + DA > AC + AC

AB + BC + CD + DA > 2AC………(3)

In AABD

AB + DA > BD ———(4)

Sum of the lengths of any two sides of a triangle is greater than the length of the third side.

In ABCD

BC + CD > BD——(5)

Sum of the lengths of any two sides of a triangle is greater than the length of the third side.

(4) + (5)

=> AB + DA + BC + CD > BD + BD

AB + BC + CD + DA > 2BD ——(6)

(3) + (6)

=> 2 (AB +BC + CD +DA) > 2AC + 2BD

=> 2 (AB + BC + CD + DA) > 2 (AC + BD)

AB + BC + CD + DA> AC + BD

5. ABCD is quadrilateral. Is AB + BC + CD + DA < 2 (AC + BD) ?

Solution: In quadrilateral ABCD, diagonals AC and BD intersect at O.

Is AB + BC +CD +DA greater than (AC+BD)

In AOAB

OA + OB > AB

Sum of the lengths of any two sides of a triangle is. greater than the length of the third side.

In AOBC

OB + OOBC

Sum of the lengths of any two sides of a triangle is greater than the length of the third side.

In ΔOCD

OC + OD > CD———-(3)

Sum of the lengths of any two sides of a triangle is greater than the length of the third side.

In ΔOAD

OA+OD>AD———(4)

Sum of the lengths of any two sides of a triangle is greater than the length of the third side.

(1) + (2) (3) + (4)

= (OA + OB) + (OB +OC) + (OC + OD) + (OA + OD) > AB + BC + CD + DA

=>2(OA+OB + OC + OD)> AB + BC + CD + DA

=> 2((OA + OQ + (OB + OD)) > AB + BC+ CD + DA

=> 2(AC + BD) > AB + BC + CD + DA

AB + BC + CD + DA < 2 (AC + BD)

6. The lengths of two sides of a triangle are 12 cm and 15 cm. Between what two measures should the length of the third side fall ?

Solution: Let the length of the third side be x cm.

Sum of the lengths of any two sides of a triangle is greater than the length of the third side.

Given two sides are 12 cm and 15 cm

12 + 15 > x

27 >x

x < 27

x + 12 > 15

x>15-12

x > 3

x 4 + 15 > 12

x > 12- 15

x > -3

length cannot be negative. The length of the third side should be any where between 3 cm and 27 cm

1. Is the sum of any two angles of a triangle is always greater than the third angle?

Solution: No, the sum of any two angles of a triangle is not always greater than the third angle

Find the unknown length x in the following figures:

1.

Find the unknown length x in the following figure

Solution:

By Pythagoras property,

x²- 3² + 4²

X² = 9 + 16

x² = 25

x = √25 => x =5 cm

2.

Find the unknown length x in the following figure 2

Solution:

By Pythagoras property,

x²= 6² + 8²

x²= 36 + 64

x² = 100

x = √100 => x = 10 cm

3)

Find the unknown length x in the following figure 3

Solution:

By Pythagoras property

x² = 15² + 8²

x² = 225 + 64

x² = 289

x = √289 => x = 17 cm

4.

Find the unknown length x in the following figure 4

Solution:

By Pythagoras property,

x²= 24² + 7²

x² = 576 + 49

x²= 625

x = √625

x = √25 x 25 = 25 cm

5.

Find the unknown length x in the following figure 5

Solution:

ABC is an isosceles triangle.

AB = AC = 37 cm

D is the mid-point of BC

BD = DC

ABD is a right – angled triangle.

By Pythagoras property,

AB² = AD² + BD²

37² = 12² + BD²

∴ BD² =37²-12²

= 1369 -144 = 1225

BD = √1225 = 35 cm

x = BD + DC = 35 + 35 = 70 cm

6)

Find the unknown length x in the following figure 6

Solution:

By Pythagoras property,

x² = 12² + 5²

x² = 144 + 25

x = √169

x=√169 =13 cm

Haryana Board Class 7 Maths Solutions For Chapter 6 Exercise-6.5 :

1. PQRis a triangle, right-angled at P. If PQ = 10 cm and PR = 24 cm find QR.

Solution:

ΔPQR is a right-angled triangle.

∠P = 90°; PQ = 10 cm; PR = 24 cm.

By Pythagoras property

QR² = PQ² + PR²

= (10)² + (24)²

= 100 + 576

find QR

QR² =676

QR = √676 = √26 x 26 = 26 cm

2. ABC is a triangle, right-angled at C.I f AB = 25 cm and AC = 7 cm, find BC.

Solution:

ABC is a right-angled triangle. ∠C = 90°; AB = 25 cm; AC = 7 cm.

find BC

By Pythagoras property,

AB² = AC² + BC²

BC² = AB²- AC²

= (25)²-(7)²

= 625 – 49

BC² = 576

BC= √576

=> BC = √24×24 => BC = 24 cm

3. A 15 m long ladder reached a window 12 m high from the ground on placing it against a wall at a distance a. Find the distance of the foot of the ladder from the wall.

Find the distance of the foot of the ladder from the wall

Solution:

Let AC be the ladder.

C is the foot of the ladder.

AC= length of the ladder = 15 m

A be the top of the ladder at 12 m high from the ground.

ABC is a right-angled triangle.

By Pythagoras property,

AC² = AB² + BC²

BC² = AC²- AB²

a² = 15²- 12²

a²= 225-144=a² = 81

=> a = 81 => a = 79×9

a = 9

Distance of the foot of the ladder from the wall is 9 m.

4. Which of the following can be the sides of a right triangle?

1) 2.5 cm, 6.5 cm, 6 cm
2) 2 cm, 2 cm, 5 cm
3) 1.5 cm, 2 cm, 2.5 cm,

In the case of right-angled triangles, identify the right angles.
Solution:

1) 2.5 cm, 6.5 cm. 6 cm

2.5 cm, 6.5 cm, 6 cm.,

(2.5)² + 6² = 6.25 + 36 = 42.25 = (6.5)²

Pythagoras property is satisfied.

The triangle with given sides is right- angled triangle.

The longest side with length 6.5 cm, is the hypotenuse and angle opposite to this side is the right angle.

2) 2 cm, 2 cm, 5 cm

2²+ 2² = 4 + 4 = 8

5² = 25

2² + 2²≠ 52

The squares of two smaller sides is not equal to the square of the third side. Pythagoras property is not satisfied.

The triangle with given sides is not a right – angled triangle.

3) 1.5 cm, 2 cm, 2.5 cm,

(1.5)² + (2)² = 2.25 + 4 = 6.25 = (2.5)²

Pythagoras property is satisfied.

The triangle with given sidesis a right-angled triangle.

The longest side with length 2.5 cm is the hypotenuse. The angle opposite to this side is a right angle.

5. A tree is broken at a height of 5m from the ground and its top touches the ground at a distance of 12 m from the base of the tree. Find the original height of the tree.

Solution: Let the original height of the tree be BA and tree broken at C.

BC = 5 cm,

AC = CD (Given)

BD = 12 m

AABC is a right-angled triangle; [B = 90°

By Pythagoras property,

CD² = BC² + BD²

= 5²2 + 12²

= 25 + 144

CD²=169

Find the original height of the tree

CD = √169 = √13X13

CD = 13 m

AB = BC + CA

= BC + CD

= 5 + 13 = 18 m

The original height of the tree =18m

6. Angles Q andR of a ΔPQR are 25° and 65°. Write which of the following is true:

(1) PQ² + QR² = RP²
(2) PQ²+ RP²= QR²
(3) RP² + QR² = PQ²

Write which of the following is true

Solution: In APQR,

∠Q = 25°; ∠R = 65° . .

By angle sum property of a triangle,

∠P + ∠Q + ∠R = 180°

∠P + 25° + 65° = 180° .

∠P + 90° = 180°

∴ ∠P= 180° – 90° = 90°

The side opposite to right angle is the hypotenuse.

QR is the hypotenuse.

QR² = PQ² + PR²(By Pythagoras property).

(2) is true.

7. Find the perimeter of the rectangle whose length is 40 cm and a diagonal is 41 cm.

Solution: ABCD is a rectangle.

AB = 40 cm

Diagonal BD = 41 cm

AABD is a right-angled triangle.

By Pythagoras property,

BD² = AB²+ AD²

AD² = BD²- AB²

= (41)²-(40)²

Find the perimeter of the rectangle

AD²=1681-1600 = 81

AD = √81 => AD = 9 cm

Perimeter of the rectangle ABCD

= 2 (AB+ AD)

= 2 (40 + 9) = 2 X 49 = 98 cm

Perimeter of the rectangle = 98 cm

8. The diagonals of a rhombus measure 16 cm and 30 cm. Find its perimeter.

Solution:

ABCD is a rhombus.

Diagonals AC and BD intersect at O.

AC = 16 cm

Find its perimeter

OA = OC = 1/2×16=8cm

BD = 30 cm

OB = OD =1/2 x 30 = 15 cm

AAOB is a right-angle triangle.

∠O=90°

By Pythagoras property,

AB² = OA²+ OB²

AB² = 8² + 15²

= 64 + 225 = 289

AB = √289 = 17 cm

In a rhombus all sides are of equal length.

AB = BC = CD = DA = 17 cm

Perimeter of the rhombus ABCD

= 4 x side

= 4 x17 = 68 cm

Perimeter of the rhombus = 68 cm

1. Which is the longest side in the triangle PQR; right-angled at P?

Which is the longest side in the triangle PQR

Solution:

In any right-angled triangle the hypotenuse is the longest side.

ΔPQR is a right-angled triangle and right-angled at P.

The side opposite to angle P is QR.

QR is the hypotenuse.

QR is the longest side.

2. Which is the longest side in the triangle ABC, right-angled at B?

Solution:

ABC is a right-angled triangle

∠B = 90°

Which is the longest side in the triangle ABC

The side opposite to angle B is AC.

AC is the hypotenuse.

AC is the longest side.

3. Which is the longest side of a right triangle ?

Solution:

The hypotenuse is the longest side of a right triangle.

4. ‘The diagonal of a rectangle produce by itself the same area as produced by its length and breadth’-This is Baudhayan Theorem. Compare it with the Pythagoras property.

Compare it with the Pythagoras property

Solution:

ABCD is a rectangle.

BD is the diagonal.

According to the question,

Area produced by the diagonal =

Area producedby the length + Area produced by the breadth

BD² = AB² + AD²

Which is nothing but Pythagoras property.

Haryana Board Class 7 Maths Solutions For Chapter 6 Very Short Answer Questions

1. Write the types of triangles according to sides.
Solution:

(1) Scalene triangle
(2) Isosceles triangle and
(3) Equilateral triangle.

2. Write the types of triangles based on angles.
Solution:

(1) Acute – angled triangle
(2) Obtuseangled triangle and
(3) Right – angled triangle.

3. State ‘Angle -sumproperty of a triangle.

Solution:

The total measure of the three angles of a triangle is 180°.

4. What is meant by altitude of a triangle?

Solution:

The perpendicular line segment from a vertex of a triangle to its opposite side is called an altitude of the triangle.

5. What is meant by ‘hypotenuse’ of a -triangle?
Solution:

In a right – angled’ triangle the side opposite to the right angle is called the hypotenuse.

6. State ‘Pythagoras property’

Solution: In a right- angled triangle the square on the hypotenuse is equal to the sum of the squares on its legs.

7. Write the features of (1) Equilateral triangle and (2) Isosceles triangle.

Solution: (1) A triangle is said to be equilateral, if each one ofits sideshas the same length.

Each angle has measure 60°.

(2) A triangle is said to be isosceles,if at least any two of its sides are of same length. Base angles opposite to the equal sides are equal.

8. Classify the following angles into acute, obtuse and right angles: 20°, 50°, 102°, 47°, 125°, 65°, 36°, 90°, 95° and 110°.

Solution:

Acute angles: 20°, 50°, 47°, 65° and 36°.

Right angle: 90°.

Obtuse angles:102°,125°, 95° and110°.

9. Sum of two interior angles of a triangle is 105°. Find the third angle.

Solution:

Sum of two interior angles of a triangle = 105°

Let the third angle be = x

Sum of three interior angles ofa triangle =180°

=> 105° + x = 180°

=> x = 180° -105°

x = 75°

The third angle = 75°.

10. In ΔPQR, if∠P=65° and ∠Q=50°, then find ∠R.

Solution: In ΔPQR, if P = 65° and Q = 50°.

The sum of three interior angles of a triangle = 180°.

P +Q + R = 180°.

=> 65° + 50° + R = 180°

115° + R= 180°

R= 180° – 115°

R= 65°

Haryana Board Class 7 Maths Solutions For Chapter 6 Short Answer Questions

11. Classify the following triangles based on the length of their sides.

Classify the following triangles

Solution:

Classify the following triangles based on the length of their sides

12. Classify the following triangles based on the measure of angles.

(1)

Classify the following triangles based on the measure of angles 1

(2)

Classify the following triangles based on the measure of angles 2

(3)

Classify the following triangles based on the measure of angles 3

Solution:

Classify the following triangles based on the measure of angles table

13. If the three angles of a triangular signboard are 2x, (x- 10)° and (x + 30)° respectively. Then find it’s angles.

Solution: The three angles of a triangular sign board are 2x°, (x- 10)° and (x + 30)° respectively.

The sum of three interior angles of a triangle =180°

2x + x-10 + x + 30 = 180°

=>4x° + 20° = 180°

=>4x° =180° -20°

=> 4x° = 160°

x = \( \frac{160}{4} \)

=>x =40°

First angle = 2x° = 2 X 40° = 80°

Second angle = (x-10)° =40°-10° = 30°

Third angle = x + 30° = 40° + 30° = 70°

14. If one angle of a triangleis 80°, find the other two angles which are equal.

Solution:

If one angle of a triangle = 80°

Given that other two angles are equal.

Let the equal angle be x

The sum of three interior angles of a triangle = 180°

=> 80° + x°+ x° = 180°

=> 80° + 2x° = 180°

=> 2x°=180° – 80°

=> 2x°= 100°

x° = \( \frac{100}{2} \)

=> x = 50°

Other two equal angles are 50° and 50°

15. The angles of a triangle arein the ratio 2: 4: 3, then find the angles.

Solution:

The ratio of angles of a triangle=2:4:3

The sum ofratio = 2 + 4 + 3 = 9

Sum of three interior angles of a triangle =180°

The value of first angle

= \( \frac{2}{9} \) x 180 = 2 x 20° = 40°

The value of second angle

= \( \frac{4}{9} \) x 180 = 4 x 20° = 80°

The value of third angle

= \( \frac{3}{9} \) x 180 = 3 x 20° = 60°

16. What are the measurements of angles of an equilateral triangle?

Solution:

Let ΔABCis an equilateral triangle, then

AB=BC = CA

We know that the angles which are opposite to equal sides are equal.

So ∠A =∠B = ∠G

Let the equal angle = ∠A=∠B=∠C=x°

The sum of three interior angles of a triangle = 180°

∠A + ∠B + ∠C = 180°

=>x° + x° + x° = 180°

=>3x = 180°

x = \( \frac{180}{3} \)

What are the measurements of angles of an equilateral trigle

x = 60°

The measurement of each angle of an equilateral triangle is 60°.

17. Which of the following angles form a triangle?

Solution: “If the sum of three interior angles of a triangle is equal to 180°”, then the three angles form a triangle.

1) 60°, 70°, 80°

Solution: Sum of three angles

= 60° +70° + 80° = 210° = 180°

It cannot form a triangle.

2) 65°, 45°, 70°

Solution: Sum of three angles

= 65° + 45° + 70° = 180°

It can form a triangle.

3) 40°, 50°, 60°

Solution: Sum of three angles

= 40° + 50° + 60° = 150° = 180°

It cannot form a triangle.

4) 60°, 30°, 90°

Solution: Sum of three angles

= 60° + 30° + 90° = 180°

It can form a triangle.

5) 38°, 102°, 40°

Solution:

Sum of three angles

= 38° + 102° + 40° = 180°

It can form a triangle.

6) 100°, 30°, 45°

Solution: Sum of three angles

= 100° + 30° + 45° = 175° = 180°

It cannot form a.triangle.

18. Find the missing angles in each of the following triangles.

1)

Find the missing anglesin each of the triangle

Solution:

The sum of three interior angles of a, triangle = 180°

∠K + ∠V + ∠S = 180°

=> 60° + 70° + ∠S = 180°

=> 130° + ∠S= 180°

=>∠S= 180°- 130°

=> ∠S= 50°

2)

Find the missing anglesin each of the triangle 2

Solution:

The sum of three interior angles of a triangle = 180°

∠B + ∠U+ ∠N = 180°

=> 105° + 55° + ∠N = 180°

160°+ ∠N= 180°

∠N= 180° – 160°

∠N= 20°

3)

Find the missing anglesin each of the triangle 3

Solution:

The sum of three interior angles of a triangle = 180°

∠A + ∠T + ∠P = 180°

=> 90° + 38° +∠P = 180°

=> 128° +∠P= 180°

=> ∠P= 180° – 128°

=> ∠P = 52°

19. Find the value of ‘x’in the given figure.

Find the value of 'x' in the given figure

Solution:

From the given figure ∠CAH = ∠TAE

[Vertically opposite angles]

From ΔACH

=> ∠CAH + 60° + 80° = 180°

[ The sum of three interior angles
= 180°]

=> ∠CAH +140° = 180°

=> ∠CAH = 180° -140°

=> ∠CAH = 40°

∠TAE = ZCAH = 40°

From ΔAET

∠TAE +∠AET +∠ATE = 180° ‘

40° + 70° + x = 180°

=> 110° + x° = 180°

=>x°= 180° -110°

x° = 70°

20. Find the value of ‘x’ in the following figures

1)

Find the value of 'x’ in the following figures 1

Solution:

From the figure, ∠A = 35°, ∠B = x°

Exterior angle = ∠ACD = 70°

Exterior angle at C = ∠A + ∠B

70° = 35° + x°

( The exterior angle of triangleis equal to sum of its interior angles)

=> 35° + x° = 70° • ‘

=> x° = 70°- 35°

x° = 35°

2)

Find the value of 'x’ in the following figures 2

Solution:

From the figure, ∠P = 4x°, ∠Q = 3x°

exterior angle at R = ∠PRS = 119°

exterior angle at R = ∠P + ∠Q

(The exterior angle of triangle is equal to sum of its interior angles)

119° = 4x° + 3x°

=> 119° = 7x°

x = \( \frac{119}{7} \)

= 17° =>x = 17°.

The interior angles 4x = 4 x17 = 68°

3x = 3×17 = 51°.

21. Find the value of V in the following triangles:

1)

Find the value of x in the following figure 1

Solution:

From ΔPEN, PE = 4 cm, PN = 4 cm

PE = PN

So it is isosceles triangle. Angles opposite to equal sides are equal
in isosceles triangle.

∠N = ∠E

x =65°

2)

Find the value of x in the following figure 2

Solution:

From ZABG,ZA = 56°, ZB = 56°

By angle sum property of a triangle

x° + 56° + 56° = 180°

x° + 112° = 180°

x° = 180° – 112°

x = 68°

[The sides opposite to equal angles
are equal.]

=>AG = BG

=>BG = 2 = 4.3 cm

22. Find the value of V and ‘y’ in the adjacent figure

Find the value of x and 'y' in the adjacent figure

Solution:

From the given figure

∠C = ∠ACL =56° (Vertically opposite angles)

Given that AC = LC. so ΔACL is an isosceles triangle. Angles opposite to equal sides are equal in isosceles triangle.

=> ∠A + ∠L = x°

=> ∠A +∠ C + ∠L = 180°

(By Angle sumproperty of a triangle)

=>x° + 56° +x° = 180°

=> 2x° + 56° = 180°

=> 2x° = 180° – 56°

=> 2x° = 124°

x = \( \frac{124}{2} \)

x = 62°

Now ∠PAC +∠CAL =180°

(Linear pair of angles)

=> y + x = 180°

=> y + 62 = 180°

=> y = 180° – 62°

y = 118°

Haryana Board Class 7 Maths Solutions For Chapter 6 Multiple Choice Question and Answers

 

1. If two angles in a triangle are 75°, 55°, what type of triangle is that ?

  1. Obtuse
  2. Acute
  3. Right
  4. Scalene

Answer: 2

2. The ratio in which the centroid of a triangle divides the median is……..

  1. 1:2
  2. 1:3
  3. 2:1
  4. 3:1

Answer: 3

3. From the adjacent figure find the values of x and y.

From the adjacent figure find the values of x and y

  1. x = 65°, y = 60°
  2. x = 55°, y = 60°
  3. x = 60°, y = 55°
  4. x = 60°, y = 65°

Answer: 2

4. The angles of the triangle are in the ratio1:2:3 than the smallest angle is……….

  1. 30°
  2. 60°
  3. 90°
  4. 80°

Answer: 1

5. A triangle can have……altitudes.

  1. 1
  2. 2
  3. 3
  4. 4

Answer: 3

6. The exterior angle of a triangle is 130° and one of its interior opposite angle is 60°,then the other opposite interior angle is

  1. 60°
  2. 80°
  3. 70°
  4. 50°

Answer: 3

7. If in a triangle two angles are equal and the third angle is 120°, what are the equal angles?

  1. 40°, 40°
  2. 30°, 30°
  3. 20°, 20°
  4. 50°, 50°

Answer: 2

8. Find x and y values from the figure.

Find x and y values from the figure

  1. x = 40°, y = 80°
  2. x = 80°, y = 40°
  3. x = 70°, y ~ 60°
  4. x = 80°, y = 70°

Answer: 1

9. An obtuse angled triangle has……. acute angles.

  1. one
  2. two
  3. three
  4. zero

Answer: 2

10. The angle in an equilateral triangle is

  1. 70°
  2. 50°
  3. 60°
  4. 40

Answer: 3

11. In a right-angled isosceles triangle the acute angle is

  1. 30°
  2. 40°
  3. 50°
  4. 45°

Answer: 4

12. An acute angled triangle has acute angles.

  1. 1
  2. 2
  3. 3
  4. 4

Answer: 3

13. A triangle which has maximum two acute anglesis

  1. Obtuse
  2. Right
  3. A and B
  4. None

Answer: 3

14. The following are acute angled triangles

  1. Equilateral
  2. Isosceles
  3. Scalene
  4. Above all

Answer: 4

15. The following is the possible third side if the two sides are 6 cm, 9 cm

  1. 1 cm
  2. 2 cm
  3. 3 cm
  4. 6 cm

Answer: 4

16. Choose the correct matching. ( )
i) AB + BC> ( ) a) BC
ii) BC- CA < ( ) b) CA
iii) AD is altitude, then AD < ( ) c) AB

  1. i-a, ii-b,iii-c
  2. i-b, ii-a, iii-c
  3. i-b, ii-c, iii-a
  4. i-c, ii-b, iii-a

Answer: 3

17. The altitude of the …………………triangle lies outside of the triangle.

  1. Acute angled
  2. Right-angled
  3. Obtuse angled
  4. Scalene

Answer: 3

18. What is ‘x’ here?

What is x here

  1. Median
  2. Radius
  3. Altitude
  4. Angular bisector

Answer: 3

19. In ΔABC if ∠A = 3, ∠B and ∠C = 2 ∠B. find all the three angles of ΔABC.

  1. 90°, 60°,30°
  2. 60°, 60°, 60°
  3. 90°, 45°, 45°
  4. 50°, 40°, 90°

Answer: 1

20. In adjacent figure x °=

In adjacent figure x =

  1. 70°
  2. 30°
  3. 100°
  4. 40°

Answer: 4

21. Which of the following are the possible sides of a triangle ?

  1. 3 cm, 5 cm, 10 cm
  2. 4 cm, 4 cm, 8 cm
  3. 3 cm, 4 cm, 5 cm
  4. None of these

Answer: 3

22. If the three angles of a triangle are in the ratio 1:2:3, then the angles are

  1. 40°, 60°, 80°
  2. 30°, 60°, 90°
  3. 50°, 100°, 150°
  4. 30°, 50°, 100°

Answer: 2

23. In ΔXYZ, ∠X =30°, ∠Y = 45° then find ∠Z

  1. 75°
  2. 15°
  3. 95°
  4. 105°

Answer: 4

24. In the given figure, the values of x + y is

 

  1. 120°
  2. 190°
  3. 110°
  4. 180°

Answer: 2

25. Angles of a triangle are 30°, 110°, x° then x is

  1. 50°
  2. 40°
  3. 60°
  4. 15°

Answer: 2

26. The lengths of two sides of an isosceles triangle are 7 cm, 8 cm then the possible third side is of length

  1. 7 cm
  2. 9 cm
  3. 8 cm
  4. 7 or 8 cm

Answer: 4

27. The exterior angle of an equilateral triangle is

  1. 60°
  2. 120°
  3. 150°
  4. 90°

Answer: 2

28. If the angles of a triangle arein the ratio 3:1:2, then biggest angle is

  1. 60°
  2. 120°
  3. 90°
  4. 30°

Answer: 3

29. The two angles of a triangle are complementary thenit is …..triangle.

  1. Acute angled
  2. Obtuse angled
  3. Right angled
  4. Equilateral

Answer: 3

30. Find x in the figure

Find x in the figure

  1. 40°
  2. 60°
  3. 50°
  4. 70°

Answer: 4

31. Find Z in the figure

Find Z in the figure

  1. 70°
  2. 60°
  3. 50°
  4. 40°

Answer: 1

32. In a ΔPQR,if ∠P = 100° and ∠Q = ∠R then ∠P + ∠R =

  1. 100°
  2. 180°
  3. 140°
  4. 40°

Answer: 3

33. Choose the correct matching.

i) 60°, 60°, 60° ( ) a) Obtuse angled triangle
ii) 45°, 45°, 90° ( ) b) Isosceles triangle
iii) 100?, 40°, 40°( ) c) Right angled triangle
iv) 90°,30°,60° ( ) d) Equilateral triangle
v) 50°, 50°, 80° ( ) e) Right angled isosceles triangle

  1. i – a,ii-b,iii – c,iv – d, v – e
  2. i – d,ii – e,iii – a, iv – c, v -b
  3. i- c,ii- d,iii – e,iv – a, v -b
  4. i – e,ii – d,iii – c, iv -b, v – a

Answer: 2

34. Least number of possible acute angles in a triangle is …

  1. 1
  2. 2
  3. 3
  4. 0

Answer: 2

35. Which type of triangle is formed by BC = 7.2 cm, AC = 6 cm and ∠C = 120°?

  1. An acute angled triangle
  2. An obtuse angled triangle
  3. A right angled triangle
  4. An isosceles triangle

Answer: 2

36. Which triangle is formed by AB = 3 cm, BC = 4 cm and AC = 8 cm ?

  1. A scalene triangle
  2. An isosceles triangle
  3. An equilateral triangle
  4. No triangle is formed

Answer: 4

37. P: An isosceles triangle is right-angled. Q: ∠A = ∠B = 45° and ∠C = 90° Which of the following statements is true?

  1. P is true and Q is not the correct explanation of P.
  2. P is false.
  3. Q is true and P is the correct explanation of Q.
  4. P is true and Q is the correct explanation of P.

Answer: 4

38. Which of the following statements is not true?

  1. A triangle can have three 60° angles.
  2. A triangle can have a right angle.
  3. A triangle can have two right angles.
  4. A triangle can have all three angles equal.

Answer: 3

39. Which of the following angles are not the angles of a triangle ?

  1. 45°, 65°, 70°
  2. 45°, 55°, 65°
  3. 60°, 60°, 60°
  4. 30°, 60°, 90°

Answer: 2

40. Sum of interior angles in a triangle is equal to

  1. Two right angles
  2. Two straight angles
  3. Right angle

Answer: 1

41. Sum of two acute angles of a right angled triangle is

  1. 90°
  2. 30°
  3. 60°
  4. 180°

Answer: 1

42. In ΔABC, which of the following is false ?

  1. AB-BC< AC,
  2. BC + CA>AB ‘
  3. AB-BC=AC
  4. None

Answer: 3

43. A triangle can have………….obtuse angle.

  1. 0
  2. 1
  3. 2
  4. 3

Answer: 2

44. The relation between x and y in the given figure expressed with ‘y’ as subject is

The relation between x and y in the given figure expressed with 'y' as subject is

  1. 2y = 180 + x
  2. \(y=\frac{1}{2}(180+x)\)
  3. 2y = 180- x
  4. x = 180 + 2y

Answer: 3

45. Following lengths of the sides of a triangle are given.In which case it is not possible to construct a triangle? (in cms)

  1. 3,4,5
  2. 6,6,6
  3. 4,4,8
  4. 3,5,7

Answer: 3

46. The sum of interior angles in a pentagon is

  1. 270°
  2. 360°
  3. 540°
  4. 480°

Answer: 3

47. The opposite interior angles are in the ratio1: 4, then ∠A, ∠B = ?

The opposite interior angles are in the ratio1 4, then A, B =

  1. 26°, 104°
  2. 104°, 26°
  3. 75°, 105°
  4. 50°, 80°

Answer: 1

48. In the adjacent figure ∠A +∠B + ∠C + ∠D + ∠E =

In the adjacent figure A + B + C + D + E

  1. 90°
  2. 360°
  3. 270°
  4. 540°

Answer: 2

49. Find the values of x and y

Find the values of x and y

  1. 135°, 80°
  2. 80°, 135°
  3. 70°, 125°
  4. 125°, 70°

Answer: 1

50. The following is the representation of line segment.

  1. AB
  2. AB
  3. AB
  4. AB

Answer: 3

51. Name the triangle with length 7cm, 8 cm, 9cm.

  1. Equilateral
  2. Isosceles
  3. Scalene
  4. Right-angled triangle

Answer: 3

52. The measure of right angle is

  1. 90°
  2. 100°
  3. 180°
  4. 80°

Answer: 1

53. Each angle in an equilateral triangle is

  1. 30°
  2. 45°
  3. 80°
  4. 60°

Answer: 4

54. An exterior angle of a triangle is of measure 115° and one of its interior opposite angle is 50°. Then the measure of the other interior angle is

  1. 165°
  2. 65°
  3. 155°
  4. 45°

Answer: 2

55. Two angles of a triangle are 50°, 60° then the third angle is ( )

  1. 10°
  2. 55°
  3. 70°
  4. 110°

Answer: 3

Fill in the blanks:

56. In any right-angled triangle,…………is the longest side.

Answer: hypotenuse

57. The total measure of the three angles of a triangle is……………..

Answer: 180°

58…………is a simple closed figure made of three line segments.

Answer: Triangle

59. If the Pythagoras property holds, the triangle must be……..

Answer: right-angled

60. A ABC is right angled at C. If AC = 5 cm and BC = 12 cm then the length of AB =

Answer: 13 cm

61. Match the following:

1. 7 cm, 7 cm, 7 cm     (  ) A) Scalene triangle

2. 4 cm, 5 cm, 6 cm     (  ) B) Obtuse- angled triangle

3. 6 cm, 6 cm, 8 cm     (  ) C) Right – angled triangle

4. 30°, 60°, 90°             (  ) D) Equilateral triangle

5. 30°; 50°, 100°           (  ) E) Isosceles triangle

Answer:

1. D . 2. A 3. E 4. C 5. B

Haryana Board Class 7 Maths Solutions For Chapter 5 Lines and Angles

Haryana Board Class 7 Maths Solutions For Chapter 5 Lines and Angles

Key Concepts

1. Line segment:

A line segment has two end points.

Line segment

A line segment.PQ is generally denoted by PQ.

2. Line (or) Straight line:

If the end points of a line segment are extended in either direction endlessly, we get a line or a straight line.

A line AB is denoted by AB

Line (or) Straight line

3. Ray:

A line which has only one end point is called a ray.

Ray

OP is a ray.It is denoted by OP.

Haryana Board Class 7 Maths Lines and Angles solutions

4. Examples of line segments in our daily life:

1. An edge of a box
2. A tube light
3. An edge.of a post card

Angle:

An angle is formed when lines or line segments meet at a common point. This common point is called vertex of angle and the line segments are called its, arms or sides.

The common point ‘O’ is the vertex.

Lines OA, OB are arms of the given angle AOB. Angle AOB is denoted by ∠AOB.

Angle

Types of Angles:

Acute angle:

An angle which is greater than 0° but less than’90° is called an acute angle.

Example: 15°, 20°, 30°, 35°, 50°, 65°, 75°, 80°, 85°

Acute angle

HBSE Class 7 Lines and Angles Solutions

Right angle:

An angle whose measure is. 90° is called a right angle.

Right angle

Types of angles and their properties Class 7 HBSE

Obtuse angle:

An angle which is greater than 90° but less than 180° is called an obtuse angle.

Obtuse angle

Example: 95°, 110°, 120°, 150°, 165°, 175° etc. are all obtuse angles.

Haryana Board Class 7 Maths Solutions For Chapter 5 Solutions

1. List ten figures around you and identify 170° the acute, obtuse and right angles found in them.

Solution:

Students can do this with the help of their teacher.

1. Can two acute angles be complement to each other?

Solution: Yes, two acute angles can be complement to each other only if their sumis equal to90°.

2. Can two obtuse angles be complement to each other?

Solution:

No, two obtuse angles cannot be complement to each other because their sum of the angles will be greater than 90°.

3. Can two right angles be complement to each other?

Solution:

No, two right angles cannot be complement to each other.

Key Questions in Lines and Angles for Class 7 HBSE

1. Which pairs of following angles are complementary?

1)70° + 20° = 90°

Solution:

70° + 20° = 90°

These angles are complementary

2)

75° + 25° = 100°

Solution:

75° + 25° = 100°

These angles are not complementary

3)

48° + 52° = 100°

Solution:

48° + 52° = 100°

90°. These angles are not complementary.

4)

35° + 55° = 90°

Solution:

35° + 55° = 90°

These angles are complementary.

Parallel lines and transversal angles Class 7 HBSE

1 What is the measure of the complement of each of the following angles ?

1) 45°

Solution:

Complement of the angle 45° is 90°- 45°= 45°

2) 65°

Solution:

Complement of the angle 65° is 90°- 65° = 25°

3) 41°

Solution:

Complement of the angle 41° is 90°- 41°= 49°

4) 54°

Solution:

Complement of the angle 54° is 90°- 54° = 36°

Practice Problems Lines and Angles Class 7 Haryana Board

3. The difference inanglesthe measuresis 12°. Find of two the measures of the angles

Solution:

Let the angle be x°

Its complement is 90″ – xc

Their difference * 12°

x°- (90°- x) =12°

x° – 90° + x° = 12°

2x° =12°+ 90°

2x° – 102°

\( \text { or } x^{\circ}=\frac{102}{2}=51^{\circ} \)

First angle = 51°

Second angle = 90° – 51° =39°

The required angles are 51°, 39°.

1. Can two obtuse angles be supplementary?

Solution:

So, two obtuse angles cannot be supplementary’ because the sum of two obtuse angles would be more than 180°:

2. Can two acute angles be supplementary?

Solution: No, two acute angles carrot be supplementary because their sum would be less than 180°.

3. Can two right angles be supplementary?

Solution:

Yes, two right angles can be supplementary. Because measure of each right angle is 90°.

90° + 90°= 180°= Supplementary

1. Find the pair of supplementary angles:

1). 

pair of supplementary angles

Solution:

Sum of these angles = 110°+ 50° = 160°

These are not supplementary angles

2) 

Sum of these angles

Solution:

Sum of these angles = 105° + 65 ° = 170°

These are not supplementary angles.

3)

Sum of these angles 3

Solution:

Sum of these angles = 50° + 130° = 180°

These are supplementary angles

4)

Sum of these angles 4

Solution:

Sum of these angles = 45° + 45° = 90°

These are not supplementary angles

HBSE 7th Class Complementary and Supplementary Angles

2. What will be the measure of the supplement of each one of the following angles?

1) 100°

Solution:

We know that the sum of two supplementary angles is 180°.

Supplement of 100° is 180° – 100° = 80°

2) 90°

Solution:

Supplement of 90° is 180° – 90° = 90°

3) 55°

Solution:

Supplement of 55° is 180° – 55° = 125°

4) 125°

Solution: Supplement of 125° is 180° – 25° = 55°

Important questions for Lines and Angles Class 7 HBSE

3. Among two supplementary angles the measure of the larger angle is 44° more than the measure of the smaller. Find their measures.

Solution:

Let the smaller angle be x.

Its supplement bigger angle is (x + 44)°

We know that the sum of two supplementary angles is 180°.

x + (x + 44)° =180°.

=> 2x + 44° = 180°

=> 2x = 180° – 44°

=>2x = 136°

\( x=\frac{136^{\circ}}{2}=68^{\circ} \)

The smaller angle is ’68°.

Its supplement larger angle is 68° + 44° = 112°

Exercise-5.1

1. Find the complement of each of the following angles:

Solution:

Sum of two complementary angles is 90°

Sum of two complementary angles is 90°.

1) Complement of the angle 20° is 90°-20° =70°

2) Complement of the angle 63° is 90°-63°= 27°

3) Complement of the angle 57° is 90°-57° =33°

2. Find the supplement of each of the following angles:

Sum of two supplementary angles is 180

Solution:

Sum of two supplementary angles is 180°.

1) Supplement of the angle 105° is 180°- 105° = 75°

2) Supplement of the angle 87° is 180°-87° = 93°

3) Supplement of the angle 154° is 180°- 154° = 26°

Sample Problems Lines and Angles Haryana Board Class 7

3. Identify which of the following pairs of angles are complementary and which are supplementary.

1) 65°, 115°

Solution:

Sum of the angles = 65° + 115°= 180°.

The given angles are supplementary

2) 63°, 27°

Solution:

Sum of the angles = 63° + 27° = 90°

The given angles are complementary.

3) 112°, 68°

Solution: Sum of the angles = 112° + 68° = 180°

The given angles are supplementary.

4) 130°, 50°

Solution: Sum of the angles = 130° + 50° = 180°

The given angles are supplementary.

5) 45°, 45°

Solution: Sum of the angles =45° + 45° = 90°

The given angles are complementary.

6) 80°, 10°

Solution: Sum of the angles = 80°+ 10° = 90°

The given angles are complementary.

4. Find the angle which is equal to its complement.

Solution:

Let the angle be x°

Its complementis 90°- x°

Given that the angle is equal to its complement.

i.e. x° = 90°- x°

=> x° + x° = 90°

=> 2x° = 90°

\(\Rightarrow x^{\circ}=\frac{90^{\circ}}{2} \Rightarrow x=45^{\circ} \)

The required angle is 45°

5. Find the angle which is equal to its supplement.

Solution:

Let the angle be x°

Its supplement is 180°- x°

Given that the angle is equal to its supplement.

i.e. x° = 180°- x°

x° + x° = 180°

2x°=180°

\( x=\frac{180^{\circ}}{2} \)

x° = 90°

The required angle is 90°

6. In the given figure, ∠1 and ∠2 are supplementary angles. If ∠1 is decreased, what changes should take place in ∠2 so that both the angles still remain Supplementary.

In the given figure, 1 and 2 are suplementry angles

Solution:

If A is decreased then 1 should be increased so that both the angles still remain supplementary.

7. Can two angles be supplementary if both of them are:

1) acute? (2) obtuse? (3) Right?

Solution:

Two angles are supplementary if their sum is 180°.

1) No;if two angles are acute (0° < x < 90°) then their sum cannot be equal to 180°.

2) No;iftwo angles are obtuse (90°<x<180°) then their sum cannot be equal to 180°.

3) Yes; if two angles are at right angles (= 90°) then their sum is equal to 180°.

Types of Angles Class 7 Haryana Board

8. An angle is greater than 45°. Is its complementary angle greater than 45° or equal to 45° or less than 45°.

Solution:

The sum of two complementary angles is 90°. Its complementary angle is less than 45°.

9. Fill in the blanks:

1) If two angles are complementary, then the sum of their measures is…………… (90°)

2)  If two angles are supplementary, then the sum of their measures is…………….. (180°)

3) If two adjacent angles are supplementary, they form a…….. (linear pair)

10. In the adjoining figure, name the following pairs of angles.

In the adjoining figure, name the pair angles

1) Obtuse vertically opposite angles.

Solution: ∠AOD and ∠BOC

2) Adjacent complementary angles.

Solution: ∠AOB and ∠AOE

3) Equal supplementary angles.

Solution: ∠BOE and ∠EOD

4) Unequal supplementary angles.

Solution: ∠EOA and ∠EOC

5) Adjacent angles that do not form a linear pair

Solution:

∠AOB and ∠AOE; ∠AOE and ∠EOD; ∠EOD and ∠COD

1. Find examples from your surroundings where lines intersect at right angles.

Solution: Comers of the walls, sides of a box.

2. Find the measures of the angles made by the intersecting lines at the vertices of an equilateral triangle.

Find the measures of the angles

Solution:

l, m, and n are three lines forming an equilateral triangle ABC. Angles formed are A, B,C We see that each angle is 60°.

3. Draw any rectangle and find the measures of angles at the four vertices made by the intersecting lines

Solution:

Draw any rectangle

PQRS is a rectangle formed by four lines k, l, m, n.

Angles at vertices P, Q, R, S are at right angles.

4. If two lines intersect, do they always intersect at right angles?

Solution:

No, they may not always intersect at right angles.

1. Suppose two lines are given. How many transversals can you draw for these lines?

Solution:

We can draw an infinite number of transversals

2. If a line is a transversal to three lines,how many points of intersections are there?

Solution: When a line is a transversal to three lines then there are three points of intersection.

3. Try to identify a few transversals in your surroundings.

Solution:

1. A road crossing two or more roads.

2. A railway line crossing several other lines.

Name the pairs of angles in each figure :

1.

Name the pairs of angles

Solution: ∠1,∠2 are pair, of corresponding angles

2.

3,4 are pair of alternate interior angles

Solution: ∠3.∠4 are pair of alternate interior angles

3.

5,6 pair of inferior angles

Solution: ∠5, ∠6 pair of inferior angles on the same side of the transversal

4.

7,8 are pair of angls

Solution: ∠7, ∠8 are pair of corresponding angles.

5.

9,10 are pair of alternate interior

Solution: ∠9, ∠10 are pair of alternate interior 1 angles.

6.

11,12 form a linear pair

Solution: ∠11, ∠12 form a linear pair.

1) Lines l \\m; t is a transversal, x =?

Given l m t is a transversal

Solution:

Given: l \\m; t is a transversal.

∠x = 60° (Alternate interior angles)

2) Lines a \\ b; c is a transversal, ∠y= ?

Lines l and m; t is a transvrsal

Solution:

Given: a \\ b; c is a transversal.

∠y = 55° (Alternate interior angles)

Complementary and supplementary angles Class 7

3) l1,l2 be two lines; t is a transversal. Is ∠1 = ∠2?

t is a transversal

Solution:

No. ∠1 = ∠2

4) Lines l\\m; t is a transversal. ∠z = ?

Solution:

Lines l m t is a transversal

∠z + 60° = 180°

∠z = 180°- 60°

= 120°

5) Lines l \\m; t is a transversal, ∠x = ?

Solution:

Lines l m; t is a transversal, x =

Given: l \\ m; t is transversal.

∠x = 120° (pairs of corresponding angles)

6) Lines l\\m,p\\q; Find a, b, c, d

lines lm pq

Solution:

Given: p\\q and l is transversal.

∠a + 60° = 180°

=> ∠a = 180°- 60° = 120°

l \\m and q is transversal.

∠a = ∠1 (pair of corresponding angles)

∠1 = 120°

∠d = ∠1 =120°(vertical opposite angles)

∠1 + ∠c = 180° (linear pair)

120° + ∠c = 180°

= ∠c =180°- 120°= 60°

∠b = ∠c

∠b = 60°

a = 120°; b = 60°; c = 60°; d = 120°

1) Is l \\m Why?

Solution:

Is l m Why

Here alternate angles are equal

l \\m

2) Is l \\m? Why?

 

Is l m Why

Solution: Corresponding angles are equal.

l\\m

3) If l\\m, what is ∠x ?

If l m, what is x

Solution:

The sum of the interior angles on the same side of the transversal are supplementary.

x + 70° = 180°

x = 180° -70°= 110°

Haryana Board Class 7 Maths Solutions For Chapter 5 Exercise 5.2 :

1. State the property that is used in each of the following statements.

State the property that is used

1). If a\\b, then ∠1 = ∠5.

Solution:

If a\\b, then ∠1 = ∠5

If a transversal intersects two parallel lines then the corresponding angles are equal

2) If ∠4 =∠6 then a\\b.

Solution:

∠4,∠6 are alternate interior angles.

∠4 =∠6 then a\\b.

If two parallel lines are cut by a transversal the alternate interior anglesare equal.

3) If ∠4 +∠5 = 180° then a \\ b.

Solution:

If two parallel lines are cut by a transversal then each pair ofinterior angles on the same side of the transversal are supplementary.

2. In the adjoining figure, identify

In the adjoining figure, identify

1) the pairs of corresponding angles.

Solution: ∠l, ∠5; ∠2,∠6; ∠3, ∠7; ∠4, ∠8

2) the pairs of alternate interior angles.

Solution: ∠2,∠8;∠3,∠5

3) the pairs of interior angles on the same side of the transversal.

Solution: ∠2,∠5;∠3,∠8

4) the vertically opposite angles.

Solution: ∠1,∠3;∠2,∠4;∠5,∠7;∠6,∠8

3. In the adjoining figure p \\ q. Find the unknown angles

Solution:

 

Find the the known angles

p\\q and l is transversal.

e + 125°= 180° (linear pair)

e = 180°- 125° = 55°

∠f = e (vertically opposite angles)

∠f = 55°

∠a = e (corresponding angles)

∠a = 55°

∠c = a (vertically opposite angles)

∠c = 55°

∠d = 125°(corresponding angles)

∠b =d (vertically opposite angles)

∠b = 125°

a = 55°;b =125°;c= 550 d =125°;e = 55°; f = 55°

HBSE Class 7 Maths Chapter 5 Guide

4. Find the value of x in each of the following figures if l\\m.

1)

Find the value of x

Solution: x+110° = 180° (Linear pair)

x=180°-110°

=> ∠x = 70°

∠x = 70° (alternate angles)

2)

Lines l and m; t is a transvrsal

Solution: l\\m and ‘a’ is a transversal.

x = 100°( corresponding angles)

5. In the given figure the arms of two angles are parallel.

If ∠ABC= 70° then find

1) ∠DGC

2) ∠DEF

 

In the given figure the arms of two angles are parallel

Solution:

1) AB\\DE and BC is a transversal

∠DGC=∠ABC (corresponding angles)

Given ∠ABC =70°

∠DGC = 70°

2) BC\\EF and DE is a transversal

∠DEF=DGC (corresponding angles)

∠DGC = 70°

∠DEF = 70°

6. In the given figures below, decide whether l is parallel to m

In the given figures below

Solution:

1) l is not parallel to m Because

126° + 44°= 170° = 180°

L is not parallel to m

Solution: 2) l is not parallel to m

∠l =75° (vertically opposite angles)

75° + 75° = 150° = 180°

Solution:

l is not parallel to m

3) l is parallel to m

∠l +57° = 180°

∠l =180° -57° = 123°

Alternate angles are equal.

Solution:

l is parallel to m

4) l is not parallel to m

∠l = ∠2 (vertically opposite angles.)

72° + 98° =170° = 180°

Haryana Board Class 7 Maths Solutions For Chapter 5 Very Short Answer Questions

1. How many end points are there for a ‘line segment’?

Solution: A line segment has two end points.

2. Define a ‘transversal’.

Solution: A line that intersect two or more lines at distinct points is called a transversal.

3. Write the angles made by a transversal.

Solution: Interior angles, exterior angles, corresponding angles, alternate interior angles, alternate exterior angles.

4. State (1) Complementary angles (2) Supplementary angles.

Solution:

(1) If the sum of the measures of two angles is 90° the angles are called complementary angles.

(2) If the sum of the measures of two angles is 180° the angles are called supplementary angles.

5. Write the properties of two parallel lines are cut by a transversal.

Solution:

1) Each pair of corresponding angles are equal in measure.

2) Each pair of alternate interior angles are equal.

3) Each pair of interior angles on the same side of the transversal are supplementary.

6. In the figure, ∠AOB and ∠COD have common vertex O. But ∠AOB, ∠COD are not adjacent angles. Why ? Give reason.

In the figure, AOB and COD

Solution:

∠AOB and ∠COD have common vertex ‘O’, but ∠AOB, ∠COD are not adjacent angles, because there is no common arm.

7. Draw a pair of adjacent angles which are supplementary to each other.

Draw a pair of adjacent angles

8. Write any three examples for vertically opposite angles in your surroundings.

Solution:

Examples for vertically opposite angles in our surroundings :

  1. Four angles madein the scissors
  2. The point where two roads intersect each other
  3. Rail road crossing signs
  4. Comers of the room

9. When a transversal intersects two lines and a pair of alternate exterior angles are equal, what can you say about the two lines?

Solution: ‘When a transversal intersects two lines and a pair of alternate exterior angles are equal’ “Then the two lines are parallel”.

10. Write any two examples for linear pair of angles in your surroundings.

Solution:

Examples for linear pair of angles:

1) An electrical pole on the road.

2) A pen stand. ,

Haryana Board Class 7 Maths Solutions For Chapter 5 Short Answer Questions

11. Is it possible for the following pair of angles to form a linear pair? If yes, draw them. If not, give reason

Sum of two angles

1) 120°, 60°

Solution: Sum of two angles

= 120°+60° = 180°

= linear pair

So 120° and 60° angles are possible to form a ‘linear pair1

2) 98°, 102°

Solution: Sum of two angles= 98°+102° =200°= 180°. So 98°, 102° angles are not possible to form a linear pair.

Because the linear pair has sum of two angles is 180° but here sum of two angles is 200°.

12. Name three pairs of vertically opposite angles in the figure.If ∠AOB =45°, then find ∠DOE.

find DOE

Solution: Three pairs of vertically opposite angles are:

1) ∠AOB, ∠DOE

2) ∠AOF, ∠COD

3) ∠BOC, ∠EOF

If ∠AOB = 45° then∠DOE =∠AOB = 45° (Vertically opposite angles)

∠DOE = 45°

13. In the given figure, the lines l and m intersect at point P. Observe the figure and find the values of x, y and z.

In the given figure, the linesl and m

Solution: In the given figure, the linesl and m intersect at point P.

y = 20° [Vertically opposite angles]

20°+x = 180° [ Linear pair on line ‘m’]

x°= 180-20°

x° =160°

z = x° = 160° [Vertically opposite angles]

z = 160°

= 160°, y = 20°, z = 160°

14. In the figure, p\\q and t is a transversal. Observe the angles formed

Observe the angles formed in the figure

1) If ∠1 = 100°, then what is ∠5 ? 

Solution: If ∠1 = 100° then ∠5 = ∠1 = ∠100° [Corresponding angles]

2) If ∠8 = 80°, then what is ∠4?

Solution: If ∠8 = 80° then ∠4 = ∠8 = ∠80° [Corresponding angles]

3) If ∠3 = 145°, then what is ∠7?

Solution: If ∠3 = 145° then ∠7 = Z3 = ∠145° [Corresponding angles]

4) If ∠6=30°, then whatis ∠2?

Solution: If ∠6 = 30° then ∠2 = ∠6 = ∠30° [Corresponding angles]

15. From the figure, state which property that is used in each of the following.

In the given figure, two lines p q and r is transversal

1) If ∠3 = ∠5 then p||q.

Solution: Alternate interior angle.

2) If ∠3 + ∠6 = 180° then p || q.

Solution: Co-interior angles are supplementary.

3) If ∠3 = ∠8 then p || q.

Solution: Corresponding angles

4) If p\\q then ∠1 = ∠8.

Solution: Alternate exterior angles

16. In the given figure, two lines p\\q and r is a transversal.If ∠3 =135°, then find the remaining angles

In the given figure, two lines p q and r is transversal

Solution:

In the given figure, p \\ q and Y is a transversal.

If ∠3 =135°, then ∠1 =∠3 = 135° ( Vertically opposite angles)

∠2+∠3 = 180°

∠2 + 135° = 180° ( Linear pair of angles)

∠2 = 180° -∠3 = 180° -135° = 45°

∠2- 45°

∠4=∠2 = 45° (Vertically opposite angles)

=> ∠5 =∠1 = 135° (Corresponding angles)

=>∠6 =∠2 = 45° (7 Corresponding angles)

=>∠7=∠4 = 45°( Corresponding angles) 80°

=>∠8 =∠3 = 135° (Corresponding angles)

∠1 =∠3 = ∠5 =∠8 =135°

∠2 = ∠4 = ∠6=∠7 = 45°

Important Concepts Lines and Angles Class 7 HBSE

17. Find the complementary, supplementary, and conjugate angle of 36°.

Solution:

The complementary angle of 36° is = 90° -36° = 54°

The supplementary angle of 36° is = 180° -36° = 144°

The conjugate angle of 36° is = 360° -36° = 324°

18. In the given figure the lines and m intersect at O. Find x

In the given figure the linesl and m intersect

Solution:

The lines l and m intersect at ‘O’

x + 40° = 120° ( Vertically opposite angles)

=>x = 120°-40° = 80°

x = 80°

Haryana Board Class 7 Maths Solutions For Chapter 5 Long Answer Questions

19. In the given figure p, q, r, and s are parallel lines and t is a transversal. Find x, y, and z.

In the given figure p, q, r and s are

Solution: In the given figure p, q,r, and s are parallel lines, and t is a transversal.

x + 80° = 180° (Co-interior angles)

x = 180°-80°

x = 100°

In the given figurep, q,r and s are parallel

x° + ∠ABq = 180° ( Linear pair)

=>100°+ ∠ABq = 180°

=> ∠ABq = 180°- 100°= 80°

y = ∠ABq = 80° ( Corresponding angles)

y = 80°

z = y = 80°(Alternate exterior angles)

z = 80°

20. In the given figure AB \\ CD and E is a point in between them. Find x + y + z.

(Hint: Draw a parallel line to AB through E)

In the given figure AB CD and E is a point

Solution: In the given figure AB \\ CD and E is a point in between them.

In the given figure AB CD and E

Draw a parallel line

EF to AB through E.

Consider AB \\ EF and

AE as a transversal.

∠BAE + ∠AEF = 180° ( Co-interior angles)

x° + ∠AEF = 180°

∠AEF = 180°- x -> 1

Consider CD || EF and CE as transversal.

∠DCE + ∠CEF = 180° ( Co-interior angles)

z + ∠CEF = 180°

∠CEF = 180°- z -> 2

1 + 2 =>

=> ∠AEF + ∠CEF = 180° – x + 180°- Z

∠AEC = 360° – x- z

( ∠AEC = ∠AEF + ∠CEF)

y = 360° – x- z

x + y + z =360°

Haryana Board Class 7 Maths Solutions For Chapter 5 Multiple Choice Question and Answers

Choose the correct answers:

1. A ray has …… endpoints.

  1. one
  2. two
  3. three
  4. four

Answer: 1

2. Find the value of x in the given figure

Find the value of xin the given figure

  1. 57°
  2. 58°
  3. 56°
  4. 55°

Answer: 1

3. Which of the following are the units of an angle?

  1. meters
  2. seconds
  3. gnpns
  4. degrees

Answer: 4

4. How many rays can we draw from a given point?

  1. 1
  2. 2
  3. 10
  4. Infinitely many

Answer: 4

5. Find the angle which is complement of itself.

  1. 90°
  2. 30°
  3. 50°
  4. 45°

Answer: 4

6. Supplementary angle of 130° =………

  1. 40°
  2. 60°
  3. 50°
  4. 90°

Answer: 3

7. One of the acute angle in right angle triangle is 30° then the other angle is

  1. 30°
  2. 90°
  3. 70°
  4. 60°

Answer: 4

8. From the adjacent figure value of x =……..

From the adjacent figure value of x is

  1. 60°
  2. 120°
  3. 90°
  4. 180°

Answer: 1

9. What is x here?

What is x here

  1. 28°
  2. 38°
  3. 48°
  4. 58°

Answer: 3

10. In the figure MN || PQ. ∠MNE = 120°, ∠EPQ = 100°, what is x here ?

In the figure MN PQ. and what is x here

  1. 20°
  2. 30°
  3. 60°
  4. 40°

Answer: 4

11. The angle which cannot be formed by scissors

  1. Acute
  2. Right
  3. Straight
  4. Obtuse

Answer: 3

12. Angles between legs of a folding chair is an example of

  1. vertically opposite angles
  2. adjacent angles
  3. corresponding angles
  4. alternate interior angles

Answer: 1

13. In the adjacent figure J \\ BC. Find x, y.

In the adjacent figure find x,y

  1. 60°, 45°
  2. 45°, 60°
  3. 75°, 45°
  4. 60°, 75°

Answer: 1

14. If two lines intersect each other, then the number of common points they have

  1. 1
  2. 2
  3. 4
  4. Infinity

Answer: 1

15. The supplementary angle of 70° is

  1. 20°
  2. 110°
  3. 290°
  4. 70°

Answer: 2

16. Find the complementary angle of 55°.

  1. 35°
  2. 45°
  3. 25°
  4. 15°

Answer: 1

17. Supplementary angle of 130° is ……….

  1. 130°
  2. 50°
  3. 70°
  4. 100°

Answer: 2

18. The supplementary angle of 100° is

  1. 10°
  2. 40°
  3. 100°
  4. 80°

Answer: 4

19. The supplement of the complement of 20° is……

  1. 160°
  2. 70°
  3. 110°
  4. 90°

Answer: 3

20. The complementary angle of 57° is…….

  1. 53°
  2. 43°
  3. 33°
  4. 23°

Answer: 3

21. The supplementary angle of 55° is.

  1. 105°
  2. 135°
  3. 125°
  4. 145°

Answer: 3

22. If an angle is double of its complement then the angle is

  1. 15°
  2. 30°
  3. 45°
  4. 60°

Answer: 2

23. An angle is 30° larger than a straight angle. Then the angle is

  1. 90°
  2. 150°
  3. 210°
  4. 390°

Answer: 3

24. In the figure, the value of x is

In the figure, the value of x is

  1. 40°
  2. 20°
  3. 60°
  4. 150°

Answer: 1

25. In the figure BOC = 60°, ∠AOC = ….

In the figure BOC = 60°, AOC =

  1. 90°
  2. 30°
  3. 120°
  4. 60°

Answer: 3

26. In the figure ABCD and AECF then ∠A- ∠C = ……………

In the figure ABCD and AE F then A- C =

  1. 90°
  2. 45°
  3. 60°

Answer: 1

27. What happens to the measurement of an angle after the extension of its arms?

  1. Doubles
  2. Triples
  3. Remains same
  4. Cannot be same

Answer: 3

28. The distance between two parallel lines is

  1. unequal
  2. equal
  3. decreases
  4. increases

Answer: 2

29. The complementary angle of an angle greater than 45°is

  1. equal to 45°
  2. less than 45°
  3. more than 45°
  4. less than 30°

Answer: 2

30. Which of the following is true?

  1. Two acute angles are supplementary
  2. Two obtuse angles are supplementary
  3. Two right angles are supplementary
  4. Two reflex angles are supplementary

Answer: 3

31. In the figure AB \\ CD and XY is the transversal. Which of the following is incorrect?

in the given figure abcd and xy i the transversal

  1. ∠p = 115°
  2. ∠q = 115°
  3. ∠q = 65°
  4. ∠r = 115°

Answer: 2

32. The sum of two angles be supplementary.If both are……

  1. Acute
  2. Obtuse
  3. Right
  4. None

Answer: 2

33. I. Assertion : ∠a = ∠b
II. Reason: These are corresponding angles

These are corresponding angles

  1. Statements I,II both are true
  2. Statements I,II both are false
  3. StatementI is true, II is false
  4. StatementI is false, II is true

Answer: 3

34. Two lines are parallel if the following statements are true

  1. Corresponding angles are equal
  2. Co-interior angles are supplementary
  3. All of the above

Answer: 4

35. According to the adjacent figure which of the following is correct where XY \\ BC?

According to the adjacent figure which of the following is correct where

  1. b = y
  2. c = x
  3.  a = b
  4. a +b + c = x + a + y

Answer: 4

36. l and m are two lines intersecting at a point. These lines are called

  1. Parallel lines
  2. Non intersecting lines
  3. None of these
  4. Intersecting lines

Answer: 3

37. OA, OB are two rays. Then which of the following is true?

OA, are two rays. Then which of the following is true

  1. a + b = 90°
  2. a – b = 0°
  3. a + b = 180°
  4. a + b = 270°

Answer: 3

38. Two lines are intersected by a transversal. The sum of the interior anglesis 130°, then the lines are

  1. parallel
  2. equal
  3. intersect
  4. none of the above

Answer: 3

39. Adjacent angles in a linear pair are

  1. 80°, 20°
  2. 60°, 120°
  3. 45°, 90°
  4. 70°, 140°

Answer: 2

40. Two complementary angles are in the ratio of 7: 2, then the angles are respectively

  1. 20°, 70°
  2. 70°, 20°
  3. 140°, 40°
  4. 35°, 10°

Answer: 2

41. Choose the correct matching.

Pair of Angles                                    Name

i) Sum of angles is 180°                       (  ) a) Linear pair

ii) Sum of angles is 90°                        (  ) b) Obtuse angle

iii) Having a common vertex and arm (  ) c) Supplementary angles

iv) Sum of adjacent angles is 180°      (  ) d) Adjacent angles

v) Angle between 90° and 180°          (  ) e) Acute angle
                                                                 f) Complementary angles

  1. i – c,ii – f,iii – d, iv – a, v – b
  2. i – a,ii -b,iii – d, iv – e, v – c
  3. i – c,ii- f,iii -b,iv – d, v-a
  4. i – a,ii- b,iii- c,iv – d, v – e

Answer: 1

42. Assume figure AB \\ CD and EF is the transversal. If angle AGH = 60° what is the measure of angle CHF?

  1. 60°
  2. 120°
  3. 180°
  4. 170°

Answer: 2

43. If the angles (2a – 10)° and (a -11)° are supplementary, what is the value of a?

  1. 47°
  2. 57°
  3. 67°
  4. 37°

Answer: 3

44. One angle of an equilateral triangle is

  1. 90°
  2. 80°
  3. 180°
  4. 60°

Answer: 4

45. Which figure shows linear pair?

Which figure shows linear pair

Answer: 3

46. The value of right angle is

  1. 180°
  2. 90°
  3. 50°

Answer: 2

47. What is the measure of ∠z if ∠x = 70°?

The value of right angle is

  1. 120°
  2. 130°
  3. 110°
  4. 140°

Answer: 3

48. If x- y = 80° what are x and y ?

From the adjacent figure value of x

  1. 100°, 80°
  2. 130°, 50°
  3. 50°, 130°
  4. 80°, 100°

Answer: 2

49. The number of all possible line segments in the figure ……..

The number of all possible line segments in the figure

  1. 6
  2. 4
  3. 3
  4. 12

Answer: 1

50. Identify the figure in which ‘n’ is transversal.

Identify the figure in which n is transversal

Answer: 3

51. Which pair of the following are complementary angles?

  1. 70°, 30°
  2. 48°, 52°
  3. 70°, 20°
  4. 35°, 55°

Answer: 3

52. Which pair of the following are supplementary angles?

  1. 110°, 50°
  2. 100°, 80°
  3. 105°, 65°
  4. 45°, 45°

Answer: 2

53. In the given figure

In the given figure find 2

[1 = 30°] then [2=?]

  1. 30°
  2. 60°
  3. 120°
  4. 150°

Answer: 4

Haryana Board Class 7 Maths Solutions For Chapter 5 Fill in the blanks :

54……….is formed when lines or line segments meet.

Answer: Angle

55. ………….. angles have a common vertex and a common arm but no common interior points.

Answer: Adjacent

56. When two lines intersect, the vertically opposite angles so formed are………..

Answer: equal

57. When lines drawn on a sheet of paper do not meet, they are………..lines.

Answer: parallel

58. When a transversal cuts two lines such that pairs of interior angles on the same side of the transversal are supplementary, the lines have to be………………

Answer: parallel

59. Match the following:

1.30°,60°                                                                                                                   (  ) A) Supplementary angles
2. 100°, 80°                                                                                                               (  ) B) Linear pair
3. Adjacent and supplementary                                                                             (  ) C) Parallel lines
4. Linesl and m intersect at 0, then the lines are called                                       (  ) D) Complementary angles
5. If there is no common point to the linesl and m, then the lines are called   (  ) E) Intersecting lines

Answer:

1. D 2. A 3. B 4. E 5: C

Haryana Board Class 7 Maths Solutions For Chapter 12 Symmetry

Haryana Board Class 7 Maths Solutions For  Chapter 12 Symmetry Key Concepts

  1. Introduction:
    Symmetry is an important geometrical concept, commonly exhibited in nature and is used almost in every field of activity. Artists, professionals, designers of clothing or jewellery, car manufacturers, architects, and many others make use of the idea of symmetry. The beehives, the flowers, the tree leaves, religious- symbols, rugs, and handkerchiefs – everywhere you. find symmetrical designs.
  2. Line symmetry: A figure has line symmetry, if there is a line about which the figure may be folded so that the two parts of the figure will coincide.
  3. Lines of symmetry for regular polygons:
    A polygon is said to be regular if all its sides are of equal length and all its angles are of equal measure. The polygon is a closed figure made, of several line segments

    1. The polygon made up of the least number of line segments is the triangle. An equilateral triangle is a regular polygon of three sides. Each of its sides has same length and each of its angles measures 60°.
    2. A square is a regular polygon of four sides. Each of its sides has’same length and each of its angles measure 90°. Its diagonals bisect each other perpendicularly.
    3. If a pentagon is regular, its sides should have equal length.
    4. A regular hexagon has all its sides equal and each of its angles measures 120°. Regular polygons are symmetrical figures. Each regular polygon has many lines of symmetry as it has several sides.
      Equilateral triangle – 3 lines of symmetry
      Square- 4 lines of symmetry
      Regular pentagon – 5 lines of symmetry
      Regular hexagon – – 6 lines of symmetry
  4. Symmetry and mirror reflection:
    The concept of linesymmetry is closely related to mirror reflection. A shape, has line symmetry. when one half of it is the mirror image of the other half. A mirror, line, thus, helps to visualized line of symmetry.
  5. Perfect symmetry: The circleIs the most symmetrical figure because it can be rotated around its centre through any angle and at the same time it unlimited number of lines of symmetry.
  6. Angle of rotational symmetry: The minimum angle of rotation of a figure to get exactly the same figure as original is called the “Angle of rotational symmetry. ”
    • The angle of rotation of symmetry of a square is 90°.
    • The angle of rotation of symmetry of a parallelogram is 180°.
    • The angle of rotation of symmetry of a circle is 0 to 360°.
  7. Order of rotational symmetry: The number of times a figure rotated through its angle of rotational symmetry before it comes to original position is called the “Order of rotational symmetry”.
    • The order of rotational symmetry of a square is 4
    • The order of rotational symmetry of an equilateral triangle is 3.
    • The formula for the order of rotational symmetry of a regular polygon \( =\frac{360^{\circ}}{\text { angle of symmetry }} \)
  8. Clearly all figures have rotational symmetry of order 1. So, we say that an object has rotational symmetry, only when the order of symmetry is more than 1.
  9. Some figures only have line symmetry,and some have only rotational symmetry and some figures have both. Squares, equilateral triangles, and circles have both the line and rotational symmetry.

Haryana Board Class 7 Maths Solutions For  Chapter 12 Symmetry Exercise 12 .1

1. Copy the figures with punched holes and find the axes of symmetry for the following:
Copy the figures with punched holes and find the axes of symmetry for the following
Solutions:

 

Each figure is symmetrical and is shown by dotted lines.

Each figure is symmetrical and is shown by dotted lines

HBSE Class 7 Symmetry Solutions

2. Given the line(s) of symmetry, find the other hole(s):

Given the line(s) of symmetry, find the other hole

Solution:

Given the line(s) ofsymmetry, find the other hole 1

Haryana Board Class 7 Maths Symmetry solutions

3. In the following figures, the mirror line (i.e. the line of symmetry) is given as a dotted line. Complete each figure performing reflection in the dotted (mirror) line.(You might perhaps place a mirror along the dotted line and look into the mirror for the image). Are you able to recall the name of the figure you complete?
In the following figures, the mirror line (i.e. the line of symmetry) is given as a
Solution:
In the following figures, the mirror line (i.e. the line of symmetry) is given as a 1
4. The following figures have more than one line of symmetry. Such figures are said to have multiple lines of symmetry.
The following figures have more than one line of symmetry
Identify multiple lines of symmetry, if any, in each of the following figures:
Identify multiple lines of symmetry, if any, in each of the following figures
Key Questions in Symmetry for Class 7 HBSE
Solution:
Identify multiple lines of symmetry, if any, in each of the following figures 1
5. Copy the figure given here. Take any one diagonal as a line of symmetry and shade a few more squares to make the figure symmetric about a diagonal.’Is there more than one way to do that? Will the figure be ” symmetric about both the diagonals?
Copy the figure given here
Solution:
Copy the figure given here 1
Yes, there is one more way to do. That is, on taking second diagonal as line of symmetry. Yes, the figure will be symmetric about both the diagonals.
Copy the figure given here 2
HBSE 7th Class Rotational Symmetry Explained
6. Copy the diagram and complete each shape to be symmetric about the mirror line(s):
Copy the diagram and complete each shape to be symmetric about the mirror line
Solution:
Copy the diagram and complete each shape to be symmetric about the mirror line 1
7. State the number of lines of symmetry for the following figures:
  1. An equilateral triangle – Three
  2. An isosceles triangle – One
  3. A scalene triangle – None
  4. A square – Four
  5. A rectangle – Two
  6. A rhombus – Four
  7. A parallelogram -None
  8. A quadrilateral -None
  9. A regular hexagon -Six
  10. A circle – Unlimited

How to find lines of symmetry Class 7 HBSE

8. What letters of the English alphabet have reflectional symmetry (i.e., symmetry related to mirror reflection) about.
  1. a vertical mirror
  2. a horizontal mirror
  3. both horizontal and vertical mirrors
Solution:
  1. About vertical mirror: A, H, I, M, O, T, U, V, W, X, Y
  2. bout horizontal mirror: B, C, D, E, H, I, K, O, X
  3. About both horizontal and vertical mirrors: H, I, O, X
9. Give three examples of shapes with no line of symmetry.
Solution:
  1. A scalene triangle
  2. A trapezium
  3. The letter G

Sample Problems Symmetry Haryana Board Class 7

10. What other name can you give to the line of symmetry of
  1. an isosceles triangle ?
  2. a circle?
Solution:
Other name to the line of symmetry of
1) an isosceles triangle is median
2) a circle is diameter.

Haryana Board Class 7 Maths Solutions For  Chapter 12 Symmetry Solutions

1.
1) Can you now tell the order of the rotational symmetry for an equilateral triangle?
Can you now tell the order of the rotational symmetry for an equilateral triangle
Solution: The order of the rotational symmetry for an equilateral triangle is 3.
2) How many positions are there at which the triangle looks exactly the same, when rotated about its centre by 120°?
Solution:
There are two positions at which the triangle looks exactly the same when rotated about its centre by 120″.
2. Which of the following shapes have rotational symmetry about the marked point?
Solution:
Which of the following shapes have rotational symmetry about the marked point
Shapes having rotational symmetry about the marked point are (1), (2), (3) and (4).
Give the order of the rotational symmetry of the given figures about the point marked x.
Give the order of the rotational symmetry of the given figures about the point marked x
Solution:
Order of the rotational symmetry of the given figures are:
(1) Four (2) Three (3) Four
Note: In a complete turn (of 360°) the number of times an object looks exactly the same is called the order of rotational symmetry.
The order of symmetry of a square is 4 For equilateral triangle it is 3.

Haryana Board Class 7 Maths Solutions For  Chapter 12 Symmetry Exercise-12.2

1. Which of the following figures have rotational symmetry of order more than 1:
Which of the following figures have rotational symmetry of order more than1 q
Solution: All figures have rotational symmetry of order more than 1
Lines of Symmetry Class 7 Haryana Board
2. Given the order of rotational symmetry for each figure.
Which of the following figures have rotational symmetry of order more than1
Solution:
(a) 2 (b) 2 (c) 3 (d) 4 (e) 4 (f) 5 (g) 6 (h) 3
By attempting to think on such lines, you will be able to fill in the following table:
By attempting to think on such lines, you will be able to fill in the following table

Solution:

By attempting to think on such lines, you will be able to fill in the following table solution

Haryana Board Class 7 Maths Solutions For  Chapter 12 Symmetry Exercise-12.3

1. Name any two figures that have both line symmetry and rotational symmetry.
Solution:
The two figures that have both line symmetry and rotational symmetry are:
  1. Equilateral triangle and
  2. Circle.
2. Draw, wherever possible, a rough sketch of
  1. a triangle with both line and rotational symmetries of order more than 1.
  2. a triangle with only line symmetry and no rotational symmetry of order more than 1.
  3. a quadrilateral with a rotational symmetry of order more than 1 but not a line symmetry.
  4. a quadrilateral with line symmetry but not a rotational symmetry of order more than 1.
Solution:
  1. A triangle with both line symmetry and rotational symmetry of order more than 1 is an equilateral triangle.
    equilateral triangle
  2. A triangle with only line symmetry and no rotational symmetry of order more than 1 is an isosceles triangle
    isosceles triangle
  3. A quadrilateral with a rotational symmetry of order more than1 but not a line symmetry is a parallelogram.
    parallelogram
  4. A quadrilateral with a line symmetry but not a rotational symmetry of order more than 1 is an isosceles trapezium.
    isosceles trapezium
HBSE Class 7 Maths Chapter 12 Guide
3. If a figure has two or more lines of symmetry, should it have rotational symmetry of order more than 1?
Solution:
Yes, a figure that has two or more lines of symmetry have a rotational symmetry of order more than 1.
4. Fill in the blanks:
Fill in the blanks 4
Solution:
Fill in the blanks 4 Answer

Haryana Board Class 7 Maths Solutions For  Chapter 12 Symmetry Fill in the blanks:

5. Name the quadrilaterals which have both line and rotational symmetry Of order more than 1.
Solution:
Square, rectangle, and rhombus.
6. After rotating by 60° about a centre, a figure looks exactly the same as its original position. At what other angles will this happen for the figure?
Solution:
The other angles are 120°, 180°, 240°,300° and 360°.
7. Can we have a rotational symmetry of order more than 1 whose angle of rotation is
  1. 45° ?
  2. 17° ?
Solution:
  1. Yes
  2. No

Haryana Board Class 7 Maths Solutions For  Chapter 12 Symmetry Very Short Answer Questions

1. What is meant by’Line Symmetry’?
Solution:
A figure has line symmetry if there is a line about which the figure may be folded so that the two parts of the figure will coincide.
Important Concepts Symmetry Class 7 HBSE
2. Write the number of lines of symmetry for (1) Regular hexagon (2) Square (3) Rectangle.
Solution:
  1. Regular hexagon –   6
  2. Square                 –   4
  3. Rectangle             –   2
3. Write
  1. Centre of rotation
  2. Angle of rotation
Solution:
  1. Rotation turns an object about a fixed point. This fixed point is called the ‘Centre of rotation’.
  2. The angle by which the object rotates is called the’Angle of rotation’.

Examples of rotational symmetry Class 7 HBSE

4. The order of symmetry for
  1. a square
  2. an equilateral triangle.
Solution:
  1. The order of symmetry for a square is 4
  2. The order of symmetry for an equilateral triangle is 3.
5. Name a few things in nature, that are symmetric.
Solution:
Leaf, Butterfly, A person’s face, Fish etc.,
6. Name 5 man-made things that are symmetric.
Solution:
Plank, paper, spectacles, blade, clock without hands, ladder.
7. What is the angle of rotational symmetry of a square?
Solution:
The angle of rotational symmetry of a square is 90°
8. What is the angle of rotational symmetry of a parallelogram?
Solution:
The angle of rotational symmetry of a parallelogram is 180°
9. What is the angle of rotational symmetry of a circle?
Solution: The angle of rotational symmetry of a circle is 0° to 360°
10. Draw any three shapes based on below sentences:
1) No line of symmetry
Solution:
No line of symmetry
2) One line of symmetry
One line of symmetry
Solution:
3) Two lines of symmetry
Two lines of symmetry
Solution:
4) Three lines of symmetry
Solution:
Three lines of symmetry
11. Find whether the following letters of the English alphabet have rotational symmetry or not. If yes, find the point of rotational symmetry (approximately), and also order of rotational symmetry.
Find whether the following letters of the English alphabet have rotational symmetry or not
Solution:
Find whether the following letters of the English alphabet have rotational symmetry or not solution
The letters of the english alphabet that have rotational symmetry are H, S, Z, O.
Order of rotational symmetry
12. Identify which of the english alphabet have point symmetry in the following:
Identify which of the english alphabet have point symmetry in the following 2
Solution:
H and S have point symmetry. Because
1) Every part of the letter has a matching part which are at the same distance from the central point.
Identify which of the english alphabet have point symmetry in the following
2) The part of the alphabet and its. matching part are in the opposite direction.
Identify which of the english alphabet have point symmetry in the following 1
Symmetry Class 7 HBSE important questions
13. Drawsome natural objects which have at least one line of symmetry.
Solution:
Draw some natural objects which have at least one line of symmetry leaf
Draw some natural objects which have at least one line of symmetry moon,lotus,lady bug

Haryana Board Class 7 Maths Solutions For  Chapter 12 Symmetry Long Answer Questions

14. Which of the following shapes have line symmetry? Which have rotational symmetry?
1)
Which of the following shapes have line symmetry Which have rotational symmetry
Solution:
  1. It has no line of symmetry.
  2. It has rotational symmetry.
2)
Which of the following shapes have line symmetry Which have rotational symmetry 2
Solution:
  1. It has lines of symmetry.
  2. It has no rotational symmetry.
3)
Which of the following shapes have line symmetry Which have rotational symmetry 3
Solution:
It has 2 lines of symmetry and rotational symmetry.
4)
Which of the following shapes have line symmetry Which have rotational symmetry 4
Solution:
It has 4 lines of symmetry and rotational symmetry.
15. Draw lines of symmetry for the following figures. Identify which of them have point symmetry. Is there any relation between lines of symmetry and point symmetry?
Solution:
Draw lines ofsymmetryfor the following figures solution
Here(1),(2),(3), and(4) figures have point symmetry.
Practice Problems Symmetry Class 7 Haryana Board
16. Cut the capital letters of English and paste them in your notebook. Draw possible number of lines of symmetry for each of the letter
Solution:
Draw possible number of lines of symmetry for each of the letter
1) How many letters have no line of symmetry? What are they?
Solution:
10 letters have no line of symmetry. They are FG J LNPQ RS Z.
2) How many letters have one line of symmetry? What are they?
Solution:
12 letters have one line of symmetry. They are A, C, D, E, I, K, M, T, U, V, W, Y.
3) How many letters have two lines of symmetry? What are they?
Solution:
2 letters have two lines of symmetry. They are H, and X.
4) How many letters have more than two lines of symmetry? What are they?
Solution:
Only one letter have more than two lines of symmetry: ‘O’.
5) Which of them have rotational symmetry? What are they?
Solution:
4 letters have rotational symmetry. They are H, O, S, and Z.
6) Which of them have point symmetry? What are they?
Solution:
7 letters have point symmetry. They are H, I, N, O, S, X and Z.
17. State the number of lines of symmetry for the following figures and draw them.
Solution:

 

1).  An equilateral triangle:

An equilateral triangle has 3 lines of symmetry.

An equilateral triangle

Reflection symmetry examples Class 7 Haryana Board

2). An isosceles triangle:

An isosceles triangle has only one line of symmetry.

An isosceles triangle

3). A scalene triangle:

A scalene triangle has no line of symmetry.

A scalene triangle

 

Haryana Board Class 7 Maths Solutions For  Chapter 12 Symmetry

Choose the correct answers:

1. A regular pentagon has lines of symmetry.
  1. 5
  2. 3
  3. 2
  4. 1
Answer: 1
2. What is the order of rotational symmetry of a square ?
  1. 4
  2. 3
  3. 6
  4. 5
Answer: 1
3. Which of the following shapes has horizontal line of symmetry?
Which of the following shapes has horizontal line of symmetry
  1. 1
  2. 2
  3. 3
  4. 4
Answer: 4
4. Which of the following is not symmetric?
  1. C
  2. L
  3. B
  4. D
Answer: 2
5. Which of the following has line of symmetry?
  1. A scalene triangle
  2. Line segment
  3. Trapezium
  4. The letter G
Answer: 2
6. Which of the following is not matched correctly? figure  order

 

Which of the following is not matched correctly

Answer: 2
7. Which of the following is odd one according to order?
  1. Rhombus
  2. Rectangle
  3. Parallelogram.
  4. Square
Answer: 4
8. Among the following which has rotational symmetry but no line of symmetry
  1. Isosceles triangle
  2. Parallelogram
  3. Circle
  4. Rhombus
Answer: 2
9. Which of the following has both line and rotational symmetry?
  1. Isosceles triangle
  2. Scalene triangle
  3. Square
  4. Parallelogram
Answer: 3
10. What is the other name for a line of symmetry of a circle?
  1. Radius
  2. Diameter
  3. Sector
  4. Arc
Answer: 2
11. Which of the following is true?
  1. A rhombus has four lines of symmetry
  2. A square has four lines of symmetry
  3. A circle has four lines of symmetry
  4. An equilateral triangle has two lines of symmetry
Answer: 2
12. What is the other name given to the line of symmetry of isosceles triangle?
  1. Median
  2. Diameter
  3. Perpendicular
  4. Radius
Answer: 1
13. Which of the letters have both lines of symmetry?
Answer: 1
14. What is the angle of rotation of B> Q equilateral triangle ?
  1. 180°
  2. 90°
  3. 120°
  4. 360°
Answer: 3
15. Statement p: The maximum angle of rotation of a figure to get exactly the same figures as original is called angle of rotational symmetry
Statement q: There is no angle of rotation for quadrilateral.
  1. Both p and q are true
  2. Both p and q are false
  3. p is false q is true
  4. p is true q is false
Answer: 3
16. How many symmetrical axes are there for a round clock?
  1. 3
  2. 4
  3. 5
  4. infinite
Answer: 4
17. Which of the following has one axis of symmetry?
  1. O
  2. B
  3. X
  4. H
Answer: 2
18. The number of axis of symmetry of an equilateral triangle
  1. 1
  2. 2
  3. 3
  4. 4
Answer: 3
19. Find the odd one out

 

Find the odd one out

Answer: 1
20. What is the mirror image of ‘D’…..

 

What is the mirror image of'D'

Answer: 2
21. What is the order of
What is the order of
  1. 0
  2. 2
  3. 3
  4. 4
Answer: 3
22. Identify the number of lines of symmetry for the given figure
Identify the number of lines of symmetry for the given figure
  1. 1
  2. 2
  3. 3
  4. 4
Answer: 2
23. Which of the following has only one line of symmetry?
Which of the following has only one line of symmetry
  1. 1
  2. 2
  3. 3
  4. 4
Answer: 4
24. Number of lines of symmetry for an equilateral triangle are
  1. 3
  2. 1
  3. 2
  4. 4
Answer: 1
25. Number of lines of symmetry for an isosceles triangle are
  1. 1
  2. 2
  3. 3
  4. 4
Answer: 1
26. Number of lines of symmetry for a circle is
  1. 1
  2. 2
  3. 5
  4. infinite
Answer: 4
27. Number of lines of symmetry for a regular pentagon
  1. 6
  2. 4
  3. 5
  4. 3
Answer: 3
28. A quarter – turn means rotation by
  1. 360°
  2. 270°
  3. 180°
  4. 90°
Answer: 4

Haryana Board Class 7 Maths Solutions For  Chapter 12 Symmetry Fill in the blanks:

29. A regular hexagon has all its sides equal and each of its angle measures………..
Answer: 120°
30. A full-turn means a rotation of…..
Answer: 360°
31. Number of lines of symmetry for the letter E……..
Answer: 1
32.. Order of rotational symmetry for the letter Z ……………
Answer: 2
33. The centre of rotation is the….. of the square.
Answer: centre
34. Number of lines of symmetry of an equilateral triangle……
Answer: 3
35. Number of lines of symmetry of an isosceles triangle…….
Answer: 1
36. Number of lines of symmetry of scalene triangle……
Answer: 0
37. A polygon with all equal sides and equal angles called ……….
Answer: regular polygon
38. Number of lines of symmetry of regular hexagon has……….
Answer: 6
39. A regular hexagon has all its sides equal and each of its angle measure is…….
Answer: 60°
40. Rotation turns an object about a fixed point. This fixed point is called……
Answer: centre of rotation
41. The angle by which the object rotates is called the……
Answer: angle of rotation
42. The order of rotational symmetry of is…..
Answer: 4
43. The angle of symmetry of adjacent figure is……
The angle of symmetry of adjacent figure is
Answer:120°
44. Each of the letters H, N, S, and Z has a rotational symmetry of order……..
Answer: 2
45. The concept line of symmetry is closely related to ……reflection.
Answer: Mirror
46. The angle of rotational symmetry for letter ‘S’ is……
Answer: 180°
47. Each regular polygon has as many lines of symmetry as it has…….
Answer: number of sides
48. The order of rotational symmetry……….
Answer:  \( \left[\frac{360^{\circ}}{x^0}\right] \)
49. The angle of rotational symmetry of adjacent figure is………..
The angle of rotational symmetry of adjacentfigure is
Answer: 90°
50. Match the following:
     
 Shape                                                                                             No. of lines of symmetry
1. Rhombus                                                                                               (  ) A) 2
2. Rectangle                                                                                              (  ) B) 0
3. Order of rotational symmetry of an equilateral triangle is              (  ) C) 4
4. A parallelogram                                                                                    (  ) D) 1
5. An isosceles triangle                                                                             (  ) E) 3
Answer:
1. C 2. A 3. E 4. B 5. D

Haryana Board Class 6 Maths Solutions For Chapter 3 Playing With Numbers

Haryana Board Class 6 Maths Solutions For  Chapter 3 Playing With Numbers

  • Divisibility Rule:
    The process of checking whether a number is divisible by a given number or not with actual division is called ‘Divisibility rule’ for that number.
  • A number is completely divisible by another number when it leaves zero as remainder.
  • A number is divisible by 2 if it has the digits 0, 2, 4, 6 or 8 in its ones place.
  • If the sum of the digits in a number is a mul- tiple of 3 then the number is divisible by 3.
  • If a number is divisible by both 2 and 3 then it is also divisible by 6.
  • A number is divisible by 9, if the sum of the digits of the number is divisible by 9.
  • A number is divisible by 5, if and only if its last digit is either 5 or 0.
  • A number is divisible by 10, if and only if its last digit is 0.
  • If the number formed by the last two digits of the given number is divisible by 4, then the given number is divisible by 4.
  • If the number formed by the last three digits of the given number is divisible by 8, then the given number is divisible by 8.
  • A number is divisible by 11 if and only if the difference of the numbers obtained on adding the alternate digits of the number is either O or divisible by 11.
    Factors:
  • A number which divides the other number without remainder is called a ‘factor’.
  • 1 is factor of every number and is the smallest of all factors.
  • Every number is a factor of itself and is the greatest of its factors.
  • Every factor is less than or equal to the given number.
  • Number of factors of a given number are countable.
  • Every number is a multiple of itself.
  • Every number is multiple of each of its factors.
  • Every multiple of a given number is greater than or equal to that number.
  • The number other than 1, with only factors namely 1 and the number itself is called a “Prime number”.
  • Numbers that have more than two factors are called “Composite numbers”.
  • ‘1’ is neither prime nor composite.
  • Finding prime numbers, easy method was given by the Greek Mathematician “Eratosthenes”.
  • The smallest even prime number is 2.
  • Every prime number other than 2 is odd.
  • The numbers which have only 1 as the common factor are called “Co-primes” or “relatively prime”.
  • Only two primes are co-primes but all the co-primes need not be primes.
  • “Twin primes” are prime numbers that differ from each other by two.
  • A number divisible by two coprime numbers is divisible by their product also.
  • If a number is divisible by another number then it is divisible by each of the factors of that number.
  • When a number is expressed as a product of its factors we say that the number has been factorized. The process of finding the factors is called ‘factorization’.
  • Methods of prime factorization :
    • Division Method
    • Factor Tree Method.
  • “Common factors” are those numbers which are factors of all the given numbers.
  • HCF – Highest Common Factor
  • GCD – Greatest Common Divisor
  • The Highest Common Factor (HCF) of two or more given numbers is the highest (or greatest) of their common factors. It is also called Greatest Common Divisor (GCD).
  • Methods for finding HCF:
    • Prime Factorization Method
    • Continued Division Method.
  • The method of division was invented by the famous Greek Mathematician “Euclid”.
  • Continued division method is useful to find the HCF of large numbers.
  • LCM – Lowest Common Multiple
  • The Lowest Common Multiple (LCM) of two or more given numbers is the lowest of their common multiples.
  • Methods for finding LCM:
    • Prime Factorization Method
    • Division Method
  • If one of the two given numbers is a multiple of the other, then the greater number will be their LCM.
  • Relation between LCM and HCF : Product of LCM and HCF of the two numbers Product of the two numbers.
  • 1221 is a “Palindrome number”.
  • If two numbers are divisible by a number then their sum and difference are also divisible by that number.
  • A factor of a number is an exact divisor of that number.
  • The number of multiples of a given number are infinite.
  • Every number is a multiple of itself.
  • A number for which sum of all its factors is equal to twice the number is called a perfect number.
  • The least perfect number is 6.
  • The numbers 2, 4, 6, 8, 10, ………… are called even numbers.
  • The numbers 1, 3, 5, 7, numbers ……. are called odd

Find the possible factors of 45, 30 and 36.

Solution. Factors of 45 are 1, 3, 5, 9, 15, 45.

Factors of 30 are 1, 2, 3, 5, 6, 10, 15, 30.

Factors of 36 are 1, 2, 3, 4, 6, 9, 12, 18, 36.

Haryana Board Class 6 Maths Solutions For  Chapter 3 Playing With Numbers Exercise-3.1

Question 1. Write all the factors of the following numbers:

Solution. a) 24

24 = 1 × 24

24 = 2 × 12

24 = 3 x 8

24 = 4 × 6

Factors of 24 are 1, 2, 3, 4, 6, 8, 12, 24.

Haryana Board Class 6 Maths Playing With Numbers solutions

b) 15

15 = 1 × 15

15 = 3 × 5

Factors of 15 are 1, 3, 5, 15.

Factors of 36 are 1, 2, 3, 4, 6, 9, 12, 18

c) 21

21 = 1 × 21

21 = 3 × 7

Factors of 21 are 1, 3, 7 and 21.

d) 27

27 = 1 × 27

27 = 3 × 9

Factors of 27 are 1, 3, 9 and 27.

e) 12

12 = 1 × 12

12 = 2 × 6

12 = 3 × 4

Factors of 12 are 1, 2, 3, 4, 6 and 12.

f) 20

20 = 1 × 20

20 = 2 x 10

20 = 4 x 5

Factors of 20 are 1, 2, 4, 5,10 and 20.

g) 18

18 = 1 x 18

18 = 2 x 9

18 = 3 x 6

Factors of 18 are 1, 2, 3, 6, 9 and 18.

h) 23

23 = 1x 23

Factors of 23 are 1 and 23.

i) 36

36 = 1 x 36

36 = 2 x 18

36 = 3 x 12

36 = 4 x 9

36 = 6 x 6

Factors of 36 are 1, 2, 3, 4, 6, 9, 12, 18 and 36.

Question 2. Write first five multiples of:

a) 5

Solution. The first five multiples of 5 are 5, 10, 15, 20, 25.

b) 8

Solution. The first five multiples of 8 are 8, 16, 24, 32 and 40.

c) 9

Solution. The first five multiples of 9 are 9, 18, 27, 36 and 45.

Question 3. Match the items in column – 1 with the items in column – 2

Class 6 Maths Chapter 3 Playing With Numbers Question 3

Solution. 1-b, 2-d, 3-a, 4-f, 5-e.

Question 4. Find all the multiples of 9 upto 100.

Solution. The multiples of 9 upto 100 are, 9, 18, 27, 36, 45,54, 63, 72, 81, 90 and 99.

Question 5. Observe that 2 x 3 + 1 = 7 is a prime number. Here, 1 has been added to a multiple of 2 to get a prime number. Can you find some more numbers of this type?

Solution. 2 × 5 + 1 = 10 + 1 = 11, Prime number

2 x 6 + 1 = 12 + 1 = 13, Prime number

2 x 8 + 1 = 16 + 1 = 17, Prime number

2 × 9 + 1 = 18 + 1 = 19, Prime number

2 x 11 + 1 = 22 + 1 = 23, Prime number

Haryana Board Class 6 Maths Solutions For  Chapter 3 Playing With Numbers Exercise 3.2

Question 1. What is the Sum of any two

a) Odd numbers

Solution. The sum of any two odd numbers is an even number.

b) Even numbers

Solution. The sum of any two even numbers is an even number.

Question 2. State whether the following statements are True (T) or False (F).

a) The sum of three odd numbers is even.

Solution. False. [For eg: 3 +5 +9 = 17 is an odd number.]

b) The sum of two odd numbers and one even number is even..

Solution. True.

c) The product of three odd numbers is odd.

Solution. True.

d) If an even number is divided by 2, the quotient is always odd.

Solution. False. [Even numbers are those which are divided by 2. The quotient need not always be odd.]

e) All prime numers are odd.

Solution. False [The prime number 2 is an even number]

f) Prime numbers do not have any factors.

Solution. False.

HCF and LCM Class 6 HBSE Maths

g) Sum of two prime numbers is always even.

Solution. False. [For eg. The total of two prime numbers 2 and 3 is not an even number.]

h) 2 is the only even prime number.

Solution. True.

i) All even numbers are composite numbers.

Solution. False.

j) The porduct of two even numbers is always even.

Solution. True.

Question 3. The numbers 13 and 31 are prime numbers. Both these numbers have same digits 1 and 3. Find such pairs of prime numbers upto 100.

Solution. 17, 71; 37,73; 79,97.

Question 4. Write down separately the prime and composite numbers less than 20.

Solution.

Class 6 Maths Chapter 3 Playing With Numbers Question 4

Question 5. What is the greatest prime number between 1 and 10?

Solution. The greatest prime number between 1 and 10 is 7.

Question 6. Express the following as the sum of two odd primes.

a) 44

Solution. 44 = 7 + 37

b) 36

Solution. 36 = 7 + 29

c) 24

Solution. 24 = 5 + 19

d) 18

Solution. 18 = 5 + 13 and 7 + 11

Question 7. Give three pairs of prime numbers whose difference is 2. (Remark: Two prime numbers whose difference is 2 are called twin primes)

Solution. The three pairs of prime numbers whose difference is 2 are. 3, 5; 5, 7:11, 13.

Question 8. Which of the following numbers are prime?

a) 23

b) 51

c) 37

d) 26

Solution. 23, 37 are prime numbers.

Question 9. Write seven consecutive composite numbers less than 100 so that there is no prime number between them.

Solution. 90, 91, 92, 93, 94, 95, 96.

Question 10. Express each of the following numbers as the sum of three odd primes:

a) 21

b) 31

c) 53

d) 61

Solution.

a) 21 = 3 + 7 + 11

b) 31 = 7 + 11 + 13

c) 53 = 11 + 13 + 29

d) 61 = 11 + 13 + 37

Question 11. Write five pairs of prime numbers less than 20 whose sum is divisible by 5.

(Hint: 3 + 7 = 10)

Solution. 2. 3; 2, 13: 3, 7:3, 17; 11, 19.

Question 12. Fill in the blanks:

a) A number which has only two factors is called a …

Answer. Prime Number

b) A number which has more than two factors is called a

Answer. Composite Number

c) 1 is neither………nor………….

Answer. Prime, Composite

d) The smallest prime number is….

Answer. 2

e) The smallest composite number is

Answer. 4

f) The smallest even number is…

Answer. 2

Haryana Board Class 6 Maths Solutions For  Chapter 3 Playing With Numbers Exercise-3.3

Question 1. Using divisibility tests, determine which of the following numbers are divisible by 2, by 3, by 4, by 5, by 6, by 8, by 9, by 10, by 11 (say: yes or no)

Class 6 Maths Chapter 3 Playing With Numbers Exercise 3.3 Question 1

 

Solution.

Class 6 Maths Chapter 3 Playing With Numbers Exercise 3.3 Question 1 Solution

Question 2. Using divisibility tests, determine which of the following numbers are divisible by 4; by 8:

a) 572

b) 726352

c) 5500

d) 6000

e) 12159

f) 14560

g) 21084

h) 31795072

i) 1700

j) 2150

Solution. a) 572

1) Test for divisibility by 4:

Number with last two digits = 72

Class 6 Maths Chapter 3 Playing With Numbers Question 2.1

∴ 72 is divisible by 4.

∴ 572 is divisible by 4.

Last two digits are divisible by 4, givne number is divisible by 4.

2) Test for divisibility by 8

Number with last three digits = 572

Class 6 Maths Chapter 3 Playing With Numbers Question 2.2

Remainder is not zero.

∴ 572 is not divisible by 8.

Last three digits are divisible by 8. So given number is divisible by 8.

b) 726352

1) Test for divisibility by 4:

Number with last two digits = 52

Class 6 Maths Chapter 3 Playing With Numbers Question 2.3

The remainder is zero.

∴ 52 is divisible by 4

∴ 726352 is divisible by 4.

2) Test for divisibility by 8:

Number with last three digits = 352

Class 6 Maths Chapter 3 Playing With Numbers Question 2.4

The remainder is 0, 352 is divisible by 8.

∴ 726352 is divisible by 8.

Prime and composite numbers Class 6 HBSE

c) 5500

1) Test for divisibility by 4:

As the last two digits are ‘0’ and.

‘0’ is divisible by 4

∴ 5500 is divisible by 4.

2) Test for divisibility by 8:

Number with last three digits = 500

Class 6 Maths Chapter 3 Playing With Numbers Question 2.5

The remainder is not zero.

∴ 500 is not divisible by 8

∴ 5500 is not divisible by 8.

d) 6000

1) Test for divisibility by 4:

The number with last two digits is 00, always divisible by 4.

∴ 6000 is divisible by 4.

2) Test for divisibility by 8:

The number with last three digits is 000, always divisible by 8.

∴ 6000 is divisible by 8.

e) 12159

1) Test for divisibility by 4:

The number with last two digits = 59

Class 6 Maths Chapter 3 Playing With Numbers Question 2.6

The remainder is not zero.

∴ 59 is not divisible by 4.

∴ 12159 is not divisible by 4.

2) Test for divisibility by 8:

The number with last three digits = 159

Class 6 Maths Chapter 3 Playing With Numbers Question 2.7

Remainder is zero.

∴ 159 is divisible by 8.

∴ 12159 is divisible by 8.

f) 14560

1) Test for divisibility by 4:

The number with last two digits = 60

Class 6 Maths Chapter 3 Playing With Numbers Question 2.8

The remainder is zero.

∴ 60 is divisible by 4.

∴ 14560 is divisible by 4.

2) Test for divisibility by 8:

The number with last three digits = 560

Class 6 Maths Chapter 3 Playing With Numbers Question 2.9

Remainder is not zero.

∴ 560 is not divisible by 8.

∴ 14560 is not divisible by 8.

g) 21084

1) Test for divisibility by 4:

The number with last two digits = 84

Class 6 Maths Chapter 3 Playing With Numbers Question 2.10

The remainder is zero.

∴ 84 is divisible by 4.1

∴ 21084 is divisible by 4.

2) Test for divisibility by 8:

The number with last two digits = 084.

Class 6 Maths Chapter 3 Playing With Numbers Question 2.11

Remainder is not zero.

∴ 84 is not divisible by 8.

∴ 21084 is not divisible by 8.

h) 31795072

1) Test for divisibility by 4:

The number with last two digits = 50

Class 6 Maths Chapter 3 Playing With Numbers Question 2.12

Remainder is zero.

∴ 72 is divisible by 4.

∴ 31795072 is divisible by 4.

2) Test for divisibility by 8:

The number with last three digits = 072

Class 6 Maths Chapter 3 Playing With Numbers Question 2.13

Remainder is zero.

∴ 72 is divisible by 8

∴ 31795072 is divisible by 8.

i) 1700

1) Test for divisibility by 4:

The number formed with last two digits = 00, which is always divisible by 4.

∴ 1700 is divisible by 4.

2) Test for divisibility by 8:

The number with last three digits = 700

Class 6 Maths Chapter 3 Playing With Numbers Question 2.14

Remainder is not zero.

∴ 700 is not divisible by 8

∴ 1700 is not divisible by 8.

j) 2150

1) Test for divisibility by 4:

The number with last two digits = 50

Class 6 Maths Chapter 3 Playing With Numbers Question 2.15

Remainder is not zero.

∴ 50 is not divisible by 4.

∴ 2150 is not divisible by 4.

2) Test for divisibility by 8:

The number with last three digits = 150

Class 6 Maths Chapter 3 Playing With Numbers Question 2.16

Remainder is not zero.

∴ 150 is not divisible by 8

∴ 2150 is not divisible by 8.

Divisibility rules Class 6 Haryana Board

Question 3. Using divisiblity tests, determine. which of the following numbers are divisible by 6:

Solution. 6 is divisible by 2, 3

a) 297144

1) Test for divisibility by 2: Units digit = 4 is divisible by 2.

∴ 297144 is divisible by 2.

2) Test for divisibility by 3:

Sum of digits = 2 + 9 + 7 + 1 + 4 + 4 = 27

27 is divisible by 3

∴ 297144 is divisible by 3.

Since 297144 is divisible by both 2 and 3, it is divisible by 6.

b) 1258

1) Test for divisibility by 2:

Units digit = 8 is divisible by 2.

∴ 1258 is divisible by 2.

2) Test for divisibility by 3:

Sum of digits = 1 + 2 + 5 + 8 = 16

16 is not divisible by 3

∴ 1258 is not divisible by 3.

∴ 1258 is not divisible by 3 but divisible by 2, So it is not divisible by 6.

c) 4335

1) Test for divisibility by 2:

Units digit = 5 is not divisible by 2.

∴ 4335 is not divisible by 2.

∴ 4335 is not divisible by 6.

d) 61233

1) Test for divisibility by 2:

Units digit = 3 is not divisible by 2.

∴ 61233 is not divisible by 2.

∴ 61233 is not divisible by 6.

e) 901352

1) Test for divisibility by 2:

Units digit = 2.

∴ 901352 is divisible by 2.

2) Test for divisibility by 3: Sum of the digits = 9 + 0 + 1 + 3 + 5 + 2 = 20.

20 is not divisible by 3

∴ 901352 is not divisible by 3.

∴ 901352 is not divisible by 6.

f) 438750

1) Test for divisibility by 2:

Units digit = 0 is always divisible by 2

∴ 438750 is divisible by 2.

2) Test for divisibility by 3:

Sum of digits = 4 + 3 + 8 + 7 + 5 + 0 = 27.

27 is divisible by 3

∴ 438750 is divisible by 3.

∴ 438750 is divisible by 2 and 3.

∴ 438750 is divisible by 6.

g) 1790184

1) Test for divisibility by 2:

Units digit = 4 is divisible by 2.

∴ 1790184 is divisible by 2.

2) Test for divisibility by 3:

Sum of the digits = 1 + 7 + 9 + 0 + 1 + 8 + 4 = 30

30 is divisible by 3

∴ 1790184 is divisible by 3.

1790184 is divisible by both 2 and 3.

∴ 1790184 is divisible by 6.

h) 12583

1) Test for divisibility by 2:

Units digit = 3 is not divisible by 2.

∴ 12583 is not divisible by 2.

∴ 12583 is not divisible by 6.

i) 639210

1) Test for divisibility by 2:

Units digit = 0

∴ 639210 is divisible by 2.

2) Test for divisibility by 3:

Sum of the digits = 6 + 3 + 9 + 2 + 1 + 0 = 21.

21 is divisible by 3

∴ 639210 is divisible by 3.

∴ 639210 is divisible by 2 and 3.

∴ 639210 is divisible by 6.

j) 17852

1) Test for divisibility by 2:

Units digit = 2

∴ 17852 is divisible by 2.

2) Test for divisible by 3:

Sum of the digits = 1 + 7 + 8 + 5 + 2 = 23

23 is not divisible by 3

∴ 17852 is divisible by 3.

∴ 17852 is not divisible by 6.

Question 4. Using divisibility tests, determine which of the following numbers are divisible by 11:

a) 5445

b) 10824

c) 7138965

d) 70169308

e) 10000001

f) 901153

Solution. a) 5445

Sum of the digits at odd places from the right = 5 + 4 = 9

Sum of the digits at even places from the right = 4 + 5 = 9

Difference = 9 – 9 = 0

‘0’ is divisible by 11.

∴ 5445 is divisible by 11.

b) 10824

Sum of the digits at odd places from the right = 4 + 8 + 1 = 13

Sum of the digits at even places from the right = 2 + 0 = 2

Difference = 13 – 2 = 11 divisible by 11

∴ 10824 is divisble by 11.

c) 7138965

Sum of the digits at odd places from the right = 5 + 9 + 3 + 7 = 24

Sum of the digits at even places from the right = 6 + 8 + 1 = 15

Difference = 24 – 15 = 9, not divisible by 11

∴ 7138965 is not divisible by 11.

d) 70169308

Sum of the digits at odd places from the right = 8 + 3 + 6 + 0 = 17

Sum of the digits at even places from the right = 0 + 9 + 1 + 7 = 17

Difference 17 – 17 = 0

‘0’ is divisible by 11

∴ 70169308 is divisible by 11.

e) 10000001

Sum of the digits at odd places from the right = 1 + 0 + 0 + 0 = 1

Sum of the digits at even places = 0 + 0 + 0 + 1 = 1

Difference = 1 – 1 = 0

‘0’ is divisible by 11.

∴ 10000001 is divisible by 11.

f) 901153

Sum of the digits at odd places from the right = 3 + 1 + 0 = 4

Sum of the digits at even places from the right = 5 + 1 + 9 = 15

Difference = 15 – 4 = 11

∴ 11 is divisible by 11

∴ 901153 is divisible by 11.

Question 5. Write the smallest digit and the greatest digit in the blank space of each of the following numbers so that the number formed is divisible by 3.

Solution. a) _6724

1) Smallest digit

Sum of the digits = 6 + 7 + 2 + 4 = 19

19 is not divisible by 3.

21 is the nearest number to 19 which is divisible by 3.

So, 2 is the smallest number that fills the blank.

∴ Smallest number is 2 i.e. 26724.

2) Largest digit

The largest digit is 8 (19 + 8 = 27)

b) 4765_2

Sum of the given digits

= 4 + 7 + 6 + 5 + 2 = 24

24 is divisible by 3

∴ Smallest digit is 0

Largest digit is 9.

Question 6. Write the digit in the blank space of each of the following numbers so that the number formed is divisible by 11:

a) 92_389

b) 8_9484

Solution. a) 92_389

The sum of digits at odd places from the right = 9 + 3 + 2 = 14

Sum of the digits at even places from the right = 8 + 9 = 17

Difference = 17 – 14 = 3

To make the difference divisible by 11, we have to add 8 to 3.

∴ Blank space must be 8.

∴ Required number-928389.

b) 8_9484

Sum of the digits at odd places from the right = 4 + 4 = 8

Sum of the digits at even places from the right = 8 + 9 + 8 = 25

Difference 25 – 8 = 17

To make the difference divisible by. 11, we have to deduct 6 from 17.

∴ Digit in the blank space = 6

∴ Required number = 869484.

Question 7. Find the common factors of

a) 8, 20

b) 9, 15

Solution. a) Factors of 8 are 1, 2, 4, 8.

Factors of 20 are 1, 2, 4, 5, 10, 20.

Common factors are 1, 2, 4.

b) Factors of 9 are 1, 3, 9.

Factors of 15 are 1, 3, 5, 15.

Common factors are 1, 3.

Chapter 3 Playing With Numbers Exercise 3.4

Question 1. Find the common factors of:

a) 20 and 28

b) 15 and 25

c) 35 and 50

d) 56 and 120.

Solution. a) 20 and 28.

Factors of 20 are 1, 2, 4, 5, 10 and 20.

Factors of 28 are 1, 2, 4, 7, 14 and 28.

Common factors are 1, 2, 4.

b) 15 and 25

Factors of 15 are 1, 3, 5 and 15.

Factors of 25 are 1, 5 and 25:

Common factors are 1,5.

c) 35 and 50

Factors of 35 are 1, 5, 7 and 35.

Factors of 50 are 1, 2, 5, 10, 25 and 50.

Common factors are 1, 5.

d) 56 and 120

Factors of 56 are 1, 2, 4, 7, 8, 14, 28, 56.

Factors of 120 are 1, 2, 3, 4, 5, 6, 10, 12, 20, 24, 30, 40, 60, 120.

Common factors are 1, 2, 4.

Question 2. Find the common factors of:

a) 4, 8 and 12

b) 5, 15 and 25

Solution. a) 4, 8 and 12

Factors of 4 are 1, 2, 4.

Factors of 8 are 1, 2, 4 and 8.

Factors of 12 are 1, 2, 3, 4, 6, 12.

Common factors are 1, 2 and 4.

b) 5, 15 and 25

Factors of 5 are 1 and 5.

Factors of 15 are 1, 3, 5 and 15.

Factors of 25 are 1,5 and 25.

Common factors are 1, 5.

Question 3. Find first three common multiples of:

a) 6 and 8

b) 12 and 18

Solution. a) 6 and 8

Multiples of 6 are 6, 12, 18, 24, 30, 30, 36, 42, 48, 54, 60, 66, 72, 78, ……….

Multiples of 8 are 8, 16, 24, 32, 40, 48, 56, 64, 72,…….

First three common multiples are 24, 48 and 72.

b) 12 and 18

Multiples of 12 are 12, 24, 36, 48, 60, 72, 84, 96, 108, 120, ………

Multiples of 18 are 18, 36, 54, 72, 90, 108, 126 …….

First three common multiples are 36, 72, 108.

Question 4. Write all the numbers less than 100 which are common multiples of 3 and 4.

Solution. The multiples of 3 are 3, 6, 9, 12, 15, 18, 21, 24, 27, 30, 33, 36, 39, 42, 45, 48, 51, 54, 57, 60, 63, 66, 69, 72, 75, 78, 81, 84, 87, 90, 93, 96, 99, 102, 105…..

Multiples of 4 are 4, 8, 12, 16, 20, 24, 28, 32, 36, 40, 44, 48, 52, 56, 60, 64, 68, 72, 76, 80, 84, 88, 92, 96, 100…..

Common multipies of 3 and 4 are 12, 24, 36, 48, 60,72, 84, 96, 108……

∴ Common multiples less than 100 are 12, 24, 36, 48, 60, 72, 84, 96

Question 5. Which of the following numbers are co-prime?

a) 18 and 35

b) 15 and 37

c) 30 and 415

d) 17 and 68

e) 216 and 215

f) 81 and 16

Solution. Two numbers having 1 as a common factor are called co-prime numbers.

a) 18 and 35

Factors of 18 are 1, 2, 3, 6, 9 and 18.

Factors of 35 are 1, 5, 7 and 35.

Common factor is 1.

∴ 18 and 35 are co-prime numbers.

b) 15 and 37

Factors of 15 are 1, 3, 5 and 15.

Factors of 37 are 1 and 37.

Common factor is 1.

∴ 15 and 37 are co-prime numbers.

c) 30 and 415

Factors of 30 are 1, 2, 3, 5, 6, 10, 15. and 30.

Factors of 415 are 1, 5, 83 and 415

Common factors are 1 and 5

∴ 30 and 415 are not co-primes.

d) 17 and 68

Factors of 17 are 1 and 17.

Factors of 68 are 1, 2, 4, 17, 34 and 68.

Common factors are 1 and 17.

∴ 17 and 68 are not co-prime numbers.

e) 216 and 215

Factors of 216 are 1, 2, 3, 4, 6, 8, 9, 12, 18, 24, 27, 36, 54, 72, 108 and 216.

Factors of 215 are 1, 5, 43, and 215.

Common factors is 1 only.

∴ 216 and 215 are co-prime numbers.

f) 81 and 16

Factors of 81 are 1, 3, 9, 27 and 81

Factors of 16 are 1, 2, 4, 8 and 16.

Common factor is 1.

∴ 81 and 16 are co-prime numbers.

Question 6. A number is divisible by both 5 and 12. By which other number will that number be always divisible?

Solution. The number divisible by both 5 and 12 is 60(12 × 5). The other number by which 60 is always divisible is 15. (15 × 4 = 60)

Question 7. A number is divisible by 12. By what other numbers will that number be divisible?

Solution. Factors of 12 are 1, 2, 3, 4, 6 and 12.

∴ The number is also divisible by 1, 2, 3, 4 and 6 also.

Question 8. Write the prime factorisation of 16, 28, 38.

Solution. 16 = 2 x 2 x 2 x 2

28 = 2 × 2 × 7

38 = 2 × 19

Haryana Board Class 6 Maths Solutions For  Chapter 3 Playing With Numbers Exercise 3.5

Question 1. Here are two different factor trees for 60. Write the missing numbers.

a)

Class 6 Maths Chapter 3 Playing With Numbers Exercise 3.5 Question 1

Solution.

Class 6 Maths Chapter 3 Playing With Numbers Exercise 3.5 Question 1 Solution

b)

Class 6 Maths Chapter 3 Playing With Numbers Exercise 3.5 Question 2

Solution.

Class 6 Maths Chapter 3 Playing With Numbers Exercise 3.5 Question 2 Solution

Question 2. Which factors are not included in the prime factorisation of a composite number?

Solution. 1 and composite factors are not included in the prime factorisation of a composite number.

Question 3. Write the greatest four digit number and express it in terms of its prime factors.

Solution. The greatest four digit number is 9999.

Class 6 Maths Chapter 3 Playing With Numbers Exercise 3.5 Question 3

∴ 99993 x 3 x 11 x 101 x 1

Factors and multiples Class 6 HBSE

Question 4. Write the smallest five digit number and express it into the form of prime factors.

Solution. The smallest five digit number is 10000

Class 6 Maths Chapter 3 Playing With Numbers Exercise 3.5 Question 4

10000 = 2 x 2 x 2 x 2 x 5 x 5 x 5 x 5 x 1.

Question 5. Find all the prime factors of 1729 and arrange them in ascending order. Now state the relation, if any; between two consecutive prime factors.

Solution.

1729 = 7 x 13 × 19 × 1

All the prime factors of 1729 are 7, 13 and 19.

The ascending order is 7, 13, 19.

We, observe that the difference is the same between two consecutive prime factors.

13 – 7 = 6

19 – 13 = 6

Question 6. The product of three consecutive number is always divisible by 6. Verify this statement with the help of some examples.

Solution. Example 1:

Let us take three consecutive numbers be 21, 22 and 23.

21 is divisible by 3

22 is divisible by 2

∴ 21 x 22 is divisible by 6.

∴ 21 x 22 x 23 is divisible by 6.

Example 2:

Let the three consecutive numbers be 47, 48 and 49.

48 is divisible by both 2 and 3.

(2 x 3 = 6)

∴ 47 x 48 x 49 is divisible by 6.

Question 7. The sum of two consecutive odd num- ber is divisible by 4. Verify this statement with the help of some examples.

Solution. Example 1:

Let us take two consecutive odd numbers 13 and 15

Sum of the number = 13 + 15 = 28.

∴ 28 is divisible by 4.

Example 2:

Let the two consecutive odd numbers be 17 and 19

Sum of the numbers = 17 + 19 = 36

∴ 36 is divisible by 4.

Example 3:

Let the two consecutive odd numbers be 25 and 27.

Sum of the numbers = 25 + 27 = 52

∴ 52 is divisible by 4.

Question 8. In which of the following expressions, prime factorisation has been done?

a) 24 = 2 x 3 x 4

b) 56 = 7 x 2 x 2 x 2

c) 70 = 2 x 5 x 7

d) 54 = 2 × 3 × 9

Solution. a) Prime factorisation has been done.

b) Prime factorisation has not been done.

c) Prime factorisation has been done.

d) Prime factorisation has not been done.

Question 9. 18 is divisible by both 2 and 3. It is also divisible by 2 × 3 = 6. Similarly, a number is divisible by both 4 and 6. Can we say that the number must also be divisible by 4 × 6 = 24? If not, give an example to justify your answer.

Solution. No! we cannot conclude that the number will be divisible by 4 × 6 = 24 if it is divisible by both 4 and 6 because 4 and 6 are not co-prime numbers they have two common factors 1 and 2.

Example: 36 is divisible by 4 and 6 but 36 is not divisible by 24.

Question 10. I am the smallest number, having four different prime factors. Can you find me?

Solution. The four different prime factors = 2, 3, 5,7.

To find out the smallest number, we have to write them in their ascending order and multiply 2 × 3 × 5 × 7 = 210.

210 is the smallest number, having four different prime factors 2,3,5 and 7.

Question 11. Find the HCF of the following:

1) 24 and 36

2) 15, 25 and 30

3) 8 and 12

4) 12, 16 and 28.

Solution. 1) 24 and 36

Factors of 24 are 1, 2, 3, 4, 6, 8, 12 and 24.

Factors of 36 are 1, 2, 3, 4, 6, 9, 12, 18 and 36.

Highest common factor is 12.

∴ HCF of 24 and 36 is 12.

2) 15, 25 and 30

Factors of 15 are 1, 3, 5 and 15

Factors of 25 are 1,5 and 25

Factors of 30 are 1, 2, 3, 5, 6, 10, 15 and 30.

∴ HCF of 15, 25 and 30 is 5.

3) 8 and 12

Factors of 8 are 1, 2, 4 and 8

Factors of 12 are 1, 2, 3, 4, 6 and 12

HCF of 8 and 12 is 4.

4) 12, 16 and 28

Factors of 12 are 1, 2, 3, 4, 6 and 12

Factors of 16 are 1, 2, 4, 8 and 16

Factors of 28 are 1, 2, 4, 7, 14 and 28.

HCF of 12, 16 and 28 is 4.

Haryana Board Class 6 Maths Solutions For  Chapter 3 Playing With Numbers Exercise 3.6

Question 1. Find the HCF of the following numbers:

a) 18, 48

b) 30,42

c) 18, 60

d) 27,63

e) 36,84

f) 34, 102

g) 70, 105, 175

h) 91, 112, 49

j) 12, 45, 75

i) 18, 54, 81

Solution. a) 18, 48

Factors of 18 are 1, 2, 3, 6, 9 and 18.

Factors of 48 are 1, 2, 3, 4, 6, 8, 12, 16, 24 and 48.

Common factors are 1, 2, 3 and 6.

∴ HCF of 18 and 48 is 6.

b) 30, 42

Factors of 30 are 1, 2, 3, 5, 6, 10, 15, and 30.

Factors of 42 are 1, 2, 3, 6, 7, 14, 21 and 42.

Common factors are 1, 2, 3 and 6.

∴ HCF of 30 and 42 is 6.

c) 18, 60

Factors of 18 are 1, 2, 3, 6, 9 and 18.

Factors of 60 are 1, 2, 3, 4, 5, 6, 10, 12, 15, 20, 30 and 60.

Common factors are 1, 2, 3, and 6.

∴ HCF of 18 and 60 is 6.

d) 27, 63

Factors of 27 are 1, 3, 9 and 27.

Factors of 63 are 1,3,7,9, 21 and 63.

Common factors are 1, 3 and 9.

∴ HCF of 27 and 63 is 9.

e) 36, 84

Factors of 36 are 1,2,3,4,6,9, 12, 18 and 36.

Factors of 84 are 1, 2, 3, 4, 7, 12, 14, 21, 42 and 84.

Common factors are 1, 2, 3, 4 and 12.

∴ HCF of 36 and 84 is 12.

f) 34, 102

Factors of 34 are 1, 2, 17 and 34.

Factors of 102 are 1, 2, 3, 6, 17, 34, 51 and 102.

Common factors are 1, 2, 17 and 34.

∴ HCF of 34 and 102 is 34.

g) 70, 105, 175

Factors of 70 are 1, 2, 5, 7, 10, 14, 35 and 70.

Factors of 105 are 1, 3, 5, 7, 15, 21, 35 and 105:

Factors of 175 are 1, 5, 7, 25, 35 and 175.

Common factors are 1, 5 and 35.

∴ HCF of 70, 105 and 175 is 35.

h) 91, 112, 49

Factors of 91 are 1, 7, 13 and 91.

Factors of 112 are 1, 2, 4, 7, 8, 14, 16, 28, 56 and 112.

Factors of 49 are 1, 7 and 49.

Common factors are 1 and 7.

∴ HCF of 91, 112, 49 is 7.

i) 18, 54, 81

Factors of 18 are 1, 2, 3, 6, 9 and 18.

Factors of 54 are 1, 2, 3, 6, 9, 18, 27 and 54.

Factors of 81 are 1, 3, 9, 27 and 81.

Common factors are 1, 3, 9.

∴ HCF of 18, 54, 81 is 9.

j) 12, 45, 75

Factors of 12 are 1, 2, 3, 4, 6 and 12.

Factors of 45 are 1,3,5,9, 15 and 45.

Factors of 75 are 1, 3, 5, 15, 25 and 75.

Common factors are 1 and 3.

∴ HCF of 12, 45 and 75 is 3.

Question 2. What is the HCF of two consecutive.

a) Numbers ?

b) Even numbers?

c) Odd numbers?

Solution. a) Numbers

HCF of two consecutive numbers is 1.

b) Even numbers

HCF of two consecutive even number is 2.

c) Odd numbers

HCF of two consecutive odd numbers is 1.

Question 3. HCF of co-prime numbers 4 and 15 was found as follows by factorisation : 4 = 2 × 2 and 15 = 3 x 5 since there is no common prime factor, so HCF of 4 and 15 is 0. Is the answer correct? If not, what is the correct HCF?

Solution. No, the answer is not correct.

The HCF of 4 and 15 is 1.

Haryana Board Class 6 Maths Solutions For  Chapter 3 Playing With Numbers Exercise – 3.7

Question 1. Renu purchases two bags of fertiliser of weights 75 kg and 69 kg. Find the maximum value of weight which can measure the weight of the fertiliser exact number of times.

Solution. Factors of 75 are 1, 3, 5, 15, 25 and 75

Factors of 69 are 1, 3, 23 and 69.

Common factors are 1 and 3.

HCF of 75 and 69 is 3.

The maximum capacity of weight which can measure the weight of the fertiliser exact number of times is 3 kg.

Question 2. Three boys step off together from the same spot. Their steps measure 63 cm, 70 cm and 77 cm respectively. What is the minimum distance each should cover so that all can cover the distance in complete steps?

Solution.

Class 6 Maths Chapter 3 Playing With Numbers Exercise 3.7 Question 2

LCM of 63, 70 and 77

= 2 x 3 x 3 x 5 x 7 x 11 = 6930

The minimum distance each should cover so that all cover the distance in complete steps is 6930 cm. or 69 m 30 cm.

Word problems on HCF and LCM for Class 6 HBSE

Question 3. The length, breadth and height of a room are 825 cm, 675 cm and 450 cm respectively. Find the longest tape which can measure the three dimen- sions of the room exactly.

Solution. Factors of 825 are 1, 3, 5, 11, 15, 25, 33, 55, 75, 165, 275 and 825.

Factors of 450 are 1, 2, 3, 5, 6, 9, 10, 15, 18,25,30,45,50, 75, 90, 150, 225 and 450.

Common factors are 1, 3, 5, 15, 25 and 75.

HCF = 75

The length of largest tape which can measure the three dimensions of the room exactly is 75 cm.

Question 4. Determine the smallest 3-digit number which is exactly divisible by 6, 8 and 12.

Solution.

Class 6 Maths Chapter 3 Playing With Numbers Exercise 3.7 Question 4

L.C.M = 2 x 2 x 3 x 1 x 2 x 1 = 24

Multiples of 24 are 24, 48, 72, 96, 120, 144……

The smallest 3-digit number which is exactly divisible by 6, 8 and 12 is 120.

Question 5. Determine the greatest 3-digit num- ber exactly divisible by 8, 10 and 12.

Solution.

Class 6 Maths Chapter 3 Playing With Numbers Exercise 3.7 Question 5

LCM = 2 × 2 × 2 × 3 × 5 = 120

Multiples of 120 are 120, 240, 360, 480, 600, 720, 840, 960, 1080 ……….

∴ The largest 3-digit number exactly divisible by 8, 10 and 12 is 960.

Question 6. The traffic lights at three different road crossings change after every 48 seconds, 72 seconds and 108 seconds respectively. If they change simultaneously at 7 a.m. at what time will they change simultaneously again?

Solution.

Class 6 Maths Chapter 3 Playing With Numbers Exercise 3.7 Question 6

L.C.M. = 2 x 2 x 2 x 2 x 3 x 3 x 3 = 432

432 seconds = \(\frac{432}{60}\) = 7 min. 12 seconds.

(1 minute = 60 secs)

∴ They will change simultaneously again at 7 min. 12 seconds after 7 am:.

Question 7. Three tankers contain 403 litres, 434 litres and 465 litres of diesel respectively. Find the maximum capacity of a container that can measure the diesel of the three containers exact number of times.

Solution. Factors of 403 are 1, 13, 31 and 403.

Factors of 465 are 1, 3, 5, 15, 31, 93, 155 and 465.

Factors of 434 are 1, 2, 7, 14, 31, 62, 217 and 434.

Common factors are 1 and 31. HCF = 31 The maximum capacity of the container that can measure the diesel of the three containers exact number of times is 31 litres.

Question 8. Find the least number which when divided by 6, 15 and 18 leave remainder 5 in each case.

Solution.

Class 6 Maths Chapter 3 Playing With Numbers Exercise 3.7 Question 8

LCM = 2 x 3 x 3 x 5 = 90

∴ Required number = 90 + 5 = 95.

Question 9. Find the smallest four digit number which is divisible by 18, 24 and 32.

Solution.

Class 6 Maths Chapter 3 Playing With Numbers Exercise 3.7 Question 9

LCM = 2 × 2 × 2 x 2 x 3 x 3 x 2 = 288

Multiples of 288 are 288 × 1

288 × 1 = 288

288 x 2 = 576

288 × 3 = 864

288 x 4 = 1152

∴ The smallest 4-digit number which is divisible by 18, 24 & 32 is 1152.

Question 10. Find the LCM of the following numbers:

a) 9 and 4

b) 12 and 5

c) 6 and 5

d) 15 and 4.

Solution. a) 9 and 4

LCM of 9 and 4 is 9 x 4 = 36

b) 12 and 5

LCM of 12 and 5 is 12 x 5 = 60

c) 6 and 5

LCM of 6 and 5 is 6 x 5 = 30

d) 15 and 4

LCM of 15 and 4 is 15 x 4 = 60

We observe a common property in the obtained LCM’s that the LCM is the product of two numbers in each case and also the LCM is always a multiple of 3.

Question 11. Find the LCM of the following numbers in which one number is the factor of the other.

a) 5,20

b) 6, 18

c) 12,48

d) 9,45

What do you observe in the results obtained?

Solution. a) 5, 20

5 = 5 x 1

20 = 2 × 2 × 5

LCM of 5 and 20 = 2 x 2 x 5 = 20

b) 6,18

6 = 2 × 3

18 = 2 x 3 x 3

LCM of 6 and 18 is 2 × 3 × 3 = 18

c) 12, 48

12 = 2 × 2 × 3

48 = 2 x 2 x 2 × 2 × 3

LCM of 12 and 48

= 2 × 2 × 2 × 2 × 3 = 48.

d) 9, 45

9 = 3 × 3

45 = 3 × 3 × 5

LCM of 9 and 45 = 3 x 3 x 5 = 45

In the results obtained, we observe that the LCM of the two numbers in which one number is a factor of the other is the greater number.

Haryana Board Class 6 Maths Solutions For  Chapter 3 Playing With Numbers Very Short Answer Questions

Question 1. Are the numbers 452, 673, 259, 356 divisible by 2? Verify.

Solution. A number is divisible by 2, if it has any of the digits 0, 2, 4, 6 and 8 in its ones place.

∴ The numbers 452 and 356 have 2 and 6 in their ones place and the other num- bers does not have.

∴ 452 and 356 are divisible by 2.

Question 2. Is 8430 divisible by 6? Why?

Solution. If a number is divisible by both 2 and 3 then it is also divisible by 6.

The ones place of 8430 is 0, so it is divisible by 2.

The sum of digits of 8430 is 8 + 4 + 3 + 0 = 15.

15 is multiple of 3. So 8430 is divisible by 3.

∴ The number 8430 is divisible by both 2 and 3. It is divisible by ‘6’.

Question 3. Test whether 6669 is divisible by 9.

Solution. The sum of digits of 6669

= 6 + 6 + 6 + 9 = 27

It is a multiple of 9. So the number 6669 is divisible by 9.

Question 4. Without actual division, find whether 8989794 is divisible by 9.

Solution. The sum of digits of 8989794

= 8 + 9 + 8 + 9 + 7 + 9 + 4 = 54

It is a multiple of 9. So the number 8989794 is divisible by 9.

Question 5. Find the smallest number that must be added to 128, so that it becomes exactly divisible by 5.

Solution. Given number is 128

If the number is exactly divisible by 5, its ones place should have the digits 5 or 0.

∴ The nearest number to 128 which is having 0 or 5 in one’s place is 130.

∴ The smallest number that must be added is 130 – 128 = 2

Question 6. What is the smallest prime number?

Solution. 2 is the smallest prime number.

Question 7. What is the smallest composite number?

Solution. 4 is the smallest composite number.

Question 8. What is the smallest odd prime number?

Solution. 3 is the smallest odd prime number.

Question 9. What is the smallest odd composite number?

Solution. 9 is the smallest odd composite number.

Question 10. Write 10 odd and 10 even composite numbers.

Solution. 10 odd composite numbers = 9, 15, 21, 25, 27, 33, 35, 39, 45, 49.

10 even composite numbers = 4, 6, 8, 10, 12, 14, 16, 18, 20, 22.

Haryana Board Class 6 Maths Solutions For  Chapter 3 Playing With Numbers Short Answer Questions

Question 11. Are the numbers 28570, 90875 divisible by 5? Verify by actual division also.

Solution. The number with zero or five at ones place is divisible by 5.

In the numbers 28570 and 90875; the digits at ones place are 0 and 5. So these numbers are divisible by 5.

Verifying by actual division:

Class 6 Maths Chapter 3 Playing With Numbers Short Answer Question 11

Question 12. Find the smallest number that has to be subtracted from 276 so that it becomes exactly divisible by 10.

Solution. Given number is 276

The digit in ones place is 6.

For the number exactly divisible by 10, the digit in ones place must be ‘0’

∴ The smallest number that has to be subtracted = 276 – 6 = 270

∴ The smallest number that has to be subtracted is 6.

Question 13. Write the greatest four digit number which is divisible by 9. Is it divisible by 3? What do you notice? 

Solution. The greatest four digit number which is divisible by 9 is 9999.

∴ The sum of digits = 9 + 9 + 9 + 9 = 36

36 is divisible by 3.

∴ 9999 is divisible by 3.

∴ Yes, the greatest four digit number which is divisible by 9 is also divisible by 3.

∴ We noticed that the numbers which are divisible by 9 are also divisible by 3.

Question 14. Write the nearest number to 12345 which is divisible by 4.

Solution. A number is divisible by 4 if the number formed by the last two digits is divisible by 4.

Given number is 12345.

Number formed by last two digits is 45.

If 1 is subtracted from it.

45 – 1 = 44 is divisible by 4.

So the nearest number to 12345 divisible by 4 is 12344.

Question 15. Find the factors of 60.

Solution. Factors of 60 = 1 × 60

= 2 x 30

= 3 x 20

= 4 x 15

= 5 x 12

= 6 x 10.

= 10 x 6

= 12 x 5

= 15 x 4

= 20 x 3

= 30 x 2

= 60 × 1

∴ The factors of 60 are = 1, 2, 3, 4, 5, 6, 10, 12, 15, 20, 30 and 60.

Chapter 3 Playing With Numbers Long Answer Questions

Question 16. Take any three 4 digit numbers and check whether they are divisible by 6.

Solution. Let us take three four digits numbers.

1) 3420 2) 7942 3) 6036

1) 3420

The ones place of 3420 is 0. So it is divisible by 2.

The sum of digits = 3 + 4 + 2 + 0 = 9.

It is divisible by 3.

∴ The number 3420 is divisible by both 2 and 3. So it is divisible by 6 also.

2) 7942

The ones place of 7942 is 2. So it is divisible by 2.

The sum of digits = 7 + 9 + 4 + 2 = 22.

It is not divisible by 3.

∴ The number 7942 is divisible by 2 but not divisible by 3. So it is not divisible by 6.

3) 6036

The ones place of 6036 is 6. So it is divisible by 2.

The sum of digits = 6 + 0 + 3 + 6 = 15.

It is divisible by 3.

∴ The number 6036 is divisible by both 2 and 3. So it is divisible by 6 also.

Question 17. Check whether the numbers 598, 864, 4782 and 8976 are divisible by 4. Use divisibility rule and verify by ac- tual division.

Solution. Divisibility rule of 4:

A number is divisible by 4 if the number formed by its last two digits. (i.e., ones and tens) is divisible by 4.

1) 598

The number formed by last two digits is 98.

98 is not divisible by 4.

∴ 598 is not divisible by 4.

2) 864

The number formed by last two digits is 64.

64 is divisible by 4.

∴ 864 is divisible by 4.

3) 4782

The number formed by last two digits is 82.

82 is not divisible by 4.

∴ 4782 is not divisible by 4.

4) 8976

The number formed by last two digits is 76.

76 is divisible by 4.

∴ 8976 is divisible by 4.

Question 18. Determine which of the following numbers are divisible by 5 and by 10.

25, 125, 250, 1250, 10205, 70985, 45880.

Check whether the numbers that are divisible by 10 are divisible by 2 and 5.

Solution. 25, 125, 10205, 70985, 250, 1250, 45880

These numbers are having ‘5’ or ‘0’ in the ones place.

∴ These numbers are divisible by 5

250, 1250, 45880

These numbers having ‘0’ in the ones place

∴ These numbers are divisible by 10

These numbers are divisible by ‘5’

these numbers are having ‘0’ in the ones place,

These numbers are also divisible by 2

∴ The numbers which are divisible by 10 are divisible by 2 and 5 also.

Difference between factors and multiples Class 6

Question 19. Write all the numbers between 100 and 200 which are divisible by 6.

Solution. The numbers divisible by 2 and 3 are divisible by 6.

The numbers divisible by 2 between 100 and 200 are = 102, 104, 106, 108, 110, 112, 114, 116, 118, 120, 122, 124 ………… 196, 198

The numbers divisible by 3 between 100 and 200 are = 102, 105, 108, 111, 114, 117, 120, 123, 126 …….. 195,198

The numbers which are divisible by both 2 and 3 are = 102, 108, 114, 120, 126, 132, 138, 144, 150, 156, 162, 168, 174, 180, 186, 192, 198

So all these numbers are divisible by 6.

Question 20. Using divisibility rules, determine which of the following numbers are divisible by 11.

1) 6446

2) 10934

3) 7138965

4) 726352

Solution. A number is divisible by 11, if the difference between the sum of the digits at odd places and the sum of the digits at even places (from the right) is either ‘0’ or multiple of 11.

1) 6446

The sum of digits at odd places = 6 + 4 = 10

The sum of digits at even places = 4 + 6 = 10

∴ Their difference = 10 – 10 = 0

So 6446 is divisible by 11.

2) 10934

The sum of digits at odd places = 4 + 9 + 1 = 14

The sum of digits at even places = 3 + 0 = 3

∴ Their difference = 14 3 = 11

It is a multiple of 11.

So 10934 is divisible by 11.

3) 7138965

The sum of digits at odd places = 5 + 9 + 3 + 7 = 24

The sum of digits at even places = 6 + 8 + 1 = 15

∴ Their difference = 24 – 15 = 9

It is not a multiple of 11.

So 7138965 is not divisible by 11.

4) 726352

The sum of digits at odd places = 2 + 3 + 2 = 7

The sum of digits at even places = 6 + 8 + 1 = 15

∴ Their difference = 18 – 7 = 1

It is a multiple of 11.

So 726352 is divisible by 11.

Haryana Board Class 6 Maths Solutions For  Chapter 3 Playing With Numbers Objective Type Questions

Choose the correct answer :

Question 1. In the following which is a composite number?

  1. 0
  2. 1
  3. 2
  4. 4

Answer. 4. 4

Question 2. Is 107 a prime number?

  1. yes
  2. no
  3. can’t say
  4. infinite

Answer. 1. yes

Question 3. Divide 29 by 4 it gives quotient

  1. 1
  2. 7
  3. 6
  4. 4

Answer. 2. 7

Question 4. A number is completely divisible by another number it leaves a remainder

  1. 1
  2. 2
  3. 3
  4. 0

Answer. 4. 0

Question 5. Which of the following numbers is divisible by 9?

  1. 197235
  2. 726352
  3. 988975
  4. 345880

Answer. 1. 197235

Question 6. Factors of 12 are

  1. 2,3
  2. 1,2,3
  3. 1,2,3,4,6
  4. 1, 2, 3, 4, 6, 12

Answer. 4. 1, 2, 3, 4, 6, 12

Question 7. The HCF of two consecutive numbers is

  1. 1
  2. The smallest number
  3. The greatest number
  4. 0

Answer. 1. 1

Question 8. The greatest two digit prime number.

  1. 77
  2. 79
  3. 97
  4. 99

Answer. 3. 97

Question 9. The number of factors of 80

  1. 8
  2. 10
  3. 11
  4. 12

Answer. 2. 10

Question 10. The another name for GCD

  1. LCM
  2. HCF
  3. GCM
  4. LCF

Answer. 2. HCF

Question 11. The number divisible by 4

  1. 200094
  2. 249874
  3. 249884
  4. 249854

Answer. 3. 249884

Question 12. The least two digit prime number

  1. 11
  2. 10
  3. 12
  4. 13

Answer. 1. 11

Question 13. 512215 is divisible by …..

  1. 2
  2. 4
  3. 11
  4. 6

Answer. 3. 11

Question 14. Example for twin prime numbers.

  1. (41,43)
  2. (42, 44)
  3. (43,45)
  4. (44, 46)

Answer. 1. (41,43)

Question 15. Number of common factors of 12 and 18 is

  1. 3
  2. 4
  3. 6
  4. 8

Answer. 2. 4

Question 16. The least prime number is

  1. 2
  2. 3
  3. 4
  4. 1

Answer. 1. 2

Question 17. 51225 is divisible by.

  1. 51
  2. 2
  3. 4
  4. 5

Answer. 4. 5

Question 18. HCF of 3, 4 is.

  1. 0.5
  2. 1
  3. 2
  4. 3

Answer. 2. 1

Question 19. Choose the correct matching.

Class 6 Maths Chapter 3 Playing With Numbers Choose the correct answer Question 19

  1. a-4, b-2, c-3, d-1
  2. a-1, b-4, c-2, d-3
  3. a-4, b-2, c-1, d-3
  4. a-2, b-4, c-1, d-3

Answer. 4. a-2, b-4, c-1, d-3

Question 20. Which of the following is not divisible by 2?

  1. 11150
  2. 63874
  3. 81111
  4. 1496

Answer. 3. 81111

Question 21. Greatest factor of 120 is………….

  1. 8
  2. 15
  3. 120
  4. 12

Answer. 3. 120

Question 22………… is neither prime nor composite.

  1. 4
  2. 3
  3. 1
  4. 2

Answer. 3. 1

Question 23. LCM of 8 and 12 is…………….

  1. 11
  2. 13
  3. 12
  4. 24

Answer. 4. 24

Question 24. HCF of 40, 56 and 60 is…………….

  1. 8
  2. 4
  3. 60
  4. 16

Answer. 2. 4

Question 25. How many prime numbers less than 100 are there?

  1. 15
  2. 25
  3. 30
  4. 26

Answer. 2. 25

Question 26. 42 = ……………..

  1. 7 × 3 × 2
  2. 7 × 7 x 9
  3. 2 × 21 × 3
  4. 7 × 6 × 4

Answer. 1. 7 x 3 x 2

Question 27. Identify a palindrome number.

  1. 1225
  2. 1331
  3. 11452
  4. 31119

Answer. 2. 1331

Question 28. 2 x 2 × 3 × 7 = ……….

  1. 84
  2. 62
  3. 81
  4. 22

Answer. 1. 84

Question 29. Number of factors of 36 is ………

  1. 6
  2. 11
  3. 10
  4. 9

Answer. 9

Question 30. The smallest odd composite number is …..

  1. 17
  2. 15
  3. 9
  4. 16

Answer. 3. 9

Question 31. Identify the correct statement from the following:

  1. 733 is divisible by 3
  2. The greatest prime number after 20 is 23
  3. 4, 6, 8, 10, 12 are even composite numbers
  4. 7221 is a factor of 11

Answer. 3. 4, 6, 8, 10, 12 are even composite numbers

Question 32. Factors of 115 are.

  1. 5
  2. 23
  3. 115
  4. All the above

Answer. All the above

Question 33. How many prime numbers are there in between 10 and 30?

  1. 9
  2. 6
  3. 8
  4. 19

Answer. 2. 6

Question 34. 9846 is not divisible by …….

  1. 9
  2. 3 only
  3. 2 only
  4. 5

Answer. 5

Question 35. Sum of the digits of 36129 is

  1. 16
  2. 3
  3. 21
  4. 8

Answer. 3. 21

Question 36. How many twin primes are below 20?

  1. 5
  2. 6
  3. 3
  4. 4

Answer. 4. 4

Question 37. 53 = …………..

  1. 13 + 17 + 20
  2. 13 + 17 + 23
  3. 18 + 10 + 16
  4. 17 + 13 + 33

Answer. 2. 13 + 17 + 23

Question 38. HCF of 28, 35 is ………

  1. 6
  2. 9
  3. 7
  4. 8

Answer. 7

Question 39. LCM of 10 and 11 is ……..

  1. 119
  2. 122
  3. 111
  4. 110

Answer. 4. 110

Question 40. LCM of a and b if HCF of a, b = 1.

  1. \(\frac{a}{b}\)
  2. a+b
  3. ab
  4. a-b

Answer. 3. ab

Question 41. Two prime numbers are said to be twin primes, if they differ each other by

  1. 0
  2. 1
  3. 2
  4. None

Answer. 3. 2

Question 42. Choose the correct matching.

Class 6 Maths Chapter 3 Playing With Numbers Choose the correct answer Question 42

  1. a-1, b-2, c-3
  2. a-3, b-2, c-1
  3. a-1, b-3, c-2
  4. a-2, b-3, c-1

Answer. 2. a-3, b-2, c-1

Question 43. The numbers which have 1 as the common factor are called…….

  1. co-prime
  2. relatively prime
  3. Both A & B
  4. None

Answer. 3. Both A & B

Question 44. The H.C.F. of any two twin primes is………….

  1. 0
  2. 1
  3. 2
  4. None

Answer. 2. 1

Question 45. The product of L.C.M and H.C.F of two numbers =

  1. Sum of two numbers
  2. Difference of two numbers
  3. Both A & B
  4. Product of two numbers

Answer. 4. Product of two numbers

Question 46. A number which divides the other number with out remainder is called

  1. factor
  2. multiple
  3. prime number
  4. composite number

Answer. 1. factor

Question 47. The number which is divisible by 1 and itself is called

  1. factor
  2. multiple
  3. prime number
  4. composite number

Answer. 3. Prime number

Question 48. If the L.C.M of 8 and 12 is 24 then their H.C.F. is

  1. 3
  2. 4
  3. 16
  4. 96

Answer. 2. 4

Question 49. If the H.C.F. of 18 and 27 is 9 then their L.C.M is

  1. 9
  2. 486
  3. 54
  4. None

Answer. 3. 54

Question 50. ……………. is a factor of 12.

  1. 7
  2. 9
  3. 4
  4. 18

Answer. 3. 4

Question 51. ……………… is a factor of every number.

  1. 3
  2. 12
  3. 0
  4. 1

Answer. 4. 1

Question 52. The fifth multiple of 6 is.

  1. 30
  2. 35
  3. 40
  4. 45

Answer. 1. 30

Question 53. Every prime number except ………. is odd.

  1. 3
  2. 4
  3. 1
  4. 2

Answer. 4. 2

Question 54. 128 is divisible by

  1. 4
  2. 3
  3. 5
  4. 9

Answer. 1. 4

Question 55. 2 x 2 x 2 x 2 = ……….

  1. 18
  2. 8
  3. 12
  4. 16

Answer. 4. 16

Question 56. HCF of 24 and 36 is ….

  1. 14
  2. 36
  3. 12
  4. 24

Answer. 3. 12

Question 57. LCM of 24 and 90 is …..

  1. 360
  2. 300
  3. 460
  4. 306

Answer. 1. 360

Question 58. Which of the following number is divisible by 2?

  1. 133
  2. 124
  3. 135
  4. 111

Answer. 2. 124

Question 59. Two numbers having only 1 as a common factor are called …………..numbers.

  1. co-prime
  2. prime
  3. odd
  4. even

Answer. 1. co-prime

Question 60. Which of the following number is not divisible by 5?

  1. 140
  2. 222
  3. 1000
  4. 110
  5. Answer. 2. 222

Haryana Board Class 6 Maths Solutions For  Chapter 3 Playing With Numbers Fill In The Blanks:

Question 61. HCF of 2021 and 2022 is

Answer. 1

Question 62. The least composite number is ………..

Answer. 4

Question 63. Number of primes below 98 is ………..

Answer. 25

Question 64. LCM of 2, 4, 6 ………………..

Answer. 12

Question 65. HCF of 3, 5, 19 is …………

Answer. 1

Question 66.

Class 6 Maths Chapter 3 Playing With Numbers Fill In The Blanks Question 66

In the above factor tree of 112 the value of x = ………..

Answer. 7

Question 67. HCF of 30 and 48 is ……………

Answer. 6

Question 68. HCF of two consecutive even numbers is ………….

Answer. 2

Question 69. Express 44 as sum of two odd primes ………….

Answer. 13 + 31

Question 70. Number of factors of 80 is …………..

Answer. 10

Question 71. 2, 4, 6, 8 are examples of ………….numbers.

Answer. even

Question 72. 2 × 3 + 1 = …………

Answer. 7

Question 73. …………….. is neither a prime nor a composite number

Answer. 1

Question 74. 4 × 17 = ……………..

Answer. 68

Question 75. Number of factors of a given number are.

Answer. finite

Question 76. Every number is a factor of ………..

Answer. itself

Question 77. 1, 2, 4 are called …………………..

Answer. factors

Question 78. Sum of the digits of a number is divisible by 3, then it is divisible by

Answer. 3

Question 79. LCM of 8 and 12 is. ……….

Answer. 24

Question 80. HCF of any two consecutive numbers is ………..

Answer. 1

Question 81. LCM means ………….

Answer. Least Common Multiple